You are on page 1of 104

G.

R.No.166519 Mar
ch31,
2009

NI
EVESPLASABASandMARCOSMALAZARTE,
Pet
it
ioner
s,

v
s.

COURTOFAPPEALS( Speci
alFor
merNi
nthDi
vi
sion)
,DOMI
NADORLUMEN,
andAURORA
AUNZO,
Respondent
s.

DECISION

NACHURA,
J.:

Assail
edi
nthi
spet i
ti
onforrevi
ewoncer t
ior
ariunderRul
e45oftheRulesofCour
tar
etheMay
12,2004Deci
sion1oftheCourtofAppeal
s(CA)i nCA-G.
R.CVNo.43085andtheDecember1,
2004Resol
uti
on2deny i
ngreconsi
derat
ionofthechall
engeddeci
sion.

Theper
ti
nentf
act
sandpr
oceedi
ngsf
oll
ow.

In1974,pet i
ti
oners3fi
ledacompl aintforr ecoveryoft i
tl
et oproper
tywithdamagesbef orethe
CourtofFi r
stInst
ance( now, Regi
onal Trial Court[RTC])ofMaasi n,Sout
hernLey t
eagai nst
respondents.Thecasewasdocket edasCi v i
lCaseNo.R- 1949.Thepropertysubjectoft hecase
wasapar celofcoconutl andi nCant ur
ing, Maasi n,SouthernLeyte,
declaredunderTax
Declarat
ionNo.3587i nt henameofpet i
tionerNi eveswi t
hanar eaof2.6360hect ares.4I nthei
r
complaint,peti
ti
onerspr ayedt hatjudgmentber enderedconf ir
mingtheirri
ghtsandl egal ti
tl
eto
thesubjectpropertyandor deri
ngthedef endant stov acatetheoccupiedpor t
ionandt opay
damages. 5

Respondents,f
ortheirpar
t,deniedpeti
tioners’al
l
egationofowner
shi
pandpossessi
onofthe
premises,andint
erposed,astheirmaindef ense,
thatthesubj
ectl
andwasi
nheri
tedbyal
lthe
part
iesfrom t
heircommonancest or
,FranciscoPlasabas.
6

Revealedi nthecourseoft hetri


alwasthatpeti
ti
onerNieves,cont
rar
ytoherall
egationsinthe
compl aint
,wasnott hesol eandabsoluteowneroftheland.Basedonthetesti
moniesof
petit
ioners’wit
nesses, t
hepr opert
ypassedonfrom Franci
scot ohi
sson,Leonci
o;thentoJov i
ta
Talam, peti
ti
onerNiev es’grandmother
;thentoAntoni
naTal am,hermother
;andthent oherand
hersibli
ngs—Jose, VictorandVi ct
ori
a.7

Aft
erresti
ngthei
rcase,
respondent
sraisedinthei
rmemor andum t
heargumentthatt
hecase
shouldhavebeenter
minatedati
ncept
ionf orpet
it
ioners’
fail
uretoimpl
eadindi
spensabl
e
part
ies,
theotherco-
owners–Jose,VictorandVictor
ia.

I
nitsApri
l19,1993Order,
8thet
ri
alcour
t,wi
thoutr
uli
ngont
hemer
it
s,di
smi
ssedt
hecase
wi
thoutprej
udice,
thus:

Thi
sCour
t,muchasi
twant
stodeci
det
hei
nst
antcaseont
hemer
it
s,bei
ngoneoft
heol
d
i
nherit
edcasesleftbehi
nd,fi
ndsdiff
icul
tyifnotimpossibi
li
tyofdoi
ngsoatthisstageofthe
proceedingswhenbothparti
eshavealreadyrestedt
heircases.Rel
uct
antl
y,i
tagreeswiththe
defendantsint
heobservat
ionthatsomei mportanti
ndispensabl
econsi
derat
ionis
conspicuousl
ywantingormissi
ng.

Iti
snottheCourt’
swishtot
urnit
sbackonthecr uci
alpartoft
hecase,whi
chisthe
pronouncementofthej
udgmenttoset
tl
etheissuesraisedint
hepleadi
ngsofthepar
tiesonce
andforall
,af
teral
lthet
ime,ef
for
tandexpensespentingoingthr
oughthetr
ialpr
ocess.

But
,rul
esarer
ules.Theyhavetobef
oll
owed,
toar
ri
veataf
airandj
ustv
erdi
ct.Sect
ion7,
Rul
e3
oft
heRulesofCourtprov
ides:

"xxxCompul
soryj
oinderofi
ndi
spensabl
epart
ies.–Part
iesi
ninterestwi
thoutwhom nofi
nal
det
ermi
nat
ioncanbehadofanactionshal
lbejoi
nedeit
herasplainti
ff
sordefendant
s."

Whatt heCour twant


stosayhereisthattheinstantcaseshoul dhavebeendismissedwithout
prejudiceal ongti
meagof orl
ackofcauseofact i
onasthepl aint
if
fsspousesMar cosMalazart
e
andNi ev esPlasabasMalazar
tehavenocompl etelegalpersonali
tytosuebyt hemselv
esalone
withoutj oi
ningthebrot
hersandsist
ersofNiev eswhoar easI NDISPENSABLEast helatt
erin
thef i
nal deter
minati
onofthecase.Notimpleadingthem, anyjudgmentwouldhav eno
effecti
v eness.

Theyarethatindi
spensablethatafi
naldecr
eewouldnecessar
il
yaffectt
heirr
ights,sothatt
he
Courtcannotproceedwithoutthei
rpr
esence.Ther
eareabundantauthor
it
iesi
nt hisregar
d.
Thus–

"Thegener al r
ulewi thr efer encet othemaki ngofpar ti
esinaci vi
lacti
onrequirest hejoinderof
alli
ndispensabl epar t
iesunderanyandal lcondit
ions,theirpresencebeingasi nequanonoft he
exerciseofj udi
cial power .(Bor l
asav .Pol i
sti
co,47Phi l.345,348)Fort hi
sr eason, ourSupreme
Courthashel dthatwheni tappear sofr ecordthatt hereareot herpersonsinterestedi nthe
subjectmat t
eroft hel it
igat ion,whoar enotmadepar t
iest otheaction,i
tisthedut yoft hecourt
tosuspendt hetrial untilsuchpar ti
esar emadeei therplainti
ffsordefendants.( Pobre,etal.v.
Blanco, 17Phi l
.156) .xxxWher ethepet it
ionfail
edt ojoinaspar tydefendantt heper son
i
nterest edinsustaini ngt hepr oceedingi nthecour t,t
hesameshoul dbedismi ssed.xxxWhen
anindi spensablepar t
yi snotbef oret hecour t
,theact i
onshoul dbedismissed.( People,etal.v.
Rodriguez, etal.
,G.R.Nos.L- 14059-62, September30, 1959)( si
c)

"Parti
esininterestwi thoutwhom nof i
naldet erminati
oncanbehadofanact ionshallbejoined
eit
herasplai nt
iffsordef endants.(Sec.7,Rule3, Rul
esofCour t)
.Theburdenofpr ocuri
ngthe
presenceofal lindispensablepartiesisont hepl aint
if
f.(39Amj ur[si
c]885).Theev i
dent
purposeoft her uleist opreventt
hemul t
ipli
cityofsuitsbyr equiri
ngthepersonarresti
ngar ight
againstt
hedef endantt oincl
udewi t
hhim, eit
herasco- plai
ntif
fsorasco-defendants,al
l
personsstandingi nt hesameposi ti
on,sot hatthewhol emat t
erindisput
emaybedet ermined
onceandf orall inonel it
igat
ion.(Palar
cav .Bagi nsi,
38Phi l.177,178).
"Anindi
spensablepar t
yisapartywhohassuchani nt
erestint
hecontr
ov er
syorsubj ectmatt
er
thatafi
nal adj
udicat
ioncannotbemade, inhisabsence,wit
houti
nqui
ringoraffect
ingsuch
i
nterest
;apar t
ywhohasnotonl yani nter
estofsuchanat uret
hatafi
naldecreecannotbe
madewi thoutaff
ectinghisi
nterestorleavingthecontr
oversyi
nsuchacondi ti
onthatitsfi
nal
determi
nat i
onmaybewhol l
yinconsistentwithequi
tyandgoodconscience.(67C.J.S.892).
I
ndispensableparti
esar et
hosewi thoutwhom noact i
oncanbef i
nal
lydetermined.
"(Sanidadv.
Cabataj
e, 5Phil
.204)

WHEREFORE,I
NVI EW OFALLTHEFOREGOI
NGCONSIDERATI
ONS,boththecomplai
ntandthe
count
ercl
aimi
ntheinst
antcasear
eor
der
edDISMI
SSEDwit
houtpr
ejudi
ce.Nopronouncement
astocost
s.

SOORDERED.
9

Aggriev
ed, peti
ti
onerselev
atedthecaset otheCA.InthechallengedMay12, 2004Decisi
on,10
theappell
at ecourtaf
fir
medt herul
ingofthetri
alcourt
.TheCA, f
urt
her,declaredt
hatthenon-
j
oinderofthei ndi
spensablepart
ieswouldviol
atetheprinci
pleofduepr ocess,andthatArt
icle
487oft heCi vi
lCodecouldnotbeappl i
edconsider
ingthatthecompl ai
ntwasnotf oreject
ment ,
butforrecov er
yoftit
leorareivi
ndicat
oryacti
on.11

Witht
heirmot
ionforr
econsi
derat
iondeni
edinthefur
therassai
l
edDecember1,
2004
Resol
uti
on,
12peti
ti
onersf
il
edtheinst
antpet
it
ion.

TheCourtgr
ant
sthepet
it
ionandr
emandst
hecaset
othet
ri
alcour
tfordi
sposi
ti
onont
he
meri
ts.

Arti
cle487oftheCi vi
lCodepr ovidesthatanyoneoft heco-ownersmaybr i
nganact i
onfor
ej
ectment .
1avvphi1.
zw+Thear t
iclecoversallkindsofact i
onsfort
herecoveryofpossession,
i
ncludinganacci onpubli
ci
anaandar eivi
ndicatoryaction.Aco-ownermayfilesuitwithout
necessaril
yjoi
ningallt
heotherco- ownersasco- pl
aint
iff
sbecausethesuiti
sdeemedt obe
i
nstit
utedforthebenefitofal
l.Anyj udgmentoft hecourtinfav
oroftheplai
ntif
fwi l
lbenefi
tthe
otherco-owners,buti
fthejudgmenti sadv erse,thesamecannotpr ej
udi
cet herightsofthe
unimpleadedco-owners.13

Wit
ht hi
sdi
squi
sit
ion,
therei
snoneedtodet
erminewhet
herpet
it
ioners’
complai
nti
sonefor
ej
ectmentorf
orrecover
yofti
tl
e.Tor
epeat
,Art
icl
e487oftheCi
vilCodeappl
i
estobothacti
ons.

Thus,peti
ti
oner
s,intheircomplai
nt,
donothav etoimpl
eadt hei
rco-owner
sasparti
es.Theonl
y
excepti
ontothi
sruleiswhent heacti
onisforthebenef
itoftheplai
nti
ffal
onewhoclaimstobe
thesoleownerandis,thus,ent
itl
edtothepossessi
onthereof.I
nsuchacase, t
heact
ionwil
lnot
prosperunl
esstheplainti
ffi
mpleadstheotherco-
ownerswhoar eindi
spensabl
epart
ies.
14

Her
e,theal
legat
ionofpetit
ionersi
nt hei
rcomplaintt
hattheyaret
hesoleownersofthepr oper
ty
i
nli
ti
gati
onisimmat er
ial
,consider
ingthattheyacknowledgeddur
ingthet
ri
alt
hatthepr opert
y
i
sco-ownedbyNi ev
esandhersi bl
ings,andthatpeti
ti
onershavebeenaut
hori
zedbyt heco-
ownerst
opursuet hecaseonthelat
ter
’sbehal
f.15I
mpleadi
ngtheot
herco-
owner
sis,t
her
efor
e,
notmandat
ory,because,asmenti
onedearl
ier
,thesui
tisdeemedtobei
nsti
tut
edf
orthebenef
it
ofal
l.

Inanyev ent
,thetri
alandappel lat
ecour t
scommi tt
edr eversi
bleerrorwhent heysummar i
ly
dismissedthecase,afterbothpar ti
eshadr estedt hei
rcasesf ollowingapr otract
edt rial
commenci ngin1974, ont hesolegroundoff ailuretoimpl eadindispensablepar t
ies.Ther ul
eis
sett
ledthatthenon-joi
nderofi ndi
spensablepar t
iesisnotagr oundf ort
hedismi ssal ofan
acti
on.Ther emedyist oimpleadt henon-partyclaimedt obei ndispensable.Par t
iesmaybe
addedbyor derofthecour tonmot i
onofthepar t
yoroni tsowni nit
iati
veatanyst ageoft he
acti
onand/ oratsucht i
mesasar ejust.I
fpetiti
onerr efusestoi mpleadani ndispensabl epart
y
despitetheorderofthecour t
,thelattermaydi smi ssthecompl aint/peti
ti
onf orthe
plai
ntif
f’
s/peti
ti
oner'
sf ai
luretocompl ytherewith.16

WHEREFORE,
premisesconsidered,
thei
nstantpetit
ionisGRANTED, andthecasei
s
REMANDEDtothet
rialcour
tforappropr
iat
epr oceedings.Thet
ri
alcourti
sfurt
herDI
RECTEDt
o
deci
deont
hemerit
soft heci
v i
lcaseWITHDI SPATCH.

SOORDERED.

ANTONI
OEDUARDOB.NACHURA

Associ
ateJust
ice

WECONCUR:

G.
R.No.165142       December10,
2007

EDUARDOL.RAYO,
 
Pet
it
ioner

v
s.

METROPOLI
TANBANKANDTRUSTCOMPANYANDBRANCH223OFTHEREGI
ONALTRI
AL
COURTOFQUEZONCI
TY,
 Respondent
s.

DECISION

QUI
SUMBI
NG,
 
J.:

Befor
eusisapet
iti
onforr
evi
ewassai
li
ngtheResol
uti
onsdatedJune15,
20041 
andAugust23,
20042 
oft
heCourtofAppeal
sinCA-
G.R.SPNo.83895forannul
mentofj
udgment
.

Theper
ti
nentf
act
sar
eundi
sput
ed.

MidasDiversif
iedExportCorp.(
Midas)
,thrui
tspresi
dent
,Mr.SamuelU.Lee,obtai
nedsi
x(6)
l
oansfrom pr i
vater
espondentMetropol
itanBankandTrustCompany(Metrobank)
,amounti
ng
t
o₱588, 870,000asev i
dencedbypromissorynot
es.Tosecurethepay
mentofan₱8, 000,
000
l
oan, Loui
svil
leRealty&Dev el
opmentCor porati
on(Louisv
il
le)
,thruit
spresident
,Mr.SamuelU.
Lee,executedinfavorofMet robank,ar
ealestatemortgageovert hr
eeparcelsofl
andsituat
ed
atNo.40Ti mogAv e.
,Brgy.LagingHanda,QuezonCi t
y,wit
hallthebuil
dingsandimprovements
ther
eon.Thepr operti
esarecov eredbyTransferCert
if
icat
esofTi t
le(TCT)Nos.N-163455,N-
166349andN- 166350issuedbyt heRegist
ryofDeedsofQuezonCi ty.

Whent hedebt or
-mor tgagorfai
ledtopay, Metrobankext ra-j
udiciall
yforeclosedt herealestate
mor tgagei naccordancewi thActNo.3135, 3 asamended.Ther eafter
,inapubl icauction,
Met robankwast hehi ghestbidder.ACertifi
cateofSal e4 datedDecember11, 2000wasdul y
registeredwi t
htheRegi str
yofDeedsofQuezonCi tyonDecember13, 2000.WhenLoui svi
ll
e
refusedt oturnovert herealpropert
ies,
onMar ch17, 2001, Met robankf i
l
edbef or et
heRegi onal
TrialCour t(RTC)
,Br anch223, QuezonCity,an expar t
e petit
ion5 fortheissuanceofawr i
tof
possessi ondocket edasLRCCaseNo.Q- 13915( 01).Afterpresent ati
onofev idence exparte,the
RTCgr antedthepet i
tioninanOr der
6 datedJul y5,2001, thedisposi t
iveporti
onofwhi chreads
asf oll
ows:

WHEREFORE, i
nconsiderat
ionoft
hef oregoingpremises,theinstantpet i
ti
onishereby
GRANTED.Upont hefi
li
ngofabondi ntheamountofONEHUNDREDTHOUSANDPESOS
([
₱]100,
000.00),l
etaWr i
tofPossessionov erthepropert
iescov eredbyTr ansferCert
if
icatesof
Tit
leNos.N-163455,N-166349&N- 166350i ssueinfavorofthepet iti
onerMETROPOLI TAN
BANK&TRUSTCOMPANYt obeimplement edbyt heDeput ySheriffofBranch223, Regional
Tri
alCourtofQuezonCi t
ybyplaci
ngt hepet i
ti
onerinpossessionov ertheparcel
soflandwi th
al
litsi
mpr ov
ement s.

SOORDERED.
7

OnSept ember24, 2001, Metr


obankpost edtherequi
redbond.Consequentl
y,awritof
possessionwasi ssuedonOct ober9,2001.Thi
swaspar ti
all
yimplementedastoTCTNo.N-
163455, asevidencedbyt heTurn-OverReceipt
8 dat
edDecember13, 2002.Thewr i
tovert
he
twor emainingpropert
ies,underTCTNos.N- 166349andN- 166350,weresubsequentl
y
i
mpl ement edasev i
dencedbyt heTur n-
OverReceipt
9 dat
edDecember3, 2003.

Meanwhil
e,onApril
3, 2002,peti
ti
onerEduar
doL.Ray ofi
l
edacompl ai
nt10 
docket
edasCivil
CaseNo.Q02-46514agai nstMetrobankf
orNull
i
ficat
ionofRealEst
ateMortgageCont
ract
(s)
andExt
raj
udici
alForeclosureSal
e, i
ntheRTC,Br
anch99, QuezonCi
ty.

OnMay13, 2004,peti
ti
onerRayof il
edwi t
htheCourtofAppeal
saPet i
ti
on11 f
orAnnulmentof
Judgmentont hegroundof" absolutelackofdueprocess.
"Pet
it
ioneral
legedthathi
s
predecessor,
Louisvi
ll
e,wasnotnot ifi
edoftheproceedi
ngsandthatSecti
on712 (ex
parte 
mot i
onorpetit
ionfort
hei ssuanceofawr i
tofpossessi
on)ofActNo.3135i s
unconstit
uti
onal.

OnJune15,2004,t
heCour tofAppeal
sdeniedthepeti
ti
onforl
ackofmer i
t.TheCourtof
Appeal
srul
edthatpet
it
ionerisnei
thert
heregist
eredownernorthesuccessor
-i
n-i
nter
estoft
he
regi
ster
edowner;hence,notarealpar ty
-i
n-i
nter
est.I
talsoruledthatt
herei
snobasi sto
chall
engetheconst
it
utional
it
yofSect ion7ofActNo.3135, asamendedasi tconsti
tut
esa
coll
ater
alatt
ackagai
nstsaidprov i
sion.Furt
her,pet
iti
oneravail
edofthewrongr emedyinf
il
ing
Civi
lCaseNo.Q02-46514.Petit
ionersoughtreconsiderat
ion,butwasli
kewisedenied.

Pet
it
ionernowcomesbef
oreusr
aisi
ngt
hef
oll
owi
ngaspr
imar
yissue:

WHETHERORNOTSECTI ON7OFACTNO.3135I SCONTRARYTOTHEDUEPROCESS


PROVI
SIONOFTHEPHI LI
PPINECONSTI TUTI
ONCONSIDERINGTHATSUCHSECTION7OF
THELAW PROVIDESORALLOWS, ACCORDINGTOTHI SHONORABLECOURT,FORANEX-
PARTEPROCEEDINGWHICHI SA"JUDICIALPROCEEDI
NGBROUGHTFORTHEBENEFITOF
ONEPARTYONLY, ANDWITHOUTNOTI CETO, ORCONSENTBYANYPERSONADVERSELY
I
NTERESTED""ORAPROCEEDINGWHEREI NRELI
EFISGRANTEDWITHOUTANOPPORTUNITY
FORTHEPERSONAGAI NSTWHOM THERELI EFI
SSOUGHTTOBEHEARD," 
ASHELDINTHE
CASEOF GOVERNMENTSERVI CEINSURANCESYSTEM VS.COURTOFAPPEALS,
169SCRA
244@ 255,
JANUARY20,1989.13

Heal
sor
aisest
hef
oll
owi
ngassecondar
yissues:

I
.

WHETHERORNOTTHEPETITI
ONERHASTHELEGALPERSONALITYTOSEEKTHE
ANNULMENTOFJUDGMENTIN[THE]SUBJECTLRCCASENO.Q-
13915(
01)
.

I
I.

WHETHERORNOTPRI VATERESPONDENTVI
OLATEDTHERULEAGAI NSTFORUM-SHOPPING
WHENITDIDNOTINFORM THEHONORABLEBRANCH223OFTHEREGI ONALTRI
ALCOURT
OFQUEZONCITYREGARDINGTHEFILI
NGOFCIVI
LCASENO.Q-02-46514FORNULLI
FICATI
ON
OFREALESTATEMORTGAGECONTRACTANDTHEEXTRA-JUDICIALFORECLOSURESALEOF
THESAMESUBJECTREALPROPERTIESANDTHEPENDENCYOFTHESAMEBEFORETHE
HONORABLEBRANCH99OFTHESAMEREGI ONALTRI
ALCOURT.14

Stat
edsimply,
thei
ssuesrai
sedare:(1)Doespetit
ionerhavethel
egal
personal
i
tyint
he
annul
mentofjudgmentpr
oceedings?(2)I
sSection7ofActNo.3135, asamended,
unconst
it
uti
onal?(
3)I
srespondentguil
tyoffor
um- shopping?

Peti
ti
onerinsi
ststhatcontr
arytotherul
ingoftheCourtofAppeals,hehasthelegal
per
sonali
ty
toi
nstit
utetheannulmentofjudgmentcaseagainstMetrobank,consi
deringt
hattheMarch25,
2002deedofassi gnmentheenteredint
owi t
hLouisv
il
leandWi nstonLinwyL.Chuamakeshim
aco-assi
gneeov erthesubj
ectrealpr
operti
es.

Fori
tspart
,Metr
obankclai
mst hatitwasnotapar t
ytothedeedofassi
gnmentamong
Loui
svi
ll
e,Chuaandpeti
ti
oner,
hence, ithasnopri
vi
tyofcontr
actwit
hpeti
ti
onerRayo.Mor
eover
,
Metr
obankpoint
soutthattherealproperti
eshadal
readybeenext
raj
udi
cial
lyf
orecl
osedwhen
pet
it
ionerandhi
sassi
gnor
sexecut
edt
hedeedofassi
gnment
.

UnderSect
ion2,15 Rule3oftheRul esofCour t
,everyactionmustbepr osecutedordef endedin
thenameoft herealpart
y-i
n-i
nt er
est,
orone" whost andstobebenef i
tedorinjuredbyt he
j
udgmentinthesui t
."
16 Areal part
y-i
n-i
nterestisonewi th"apresentsubstanti
al i
nterest"which
meanssuchinterestofapar t
yi nthesubjectmat t
eroft heact i
onaswi l
lenti
tl
ehi m, underthe
subst
anti
velaw,torecoveriftheev i
denceissuf fi
cient,orthathehasthelegal t
itl
eto
demand.17

Now,ispetiti
onerRay oarealparty
-i
n-i
nt er
est?Ini
tial
l
y,werecognizedher
einpeti
tionerasthe
co-
assigneeoft hesubjectr
ealpropertiesasshowni ntheMar ch25,2002deedofassi gnment.
However,whi l
epeti
ti
onerwoul dbei nj
uredbyt hejudgmentinthi
ssuit,wefi
ndthatpet i
ti
oner
hasnopr esentsubstanti
ali
nteresttoinsti
tut
et heannulmentofjudgmentproceedingsand
nul
li
fytheor dergr
antingthewritofpossession.

Fir
st,therewasnov i
olat i
onofpet it
ioner’
srightt
oconst i
tut
ionalduepr ocess.Inalongl i
neof
cases,18 wehav econsi stentl
yruledthattheissuanceofawr itofpossessioni nfavorofthe
purchaserinaf orecl
osur esaleofamor tgagedpropertyunderSection7ofActNo.3135, as
amendedi sami nist
erialdutyofthecour t.Thepurchaseroftheforeclosedpr opert
y,upon ex
parteappli
cati
onandt hepost i
ngoft herequi r
edbond,hast heri
ghttoacqui repossessionofthe
for
ecl osedpropertydur i
ngt he12-mont hredemptionperiodandwi t
hmor ereason,afterthe
expirati
onofther edempt ionperiod.

An expar te peti
ti
onf ortheissuanceofawr i
tofpossessi onunderSection7ofActNo.3135i s
not,str
ictl
yspeaki ng, a"j
udicial pr
ocess"ascont emplatedinAr t
icl
e43319  oftheCi vi
lCode.Itis
ajudicialproceedingf ortheenf orcementofone’ sri
ghtofpossessi onaspur chaserina
for
eclosur esale.Itisnotanor dinarysuitfil
edincour t
,bywhi chonepar ty"suesanot herforthe
enforcementofawr ongorpr otecti
onofar ight
,orthepr eventi
onorredr essofawr ong."I
tisa
non-li
ti
giouspr oceedi ngauthor i
zedi nanext raj
udici
al f
oreclosur
eofmor tgagepur suanttoAct
No.3135, asamended, andisbr oughtforthebenef i
tofonepar t
yonly,andwi t
houtnot i
ceto,or
consentbyanyper sonadv erselyinterested.Iti
sapr oceedingwher ether eli
efisgranted
withoutrequi ri
nganoppor t
uni tyfortheper sonagainstwhom t hereli
efissoughtt obeheard.
Nonot icei sneededt obeser v eduponper sonsinterestedinthesubjectpr operty
.20

Second, i
nthedeedofassi gnment ,peti
ti
oneralsoacknowl edgedt hatt hesubjectreal
propertieswer ealr
eadysoldatv ari
ousext r
ajudicialforeclosuresal esandboughtbyMet robank.
Clearl
y, peti
ti
onerrecognizedt hepr i
orexist
ingr i
ghtofMet r
obankast hemor tgagee-pur
chaser
overthesubj ectrealpropert
ies.21 Actualknowl edgeofapr i
ormor t gagewi t
hMet robankis
equivalenttonoticeofr egi
strati
on22  i
naccordancewi thAr ti
cle212523  oftheCivilCode.
Conformabl ywi t
hAr ti
cles131224  and212625  oft heCi v
ilCode, ar eal r
ightorli
eninf av
orof
Met r
obankhadal readybeenest ablished,subsisti
ngov erthepr oper ti
esunt i
lthedischargeof
theprincipalobli
gation,whoev erthepossessor (s)oft helandmi ghtbe. 26 Aspetit
ionerisnota
partywhosei nt
erestisadverset othatofLoui svil
le,therewasnobart ot hei
ssuanceofawr i
tof
possessi
ontoMetr
obank.I
tdoesnotmatt
erthatpeti
ti
onerwasnotspeci
fi
cal
l
ynamedi
nthe
wri
tofpossessi
onnornoti
fi
edofsuchpr
oceedings.
1avvphi
1

Thir
d,weal sonotethatpeti
ti
oneravai
l
edoft hewrongr emedyi nfi
li
ngCivi
lCaseNo.Q02-
46514,fornull
if
icat
ionofrealest
atemortgageandext r
ajudi
cialfor
eclosur
esale,mor
ethansi
x
(6)monthsaf t
ertheissuanceofthewri
tofpossessionconsideringthemandateofSect
ion
827 ofActNo.3135, asamended.Hence,evenpeti
ti
oner ’
sactionforannulmentofj
udgment
cannotprosperasitcannotbeasubstit
uteforalostremedy.

Now, peti
ti
onerischal
lengingtheconsti
tut
ionali
tyofSect
ion7ofActNo.3135, asamended.He
aversthatSecti
on7v i
olatesthedueprocessclausebecause,bythemer ef
il
ingofan 
ex
partemoti
oninthepropercadast r
alcour
t,t
hepur chaseri
naforeclosur
esaleisal
lowedt
o
obtainpossessi
onofthef orecl
osedpropertyduri
ngtheredempti
onper i
od.

TheCourtofAppealsruledt hatpet i
ti
oner’
sat
tempttochall
engetheconsti
tuti
onali
tyofSecti
on
7ofActNo.3135, asamended, consti
tut
esacoll
ater
alat
tackthati
snotallowed.Wef ul
lyagr
ee
wit
ht heappell
atecour
t’sruling.Forreasonsofpubli
cpol
icy,
theconsti
tut
ional
ityofalaw
cannotbeattackedcol
laterally
.28

Withregardtof orum-shopping;forum-shoppi ngi sthef i


l
ingofmul tipl
esuitsinvolvingt hesame
parti
esforthesamecauseofact ion,eithersimul t
aneousl yorsuccessivel
y,forthepur poseof
obtai
ningaf avorablejudgment. 
Itexistswher et heelement sof 
li
ti
spendent i
a arepr esentor
whereaf i
nal j
udgmenti nonecasewi l
lamountt o r
esj udicata 
i
nanot her.
29 Thei ssuanceoft he
wri
tofpossessi onbei ngami ni
sterialfuncti
on, andsummar yinnature,i
tcannotbesai dt obea
j
udgmentont hemer i
ts.Iti
sonlyani ncidentint hetransferoftitl
e.Hence,asepar atecasef or
annulmentofmor tgageandf or
eclosuresal ecannotbebar redby l
iti
spendent i
a or res
j
udicata.
30 Clearly,i
nsofarasLRCCaseNo.Q- 13915( 01)andCi vilCaseNo.Q02- 46514ar e
concerned,Met robankisnotguiltyoffor um-shopping.

WHEREFORE,thepet
it
ionisDENI
EDf orl
ackofmeri
t.Theassai
l
edResol
uti
onsdat
edJune15,
2004andAugust23,
2004oftheCourtofAppeal
sinCA-G.
R.SPNo.83895areher
eby
AFFI
RMED.Costsagainstt
hepet
it
ioner.

SOORDERED.

LEONARDOA.QUI
SUMBI
NG

Associ
ateJust
ice

WECONCUR:

G.
R.No.186305,
Jul
y22,
2015

V-
GENT,
INC.
,Pet
it
ioner
,v.MORNI
NGSTARTRAVELANDTOURS,
INC.
,Respondent
.

DECISION
BRI
ON,
J.:

Wer esol
vethepet
it
ionforrevi
ewoncerti
orar
ifi
l
edtochal
l
engetheNovember11,2008
Deci
sionandFebruary5,2009Resol
uti
onoftheCour
tofAppeal
s(CA)i
nCA-G.R.SPNo.
97032.1chanr
obl
esl
aw

ANTECEDENTS

Someti
mei nJuneandinSeptember1998,t
hepet
iti
onerV-Gent
,Inc.(
V-Gent
)boughttwent
y-si
x
(26)
2two-wayplaneti
cket
s(Manil
a-Eur
ope-
Manil
a)from t
herespondentMorni
ngStarTrav
el
andTours,I
nc.(
Mor ni
ngStar
).

OnJune24, 1998andSeptember28,1998,V-Gentret
urnedatotaloff
if
teen(15)unusedt
icket
s
wor t
h$8, 747.
50tothedefendant
.Ofthe15,MorningStarref
undedonlysix(
6)ticket
sworth
$3,445.62,Morni
ngStarrefusedt
oref
undt heremaini
ngnine(9)unusedti
cketsdespit
e
repeateddemands.

OnDecember15, 2000,peti
ti
onerV-Gentfi
ledamoneyclaim agai
nstMor
ningStarforpay
ment
oft
heunrefunded$5,301.88plusat
torney'
sfees.Thecomplai
ntwasraf
fl
edtoBr anch2ofthe
Metr
opol
itanTrial
Court(MeTC)ofMani laanddocketedasCivi
lCaseNo.169296-CV.

Mor ningStarcounter
edt hatV-Gentwasnotenti
tl
edt oaref
undbecauset heticket
swere
boughtont heai
rl
inecompany 's"buyone,t
akeone"promo.Itall
egedthatther
ewer eonly
fourteen(14)unusedt i
cketsandonlyseven(7)ofthesewererefundabl
e;consider
ingt
hatithad
alr
eadyr efundedsix(6)ti
ckets(whichismoreorless50%of14) ,t
hentherewasnot hi
ngelseto
refund.

MorningStaral
soquest
ionedV-
Gent'
sper
sonali
tytofi
let
hesuit
.Itasser
tedthatthe
passenger
s,inwhosenamestheti
cket
swereissued,
arether
ealparti
es-
in-
int
erest.

OnJanuar y27, 2006,afterdueproceedings,theMeTCdismissedthecomplai


ntforlackofa
causeofact i
on.Citi
ngRul e3, Sect
ion3oft heRulesofCourt,
3theMeTCdeclaredthat,as
agentofthepassenger swhopai dfortheticket
s,V-
Gentstoodastherealpar
ty-
in-i
nter
est
.
Nev er
thel
ess,itsti
lldismissedthecompl ai
ntbecauseV-Gentfai
ledtoprovei
tsclai
m bya
preponderanceofev idence.

V-Gentappeal
edt
otheRegi
onal
Tri
alCour
t(RTC)andt
hecasewasdocket
edasCi
vi
lCaseNo.
06-115050.

OnSeptember25,2006,t
heRTCgr ant
edtheappeal af
terfi
ndingthatV-
Genthadestabl
i
shedits
cl
aim byapr
eponderanceofev
idence.I
tsetasi
det heMeTC' sjudgmentandorder
edMor ni
ng
St
artopayV-Genttheval
ueofthenine(9)unr
efundedticket
spl usat
tor
ney'
sfees.

MorningSt
arfi
ledapeti
ti
onf orr
evi
ewwit
htheCA;thecasewasdocket
edasCA-
G.R.SPNo.
97032.Morni
ngStarquest
ionedtheRTC'
sappr
eci
ationoft
heevi
denceandf
act
ualconcl
usi
ons.
I
talsoreit
eratedit
squestionaboutV-
Gent
'sl
egal
standi
ng,
submi
tt
ingonceagai
nthatV-
Genti
s
nottherealpart
y-i
n-i
nter
est.

OnNov ember11,
2008, t
heCAgr ant
edthepeti
tionforrevi
ewanddismissedV-Gent
'scompl
aint
.
TheCAheldthatV-Gentisnotareal
part
y-i
n-i
nterestbecauseitmer
elyact
edasanagentofthe
passenger
swhoboughtt heti
cketsf
rom Morni
ngSt arwiththei
rownmoney .

V-
Gentmovedf
orreconsi
der
ati
on,whichmoti
ontheCAdeni
edonFebr
uar
y5,
2009,
thus
cl
ear
ingt
hewayforthepr
esentpet
it
ionforr
evi
ewoncert
ior
ari
.

THEPETI
TION

V-Gentarguest hattheCAerredinrul
ingt
hatiti
snott her eal
party
-i
n-i
nterest.I
tasser
ts:
(1)that
theissueofitslegalstandi
ngtofil
ethecomplainthasalreadybecomef i
nal becauseMorning
Stardidnotappeal theMeTC'sruli
ngontheissue; (
2)thatiti
sarealparty
-in-i
nter
esti
nfi
lingthe
compl ai
nt;and(3)thatMorningStari
salr
eadyest oppedfrom questi
oni
ngV- Gent'sl
egal
standingtofil
et hecomplai
nt.

I
nitsComment ,Mor ni
ngSt arcounters:(1)thatithadnoobl igati
ontoappealtheMeTC
j
udgmentdismissingthecompl aintinitsfavor;(2)thattheMeTCdi dnotspecifi
cal
l
ystatethat
V-
Gentistherealparty
-in-
inter
est;(3)thatther ealpart
ies-
in-
inter
estaret
hepassengersnamed
ontheti
cket
s;and( 4)thatitmadenoadmi ssionsthatwoul destopitf
rom denyi
ngtheref
und.

OURRULI
NG

V-Gentmai nt
ainsthatt
heMeTCdet erminedthati
twasthereal
part
y-i
n-i
nter
est
.Itarguesthat
sinceMor ni
ngStardidnotappealt
hisspecif
icfi
ndi
ngwitht
heRTC, t
hentheMeTC' srul
ingon
thispointhadalr
eadybecomef i
nalandconclusi
ve;t
her
efor
e,Morni
ngStarcannol ongerrevi
ve
theissuebeforetheCA.

Wedi
sagr
eewi
thV-
Gent
.

TheMeTCdi smissedV-Gent'scompl aintagai


nstMorningStarrerforfailuretoproveitscl
aim.
Thi
sdismissalmeantthatthepl ai
ntif
fdidnotproveav i
olati
onofi tsrightforwhichthe
def
endantshouldbeheldl i
able.Thisruli
ngwaspl ai
nlyajudgmenti nMor ningStar
'sfavorand
onethati
thadnocauset oquest i
on.Indeed,i
twouldbelegallyil
logicalforMor ni
ngSt art
ofil
e
anappealtoquesti
onar uli
ngofdi smissalini
tsfavor.

V-Gentalsoar
guesthati
tisareal
party
-i
n-i
nter
estwit
hlegalst
andi
ngtoi
nst
itutethecompl
aint
againstMorni
ngStar
.Inthepr
esentpeti
ti
on,i
tstat
es:
chanRobl
esvi
rt
ual
Lawl
ibrary

36.TheCour tofAppeal schoset oignor


ethefactthatwhi letheplaneti
cketsbor ethenamesof
theindivi
dualpassengers,therespondentadmitt
edt hatitwast hepeti
ti
onert hattr
ansacted
businesswithitconcerningthepurchaseoftheseplanet ickets.Botht
hepur chaseorderand
receiptofpaymentswer eundert henameoft hepeti
t i
oner.Thus, si
nceitwast hepetit
ionerwho
purchasedtheseplaneti
cketsonbehalfofthepassengers,
therespondentvol
untari
lyr
efunded
totheformerthev al
ueofsix(6)unusedret
urnti
cketsinthetot
alamountofUS$3, 445.
62.
Though,forreasonsitdi
dnotr ev
ealtopeti
ti
oner
,itref
usedtorefundtherest.
4(Emphasis
suppli
ed.)

V-Gentadmitst hati
tpurchasedtheplaneticketsonbehalfofthepassenger
sast helatter
's
agent.
5Thet icketswereissuedinthenameoft hepassengersandpaidforwit
ht he
passengers'money .Nodisputeorconclusi
oni nthelowercourts'
mindsonthispoint
; hence,
boththeMeTC6andt heCA7commonl yfoundt hatV-Gentact
edasanagentoft hepassengers
wheni tpur
chasedt hepassengers'pl
anet i
ckets.

However,
whiletheMeTChel dthatV-Gentcoul
dsueasanagentactingi
nhi
sownnameon
behal
fofanundiscl
osedprinci
pal,t
heCAhel dthati
tcoul
dnotbecauset
herequi
rement
sfor
suchasuitbytheagenthadnotbeensat i
sfi
ed.

Weagr
eewi
tht
heCour
tofAppeal
s.

Ever
yacti
onmustbepr osecutedordefendedinthenameoft herealpar
ty-
in-
int
erest-thepar
ty
whostandst
obebenef i
tedori nj
uredbythejudgmentinthesui
t.
8I nsui
tswhereanagent
repr
esent
saparty,
theprinci
pal i
stherealpart
y-i
n-i
nter
est
;anagentcannotfi
leasuitinhisown
nameonbehalfofthepri
ncipal.

Rule3,
Secti
on3oft heRul
esofCour
tpr
ovi
dest
heexcept
ionwhenanagentmaysueorbe
suedwit
houtjoi
ningthepr
inci
pal
.

Section3.Repr esentativesaspar t
ies.-Wheretheact i
oni sall
owedt obepr osecut
edand
defendedbyar epresent at
iveorsomeoneact inginaf iduciar
ycapaci t
y,thebenef i
ciar
yshallbe
i
ncludedi nt het i
tl
eoft hecaseandshal lbedeemedt obet herealparty-
in-i
nterest
.A
representati
v emaybeat r
usteeofanexpr esstrust,aguardian,anexecut ororadmi ni
strat
or,or
apar tyauthorizedbyl awort heseRules.Anagentact inginhisownnameandf orthebenefitof
anundi sclosedpr incipalmaysueorbesuedwi thoutjoini
ngt hepri
ncipal exceptwhent he
contractinvolvest hingsbelongingt othepri
ncipal.(
Emphasi ssupplied.)

Thusanagentmaysueorbesuedsol elyini
tsownnameandwi t
houtj
oini
ngthepr i
nci
palwhen
thefol
lowingelementsconcur:(
1)t
heagentactedi
nhisownnameduringthetransact
ion;
(2)
theagentactedforthebenefi
tofanundiscl
osedpr
inci
pal
;and(
3)thet
ransact
iondidnot
i
nv ol
vethepropert
yoftheprinci
pal
.

Whent heseel
ementsarepresent,t
heagentbecomesboundasift
het
ransact
ionwer
eit
sown.
Thi
sruleisconsi
stentwit
hArti
cle1883oftheCivi
lCodewhich
say
s:chanRobl
esvi
rtual
Lawli
brary

Art
.1883.I
fanagentactsi
nhisownname, thepr
inci
palhasnorightofact
ionagainstt
he
per
sonswithwhom theagenthascont
ract
ed;nei
therhavesuchpersonsagai
nstthepri
nci
pal
.
Insuchcase,t
heagentistheonedi
rect
lyboundinfavoroft
hepersonwithwhom hehas
contr
acted,
asifthetr
ansacti
onwer
ehisown, exceptwhenthecont
racti
nvol
vest
hings
bel
ongingtothepri
nci
pal.

Theprovi
sionsoft
hisart
icl
eshallbeunderst
oodtobewi
thoutpr
ejudi
cet
otheact
ionsbet
ween
thepr
inci
palandagent.
chanrobl
esvi
rt
uall
awli
brar
y

I
nt hepresentcase,onl
ythefi
rstel
ementi spr esent
;thepurchaseorderandther ecei
ptwerein
thenameofV- Gent.However,t
heremainingel ementsar eabsentbecause:(
1)V-Gentdi scl
osed
thenamesoft hepassengerstoMorningSt ar—i nfacttheti
cketswereintheirnames;and( 2)
thetr
ansactionwaspaidusingthepassenger s'money .Ther
efore,
Rule3,Section3oft heRules
ofCourtcannotapply.

Todef i
netheactualfact
ualsi
tuat
ion,
V-Gent,theagent,issuingt orecovert
hemoneyofits
pri
ncipal
s—t hepassengers—whoar et her
eal part
ies-
in-i
nterestbecausetheyst
andtobe
i
njuredorbenefi
tedincaseMor ni
ngStarref
usesoragr eest ogr antther
efundbecauset
he
moneybel ongst
ot hem.From thi
sperspect
ive,V-Gentevidentlydoesnothav eal
egalst
anding
tofi
lethecomplaint.

Finall
y,V-
Gentar
guesthatbymaki ngapart
ialr
efund,Morni
ngSt
arwasal
readyestoppedfr
om
refusi
ngtomakeaf ul
lrefundonthegroundthatV-Genti
snott
hereal
par
ty-
in-
int
erestto
demandr eimbur
sement.9chanr
oblesl
aw

Wef
indnomer
iti
nthi
sar
gument
.

Thepowertocoll
ectandrecei
vepaymentsonbehal
fofthepr
inci
pal
isanor di
nar
yactof
admini
str
ati
oncoveredbythegener
alpowersofanagent.
10Ontheotherhand,t
hef
il
ingof
sui
tsisanactofstr
ictdomini
on.

UnderArti
cle1878( 15)oftheCi vi
lCode,adul yappoi nt
edagenthasnopowert oexerci
seany
actofstri
ctdominiononbehal fofthepr i
ncipal unl
essauthori
zedbyaspeci alpowerofattorney
.
Anagent '
sauthori
tytofil
esuitcannotbei nferredfrom hisauthori
tytocollectorr
eceive
payments;thegrantofspecialpower scannotbepr esumedf r
om thegr antofgeneralpowers.
Mor eov
er,theauthori
tytoexercisespecialpower smustbedul yestabl
i
shedbyev i
dence,even
thoughitneednotbei nwriti
ng.11chanroblesl
aw

Bygrantingthei ni
ti
alref
und, MorningStarrecogni
zedV-Gent'
sauthori
tytobuytheti
cketsand
col
lectrefundsonbehal fofthepassenger s.However,Morni
ngStar'
srecognit
ionofV-Gent'
s
author
it
yt ocollectarefundforthepassenger sisnotequi
val
enttorecognit
ionofV-Gent'
s
author
it
yt oini
tiateasuitonbehal fofthepassengers.Morni
ngStartheref
ore,i
snotestopped
fr
om quest i
oningV- Gent'
slegalstandi
ngt oini
ti
atethesui
t.

WHEREFORE,
premi
sesconsi
der
ed,
weDENYt
hepet
it
ionf
orl
ackofmer
it
.

SOORDERED.
cral
awl
awl
i
brar
y
Car
pio,
(Chai
rper
son)
,Del
Cast
il
lo,
Mendoz
a,andLeonen,
JJ.
,concur
.

Endnot
es:

RESIDENTMARI NEMAMMALSOFTHEPROTECTEDSEASCAPETANONSTRAI T,E.G.,


TOOTHEDWHALES, DOLPHI NS,PORPOISES, ANDOTHERCETACEANSPECI ES, JOINEDINAND
REPRESENTEDHEREI NBYHUMANBEI NGSGLORI AESTENZORAMOSANDROSE- LIZAEISMA-
OSORI O,
INTHEI RCAPACI TYASLEGALGUARDI ANSOFTHELESSERLI FE-
FORMSANDAS
RESPONSI BLESTEWARDSOFGOD' SCREATI ONS,Peti
ti
oners,v
.SECRETARYANGELOREYES,
INHISCAPACI TYASSECRETARYOFTHEDEPARTMENTOFENERGY( DOE),SECRETARYJOSE
L.ATIENZA, I
NHI SCAPACI TYASSECRETARYOFTHEDEPARTMENTOFENVI RONMENTAND
NATURALRESOURCES( DENR) ,LEONARDOR.SI BBALUCA, DENRREGIONALDI RECTOR-
REGIONVI IANDI NHISCAPACI TYASCHAI RPERSONOFTHETANONSTRAI TPROTECTED
SEASCAPEMANAGEMENTBOARD, BUREAUOFFI SHERI ESANDAQUATI CRESOURCES( BFAR)
,
DIRECTORMALCOLM I .SARMI ENTO,JR.,
BFARREGI ONALDI RECTORFORREGI ONVI IANDRES
M.BOJOS, JAPANPETROLEUM EXPLORATI ONCO. ,LTD.(
JAPEX),ASREPRESENTEDBYI TS
PHILIPPI
NEAGENT, SUPPLYOI LFIELDSERVI CES,
INC. ,
Respondent
s.

G.
R.No.181527

CENTRALVI SAYASFI SHERFOLKDEVELOPMENTCENTER( FI


DEC) ,
CERI LOD.ENGARCIAL,
RAMONYANONG, FRANCISCOLABI D,INTHEI RPERSONALCAPACI TYANDAS
REPRESENTATIVESOFTHESUBSI STENCEFI SHERFOLKSOFTHEMUNI CI
PALITI
ESOF
ALOGUINSANANDPI NAMUNGAJAN, CEBU,ANDTHEI RFAMILIES,ANDTHEPRESENTAND
FUTUREGENERATI ONSOFFI LIPINOSWHOSERI GHTSARESI MI LARLYAFFECTED, Peti
ti
oners,
v.SECRETARYANGELOREYES, INHI SCAPACI TYASSECRETARYOFTHEDEPARTMENTOF
ENERGY( DOE),JOSEL.ATI ENZA, I
NHI SCAPACI TYASSECRETARYOFTHEDEPARTMENTOF
ENVIRONMENTANDNATURALRESOURCES( DENR),LEONARDOR.SI BBALUCA,INHIS
CAPACITYASDENRREGI ONALDI RECTOR- REGIONVIIANDASCHAI RPERSONOFTHETAÑON
STRAITPROTECTEDSEASCAPEMANAGEMENTBOARD, ALANARRANGUEZ, INHI SCAPACITY
ASDIRECTORENVI RONMENTALMANAGEMENTBUREAU- REGIONVI I,DOEREGIONAL
DIRECTORFORREGI ONVI I
I1ANTONI OLABI OS,JAPANPETROLEUM EXPLORATI ONCO.,LTD.
(JAPEX),
ASREPRESENTEDBYI TSPHI LI
PPINEAGENT, SUPPLYOI LFIELDSERVICES,I
NC.,
Respondent
.

DECISION

LEONARDO-
DECASTRO,
J.:

BeforeUsaretwoconsol
idatedPet
it
ionsf i
l
edunderRule65ofthe1997RulesofCourt,
concerni
ngServi
ceContr
actNo.46(SC- 46),
whichal
lowedtheexpl
orat
ion,
devel
opment ,and
exploi
tat
ionofpet
rol
eum resour
ceswithinTañonStr
ait
,anarr
owpassageofwat ersi
tuat
ed
betweentheisl
andsofNegrosandCebu. 2
ThePetit
iondocket
edasG. R.No.180771isanori
ginal
Peti
ti
onforCerti
orari
,Mandamus,and
I
njuncti
on,whi
chseekst oenj
oinrespondent
sfr
om implement
ingSC-46andt ohavei
tnull
i
fied
forwil
l
fulandgrossvi
olati
onofthe1987Consti
tut
ionandcert
ainint
ernati
onalandmunici
pal
l
aws.3

Likewise,thePeti
ti
ondocket edasG. R.No.181527i sanoriginalPeti
tionforCer t
iorar
i,
Prohibiti
on,andMandamus, whi chseekst onull
if
ytheEnv i
ronment alCompl i
anceCer tif
icate
(ECC)i ssuedbytheEnv i
ronment alManagementBur eau(EMB)oft heDepar t
mentof
Env i
ronmentandNat uralResources( DENR),RegionVIIi
nconnect ionwi t
hSC- 46;toprohibit
respondent sfr
om i
mplement i
ngSC- 46;andtocompel publi
cr espondentstopr ovidepetit
ioner
s
accesst otheperti
nentdocument si
nv ol
vingt
heTañonSt raitOilExplorat
ionPr oj
ect.4

ANTECEDENTFACTSANDPROCEEDI
NGS

Peti
ti
oner sinG.R.No.180771, col l
ectivel
yrefer
redt oast he" ResidentMar ineMammal s"int
he
peti
ti
on, arethetoothedwhal es, dolphins,por
poises, andot hercet aceanspeci es,whichi
nhabit
thewat ersinandar oundtheTañonSt rait
.Theyar ejoinedbyGl ori
aEst enzoRamos( Ramos)
andRose- LizaEisma-Osorio(Eisma- Osori
o)ast heirl
egal guar di
ansandasf ri
ends(tobe
coll
ecti
v el
yknownas" theStewar ds")whoal l
egedlyempat hizewi t
h, andseekt heprotect
ionof,
theaforement i
onedmar i
nespeci es.Alsoimpleadedasanunwi ll
i
ngco- petiti
onerisformer
Presi
dentGl ori
aMacapagal -Arroy o, f
orherexpressdecl arationandunder takingintheASEAN
Chartertopr ot
ecttheTañonSt rait,amongot hers.5

Peti
tionersinG. R.No.181527aretheCent ralVisayasFi sherfol
kDev el
opmentCenter(FI
DEC),a
non-stock,non-profi
t,non-
gover
nment alorganizati
on, establi
shedforthewelfar
eofthe
marginal f
isherfol
kinRegionVII
;andCer il
oD.Engar cial (Engarci
al)
,RamonYanong( Yanong)
andFr anciscoLabid(Labid)
,int
heirpersonalcapaci tiesandasr epresent
ati
vesofthe
subsistencefisherf
olkofthemunicipali
ti
esofAl ogui nsanandPi namungajan,Cebu.

Namedasr espondentsinbothpeti
ti
onsar et hel
ateAngeloT.Rey es,asthenSecret
aryoft he
DepartmentofEner gy(DOE);
JoseL.At ienza,asthenSecretar
yoft heDENR; Leonar
doR.
Sibbaluca,asthenDENR- Regional
DirectorforRegionVIIandChairmanoft heTañonSt rait
ProtectedSeascapeManagementBoar d;JapanPet r
oleum Expl
orationCo.,Ltd.(
JAPEX),a
companyor ganizedandexist
ingundert helawsofJapanwi thaPhi li
ppi
nebr anchoff
ice;and
SupplyOi l
fi
eldSer v
ices,I
nc.(
SOS),astheal legedPhil
ippi
neagentofJAPEX.

I
nG.R.No.181527,thefol
lowi
ngwereimpl eadedasaddi
ti
onal publ
icrespondent
s: AlanC.
Ar
ranguez(Ar
ranguez)andAntoni
oLabios( Labi
os)
,int
heircapacit
iesasthenDirectoroft
he
EMB,RegionVI
Iandt henRegi
onalDi
rectoroftheDOE,RegionVII,r
especti
vely
.6

OnJune13, 2002, t
heGov ernmentofthePhil
ippines,act
ingthroughtheDOE, enter
edintoa
Geophysi
calSur veyandExplorati
onContract
-102( GSEC-102)withJAPEX.Thi scontract
i
nvolv
edgeol ogicalandgeophy si
calst
udi
esoft heTañonSt rai
t.Thestudiesincludedsurface
geol
ogy,sampl eanalysi
s,andreprocessi
ngofsei smicandmagnet icdata.JAPEX, assist
edby
DOE,al
soconduct
edgeophy
sical
andsat
ell
i
tesur
vey
s,aswel
lasoi
landgassampl
i
ngi
nTañon
St
rai
t.
7

OnDecember21, 2004,
DOEandJAPEXf ormal
lyconv er
tedGSEC-102i
ntoSC-46fort
he
expl
orat
ion,development,
andproducti
onofpetroleum r
esour
cesinablockcover
ing
appr
oximately2,850squareki
l
omet er
soffshor
et heTañonStrai
t.
8

From May9t o18, 2005,JAPEXconductedseismicsur


veysi
nandaroundtheTañonSt
rai
t.A
multi
-channelsub-bott
om prof
il
ingcov
eringapproxi
matel
y751ki
lometer
swasalsodoneto
deter
mi nethearea'sunderwat
ercompositi
on.
9

JAPEXcommi tt
edtodril
loneexplor
at i
onwelldur
ingt hesecondsub-phaseoftheproject.Si
nce
thewell
wast obedri
ll
edinthemar i
newat er
sofAl oguinsanandPinamungajan,wherethe
TañonStrai
twasdeclar
edapr ot
ectedseascapein1988, 10JAPEXagr eedtocompl ywiththe
Envi
ronmentalI
mpactAssessmentr equir
ementspur suantt
oPr esi
denti
alDecr
eeNo.1586,
enti
tl
ed"Est
abli
shi
ngAnEnv ir
onment alI
mpactStatementSy stem,Incl
udingOther
Envi
ronmentalManagementRelat
edMeasur esAndForOt herPurposes.
"11

OnJanuary31, 2007,
theProtect
edAreaManagementBoard12oftheTañonStrai
t(PAMB-
TañonStrai
t)i
ssuedResolutionNo.2007-
001,
13wher
einitadopt
edtheIni
ti
alEnvi
ronmental
Examinat
ion(IEE)commissionedbyJAPEX,andf
avor
ablyrecommendedtheapprovalof
JAPEX'sappli
cationf
oranECC.

OnMarch6,2007,t
heEMBofDENRRegi onVI IgrantedanECCt otheDOEandJAPEXf orthe
off
shor
eoilandgasexplorati
onproj
ectinTañonSt r
ait
.14Monthslater
,onNov ember16,2007,
JAPEXbegantodri
llanexplorat
orywell
,withadept hof3,150meters,nearPinamungaj
ant own
i
nthewesternCebuProvince.
15Thisdri
lli
nglastedunt i
lFebr
uary8,2008.16

I
twasi nviewoftheforegoingstat
eofaffair
sthatpeti
ti
onersappl
iedtot
hisCourtforr
edress,
viatwosepar at
eorigi
nalpet i
ti
onsbothdatedDecember17, 2007,
wherei
ntheycommonl yseek
thatrespondentsbeenjoinedf r
om i
mplement i
ngSC-46for,
amongot her
s,vi
olat
ionofthe1987
Const i
tut
ion.

OnMar ch31,2008,SOSfi
ledaMot i
ontoStri
ke17itsnameasar espondentonthegr oundthat
i
tisnotthePhili
ppi
neagentofJAPEX.Insupportofit
smotion,i
tsubmitt
edthebr anchoffi
ce
appli
cati
onofJAPEX,18wherei
nthelat
ter'
sresi
dentagentwasclearl
yidenti
fi
ed.SOScl ai
med
thati
thadactedasamer elogi
sti
cscontr
actorforJAPEXinit
soilandgasexplorati
onactivi
ti
es
i
nt hePhil
ippi
nes.

Petit
ionersResi
dentMar ineMammal sandSt ewardsopposedSOS'smot i
onont hegroundthat
i
twaspr ematur
e,itwaspr o-f
orma,anditwaspatentl
ydil
ator
y.Theyclai
medt hatSOSadmi tt
ed
that"i
tisinlawa(sic)pri
v ytoJAPEX"sincei
tdidthedri
ll
ingandotherexplorati
onactivi
ti
esin
TañonSt rai
tundertheinstruct
ionsofi
tspri
ncipal
,JAPEX.Theyarguedthatitwoul dbe
premat uret
odropSOSasapar tyasJAPEXhadnoty etbeenjoi
nedinthecase; andt
hatitwas
"conv
enient"forSOStoaskt heCour tt
osimplydropitsnamef rom t
hepar
ti
eswhenwhati t
shoul
dhav edonewast oeithernotifyoraskJAPEXt ojoini
tinitsmoti
ont
oenableproper
substi
tut
ion.Atthisj
unct
ure, pet
it
ionersResi
dentMar i
neMammal sandSt
ewardsalsoasked
theCourttoimpleadJAPEXPhi l
i
ppinesasaco­r espondentorasasubst
it
utefori
tsparent
company ,JAPEX.19

OnApr
il8,
2008,
theCour
tresol
vedt
oconsol
i
dat
eG.
R.No.180771andG.
R.No.181527.

OnMay26,2008,t
heFI
DECmanifest
ed20thatt
heywereadopt
ingi
ntot
otheOpposi
ti
onto
St
ri
kewit
hMot i
ontoI
mpleadf
il
edbypeti
ti
onersResi
dentMari
neMammalsandStewardsi
n
G.
R.No.180771.

OnJune19,2008,
publicr
espondent
sfi
ledt
heirMani
festat
ion21thatt
heywer
enotobj
ect
ingt
o
SOS'
sMotiontoStr
ikeasitwasnotJAPEX'
sresi
dentagent.JAPEXduri
ngal
lthi
sti
me,di
dnot
fi
l
eanycommentatal l
.

Thus,onFebr uary7, 2012,t


hisCourt,i
nanef forttoensur ethatal
lthepart
iesweregiv
enampl e
chanceandoppor tunit
ytoanswert heissuesher ei
n,issuedaResol ut
iondi
rect
ingtheCourt'
s
processservici
nguni ttoagainservethepar t
ieswithacopyoft heSeptember23,2008
Resolut
ionoft heCour t,whi
chgav eduecour setot hepetiti
onsinG.R.Nos.180771and181527,
andwhi chrequiredthepar t
iestosubmi tthei
rrespectivememor anda.TheFebruary7,2012
Resolut
ion22r eadsasf oll
ows:chanroblesvi
rt
uallawli
brary

G.R.No.180771( ResidentMar ineMammal soft heProtect


edSeascapeTañonSt rait
,e.g.
,
ToothedWhal es, Dolphins,PorpoisesandOt herCetaceanSpecies,etal.vs.Hon.AngeloRey es,
i
nhi scapacityasSecr etaryoftheDepar t
mentofEner gy,etal
.
)andG. R.No.181527( Centr
al
VisayasFisherfolkDev elopmentCent er,etal
.vs.Hon.AngeloRey es,etal.
).-TheCour t
Resolvedtodi r
ectt hePr ocessSer vi
cingUnitt
oRE- SENDt heresol
uti
ondat edSept ember23,
2008t othefoll
owi ngpar ti
esandcounsel ,t
ogetherwiththi
s
resol
ution:
chanr oblesvir
tuall
awlibrar
y

At
ty.Ar
ist
eoO.Car
iño20t
hFl
oorPear
lbankCent
re

Counsel
forRespondentSuppl
y 146Val
eroSt
reet

Oi
l
fiel
dSer
vices,
Inc.Sal
cedoVi
l
lage,
Makat
iCi
ty

JAPEXPhi
l
ippi
nesLt
d. 20t
hFl
oorPear
lbankCent
re

146Val
eroSt
reet

Sal
cedoVi
l
lage,
Makat
iCi
ty
JAPEXPhi
l
ippi
nesLt
d. 19t
hFl
oorPear
lbankCent
re

c/
oAt
ty.Mar
iaFar
ahZ.
G. 146Val
eroSt
reet

Ni
col
as-
Suchi
anco Sal
cedoVi
l
lage,
Makat
iCi
ty

At
ty.Mar
iaFar
ahZ.
G.Sui
te2404Di
scov
eryCent
re

Ni
col
as-
Suchi
anco 25ADBAv
enue

Resi
dentAgentofJAPEX Or
ti
gasCent
er,
Pasi
gCi
ty

Phi
l
ippi
nesLt
d.

ThisResolutionwasper sonall
yservedt otheabov eparti
es,attheaboveaddressesonFebr uary
23,2012.OnMar ch20, 2012,JAPEXPhi li
ppines,Ltd.(
JAPEXPH) ,bywayofspecialappearance,
fi
ledaMot iont oAdmi t
23i t
sMot i
onf orClari
ficat
ion,
24wher ei
nJAPEXPHr equestedtobe
cl
arif
iedast owhet herornotitshoulddeem t heFebruary7,2012Resoluti
onast hisCourt
's
Orderofitsinclusi
onint hecase,asithasnotbeeni mpleaded.Ital
soall
egedthatJAPEXPH
hadalreadyst oppedexplorati
onacti
v i
ti
esi ntheTañonSt rai
twaybackin2008, render
ingthis
casemoot .

OnMar ch22, 2012,JAPEXPH, al


sobyspecialappearance,fil
edaMot i
onforExtensi
onof
Time25tof i
leit
sMemor andum.Itstat
edthatsinceitreceivedtheFebruar
y7,2012Resolut
ion
onFebruary23, 2012,i
thadunt i
lMarch22,2012t ofi
leitsMemor andum.JAPEXPHt henasked
foranadditi
onal t
hir
tydays,supposedl
ytogivethisCour tsomet i
met oconsi
deritsMoti
onfor
Clar
if
icat
ion.

OnApr i
l24,2012,thi
sCourtissuedaResol
uti
on26gr
ant
ingJAPEXPH'sMoti
ontoAdmi
tit
s
Moti
onf orClar
if
icati
on.ThisCourt
,addr
essi
ngJAPEXPH'sMoti
onforCl
ari
fi
cat
ion,
hel
d:chanrobl
esvir
tual
lawl
ibrar
y

Withregardtoi t
sMot i
onforCl arifi
cati
on( BySpeci al Appear ance)dat edMar ch19,2012, t
his
CourtconsidersJAPEXPhi lippines.Ltd.asar eal par ty
-in-
int
erestint hesecases.UnderSect ion
2,Rule3oft he1997Rul esofCour t,arealpart
y -
in-interestisthepar tywhost andstobe
benefit
edori nj
uredbythej udgmenti nt hesuit
, ort hepar tyenti
tledtot heav ail
softhesui t
.
ContrarytoJAPEXPhi l
ippines, Ltd.
'sall
egationt hati ti
sacompl et
elydistinctcorporat
ion,whi ch
shouldnotbeconf usedwi t
hJAPEXCompany ,Lt d.
, JAPEXPhi l
ippines,Ltd.isamer ebr anch
off
ice,establ
ishedbyJAPEXCompany ,Lt
d.fort hepur poseofcar ryi
ngoutt helatt
er'
sbusi ness
tr
ansactionshereinthePhi li
ppines.Thus, JAPEXPhi li
ppines,Ltd.,hasnosepar atepersonalit
y
fr
om itsmot herfor
eigncorpor ati
on, t
hepar t
yimpl eadedi nthiscase.

Mor
eover,Sect
ion128oftheCorporati
onCodeprov
idesfort
heresponsi
bil
i
tiesanddut
iesofa
r
esi
dentagentofaforei
gncorporat
ion:
chanr
obl
esvi
rtual
l
awli
brar
y
SECTI ON128.Resi dentagent ;servi
ceofpr ocess.—TheSecur itiesandExchangeCommi ssion
shallrequir
easacondi ti
onpr ecedenttot heissuanceofthel i
censet ot ransactbusi nessint he
Phili
ppinesbyanyf or eigncor porat
ionthatsuchcor porat
ionf il
ewi ththeSecur i
ti
esand
ExchangeCommi ssi onawr i
ttenpowerofat t
orneydesi
gnatingsomeper sonwhomustbea
residentofthePhi l
ippi nes,onwhom anysummonsandot herlegal pr
ocessesmaybeser vedin
allacti
onsorot herl egal proceedingsagai nstsuchcorporation, andconsent i
ngt hatservi
ce
uponsuchr esidentagentshal lbeadmittedandhel dasval i
dasi fser
v edupont hedul y
authorizedofficersoft hef oreigncorporationatitshomeof fice.Anysuchf orei
gncor poration
shallli
kewiseexecut eandf i
lewi t
htheSecur i
ti
esandExchangeCommi ssionanagr eementor
sti
pulation,execut edbyt hepr operauthor it
iesofsaidcor
por ation,inform andsubst anceas
foll
ows:

"The( nameoff oreigncorporat


ion)doesher ebystipulateandagr ee,inconsiderat
ionofitsbeing
grantedbyt heSecur i
ti
esandExchangeCommi ssi
onal i
censetot ransactbusinessinthe
Phil
ippi nes,thatifatanytimesaidcor porationshal l
ceaset otransactbusi nessinthe
Phil
ippi nes,orshall bewit
houtanyr esidentagenti nthePhi l
ippinesonwhom anysummonsor
otherlegal processesmaybeser v
ed, t
heni nanyact ionorpr oceedingar i
singoutofany
businessort ransactionwhichoccurredint hePhi l
i
ppi nes,servi
ceofanysummonsorot her
l
egal pr ocessmaybemadeupont heSecur i
tiesandExchangeCommi ssi
onandt hatsuch
servi
ceshal lhavet hesamef orceandef fectasi fmadeupont hedul y-aut
hor i
zedoffi
cersofthe
corpor ati
onati tshomeof f
ice.
"

Whenev ersuchservi
ceofsummonsorot herprocessshallbemadeupont heSecur
iti
esand
ExchangeCommi ssi
on,theCommi ssi
onshall,wi
thinten(10)daysthereaf
ter
,tr
ansmi tbymaila
copyofsuchsummonsorot herl
egalprocesstothecorporati
onatitshomeorpr i
ncipalof
fice.
Thesendingofsuchcopybyt heCommi ssionshallbeanecessarypartofandshallcomplete
suchservice.Al
lexpensesincur r
edbytheCommi ssionforsuchser
v i
ceshallbepai
di nadvance
bythepar t
yatwhosei nstancetheservi
cei smade.

I
ncaseofachangeofaddressoft
heresidentagent
,itshal
lbehisori
tsdutyt
oimmedi
atel
y
not
if
yinwr
it
ingtheSecur
it
iesandExchangeCommi ssionofthenewaddress.

Iti
sclearfr om thef oregoi
ngpr ovi
siont
hatt
hefuncti
onofar esi
dentagentistoreceive
summonsorl egal processesthatmaybeservedinall
actionsorotherlegalproceedings
againstthef oreigncor por
ati
on.Thesecaseshavebeenpr osecutedinthenameofJAPEX
Company ,Ltd.
, andJAPEXPhi li
ppi
nesLtd.
,asit
sbranchof f
iceandresidentagent,hadbeen
receiv
ingthev ar i
ousr esolut
ionsfr
om thi
sCourt,
asevidencedbyRegi str
yReturnCar dssigned
byitsrepresent ati
ves.

Andintheinter
estofj
usti
ce,
thisCour
tresol
vedtograntJAPEXPH'smoti
onforext
ensi
onof
ti
metof i
l
ei t
smemor andum,andwasgivenunt
ilApr
il21,2012,
aspray
edfor
,wit
hinwhi
chto
complywiththesubmissi
on.
27

Wi
thoutf
il
ingi
tsMemor
andum,
JAPEXPH,
onMay14,
2012,
fil
edamot
ion,
aski
ngt
hisCour
t
foranaddit
ionalt
hir
tydayst
of i
l
eitsMemorandum, tobecount
edf
rom May8,2012.I
tjust
if
ied
i
tsrequestbyclai
mingthatt
hisCourt
'sApr
il24,2012Resol
uti
onwasi
ssuedpastit
srequested
deadl
ineforfi
li
ng,whichwasonApril
21,2012.28

OnJune19,2012,t
hisCour tdeni
edJAPEXPH'ssecondr
equestf
oraddi
ti
onal
timet
ofi
l
eit
s
Memorandum anddispensedwi t
hsuchfi
l
ing.

Sincepeti
ti
onershadalreadyfi
ledtheirr
especti
vememoranda,
29andpubli
crespondentshad
earli
erf
il
edaMani fest
ation30thattheywereadopt
ingt
heirCommentdatedMarch31, 2008as
theirmemorandum, t
hisCourtsubmi t
tedthecasef
ordeci
sion.
chanRobl
esvi
rt
ualLawl
ibrar
y

Pet
it
ioner
s'Al
l
egat
ions

Pr otestingt headv erseecol ogical impactofJAPEX' soil explorat


ionacti
vi
tiesint heTañonSt rai
t,
pet i
tionersResi dentMar i
neMammal sandSt ewar dsav ert hatastudymadeaf terthesei smi c
sur veyshowedt hatt hef i
shcat chwasr educeddr asti
callyby50t o70per cent.Theycl aimt hat
bef orethesei smi csur vey,theav erageharvestperdaywoul dbefrom 15to20ki l
os; butaf ter
theact iv i
ty,thef isher fol
kcoul donl ycatchanav erageof1t o2kil
osaday .Theyat t
ri
but et his
"reducedf i
shcat ch"t othedest ructionofthe" payao"al soknownast he"fishaggr egating
dev ice"or" artif
icial reef."
31Pet iti
onersResi dentMar i
neMammal sandSt ewar dsalsoi mput e
thei ncidencesof" fishki l
l"
32obser vedbysomeoft helocal fi
sherf
olktothesei smicsur v ey.
Andt heyf urtheral leget hatt heECCobt ai
nedbypr i
vater espondentJAPEXi sinv al
idbecause
publ icconsul t
ationsanddi scussionswi ththeaf fectedst akeholders,
apre- r
equisitetot he
i
ssuanceoft heECC, werenothel dpri
ortotheECC' sissuance.

Init
ssepar atepeti
ti
on, petit
ionerFI DECconf ir
mspet it
ionersResi dentMar i
neMammal sand
Stewar ds'all
egati
onsofr educedf i
shcat chandlackofpubl icconsul tati
onsordi scussionswi th
thef i
sherfolkandotherst akehol derspriortotheissuanceoft heECC.Mor eover,i
tall
egest hat
duringt heseismicsur v
ey sanddr il
li
ng,itwasbarredf r
om ent eri
ngandf ishingwi t
hina7-
kil
omet erradiusfrom thepoi ntwher etheoilri
gwasl ocated,anar eagr eaterthant he1.5-
kil
omet erradius"exclusionzone"st atedintheIEE.33Italsoagr eesi ntheal l
egationthatpubl i
c
respondent sDENRandEMBabusedt heirdi
scret
ionwhent heyissuedanECCt opublic
respondentDOEandpr ivater espondentJAPEXwi t
houtensur ingt hestri
ctcompl i
ancewi ththe
procedur alandsubstant i
v erequirement sundertheEnv ironment al I
mpactAssessmentsy stem,
theFisher i
esCode, andt heirimpl ement i
ngr ul
esandr egulati
ons.34I tfurtherclaimsthat
despitesev eralr
equest sforcopi esofal lthedocument sper t
ainingt othepr ojectinTaflonStrai
t,
onlycopi esofthePAMB- TañonSt r
aitResoluti
onandt heECCwer egi vent othefisher
folk.35

Publ
i
cRespondent
s'Count
er-
All
egat
ions

Publi
crespondents,
throught heSolici
torGeneral
,contendt hatpet
iti
oner
sResidentMari
ne
Mammal sandSt ewardshav enolegal standi
ngtofil
ethepr esentpeti
ti
on;t
hatSC-46doesnot
vi
olat
ethe1987Const ituti
onandt hev ar
iouslawscitedinthepet i
ti
ons;t
hattheECCwas
i
ssuedinaccordancewi thexist
inglawsandr egul
ations;thatpubli
crespondent
smaynotbe
compel l
edbymandamust ofurni
shpetiti
onerscopi
esofalldocument srel
atingtoSC-46;and
thatallt
hepeti
ti
onersfai
ledtoshowt hattheyareenti
tl
edtoinjuncti
vereli
ef.Theyfur
ther
contendthatt
heissuesraisedinthesepetiti
onshavebeenrenderedmootandacademi cbyt he
factthatSC-
46hadbeenmut uall
yterminatedbythepart
iestheret
oeffecti
veJune21, 2008.36

I
SSUES

Thefoll
owingarethei
ssuesposit
edbypet
it
ioner
sResi
dentMar
ineMammal
sandSt
ewar
dsi
n
G.R.No.180771:
chanr
oblesv
irt
uall
awl
i
brar
y

WHETHERORNOTPETI
TIONERSHAVELOCUSSTANDITOFI
LETHEI
NSTANTPETI
TION;

WHETHERORNOTSERVI
CECONTRACTNO.46I
SVI
OLAT[
IVE]OFTHE1987PHI
LIPPI
NE
CONSTI
TUTI
ONANDSTATUTES;

WHETHERORNOTTHEON- GOINGEXPLORATI
ONANDPROPOSEDEXPLOI TATI
ONFOROI L
ANDNATURALGASAT,AROUND,ANDUNDERNEATHTHEMARI NEWATERSOFTHETANON
STRAI
TPROTECTEDSEASCAPEISI
NCONSISTENTWI THTHEPHILI
PPI
NECOMMI TMENTSTO
I
NTERNATI
ONALENVIRONMENTALLAWSANDI NSTRUMENTS;
AND

WHETHERORNOTTHEI SSUANCEOFTHEENVIRONMENTALCOMPLI
ANCECERTI
FICATE
(ECC)I
NENVIRONMENTALLYCRI
TICALAREASANDHABI
TATSOFMARINEWI
LDLI
FEAND
ENDANGEREDSPECI
ESISLEGALANDPROPER.37

Meanwhil
e,i
nG. R.No.181527,
peti
ti
onerFI
DECpr
esent
edt
hef
oll
owi
ngi
ssuesf
orour
consi
der
ati
on:chanrobl
esvi
rt
ual
lawl
ibr
ary

WHETHERORNOTSERVICECONTRACTNO.46EXECUTEDBETWEENRESPONDENTSDOE
ANDJAPEXSHOULDBENULLIFI
EDANDSETASIDEFORBEI
NGINDIRECTVIOLATI
ONOF
SPECI
FICPROVI
SIONSOFTHE1987PHI
LIPPI
NECONSTI
TUTI
ONANDAPPLICABLELAWS;

WHETHERORNOTTHEOFF- SHOREOI
LEXPLORATI
ONCONTEMPLATEDUNDERSERVI
CE
CONTRACTNO.46ISLEGALLYPERMI
SSI
BLEWITHOUTALAW BEI
NGDULYPASSED
EXPRESSLYFORTHEPURPOSE;

WHETHERORNOTTHEOI LEXPLORATI
ONBEINGCONDUCTEDWITHINTHETANONSTRAI
T
PROTECTEDSEASCAPEVI
OLATESTHERIGHTSANDLEGALPROTECTI
ONGRANTEDTO
PETI
TIONERSUNDERTHECONSTITUTI
ONANDAPPLICABLELAWS.

WHETHERORNOTTHEI SSUANCEOFTHEENVIRONMENTALCOMPLIANCECERTIFI
CATE
(ECC)FORSUCHANENVI RONMENTALLYCRI
TICALPROJECTINSI
DEANENVIRONMENTALLY
CRITI
CALAREASUCHASTHETANONSTRAI TPROTECTEDSEASCAPECONFORMEDTOLAW
ANDEXI STI
NGRULESANDREGULATIONSONTHEMATTER.

WHETHERORNOTTHERESPONDENTSMAYBECOMPELLEDBYMANDAMUSTOFURNISH
PETI
TIONERSWI
THCOPI
ESOFTHEDOCUMENTSPERTAI
NINGTOTHETANONSTRAI
TOIL
EXPLORATI
ONPROJECT.
38

I
ntheseconsol
i
dat
edpeti
ti
ons,
thisCour
thasdet
erminedt
hatt
hev
ari
ousi
ssuesr
aisedbyt
he
pet
it
ioner
smaybecondensedi
ntotwopri
maryi
ssues:

Pr
ocedur
alIssue:LocusSt
andi
oft
heResi
dentMar
ineMammal
sandSt
ewar
ds,
pet
it
ioner
sin
G.
R.No.180771;and

Mai
nIssue:
Legal
i
tyofSendeeCont
ractNo.46.

DI
SCUSSI
ON

Attheoutset,
thi
sCourtmakescl
earthatthe'"
mootandacademi cpr
inci
ple'
isnotamagical
for
mulathatcanautomati
cal
lydi
ssuadethecour t
sinr
esol
vingacase."Court
shavedeci
ded
casesotherwi
semootandacademicundert hefol
lowi
ngexcept
ions:

1)Ther
eisagr
avev
iol
ati
onoft
heConst
it
uti
on;

2)Theexcept
ional
char
act
eroft
hesi
tuat
ionandt
hepar
amountpubl
i
cint
eresti
sinv
olv
ed;

3)Theconst
it
utionali
ssuer
aisedr
equi
resf
ormul
ati
onofcont
rol
l
ingpr
inci
plest
ogui
det
he
bench,
thebar,
andt hepubl
i
c;and

4)Thecasei
scapabl
eofr
epet
it
iony
etev
adi
ngr
evi
ew.
39

Inthi
scase,despit
etheterminati
onofSC- 46,thisCourtdeemsitnecessarytoresolvet
hese
consoli
datedpeti
ti
onsasalmostal loftheforegoingexcepti
onsarepresentinthi
scase.Both
peti
ti
onersall
egethatSC-46isviolat
iveoftheConst i
tut
ion,t
heenv i
ronmentalandliv
eli
hood
i
ssuesr ai
sedundoubtedl
yaffectthepublic'
sinterest
,andtherespondents'
contestedacti
ons
arecapableofrepet
it
ion.
chanRoblesvirt
ualLawli
brary

Pr
ocedur
alI
ssues

LocusSt
andi
ofPet
it
ioner
sResi
dentMar
ineMammal
sandSt
ewar
ds

TheResi dentMar i
neMammal s,thr
ought heSt ewards,"cl
aim"thattheyhav ethel egalst
anding
tof il
ethisactionsincet heyst andtobebenef i
tedori nj
uredbyt hejudgmentint hissuit.
40
CitingOposav .Factor
an, Jr.
,41theyalsoasser ttheirri
ghttosuef orthefait
hfulper f
ormanceof
i
nt ernati
onal andmuni cipal envir
onment allawscr eatedinthei
rf av
orandf ortheirbenefit
.Inthi
s
regar d,t
heypr opoundt hatt heyhav etherighttodemandt hattheybeaccor dedt hebenefits
grant edtothem inmul tilaterali
nternati
onal i
nstr
ument sthatthePhi l
ippi
neGov er nmenthad
signed, undertheconceptofst i
pulati
onpouraut rui.42

Forthei
rpart
,theStewar
dscontendt hatther
eshoul dbenoquest i
onoft hei
rri
ghtt
orepr
esent
theResi
dentMar i
neMammal sast heyhav estakesinthecaseasf or
erunnersofacampai
gnto
buil
dawarenessamongt heaff
ectedr esi
dentsofTañonSt rai
tandasst ewardsoft
he
envi
ronmentsincethepr
imarysteward, t
heGov ernment,hadfai
ledinit
sdutytoprot
ectt
he
env
ironmentpur
suantt
othepubl
i
ctr
ustdoct
ri
ne.
43

Peti
ti
oner
sResidentMarineMammalsandSt ewardsalsoaverthatt
hisCour
tmayl
owert
he
benchmarki
nlocusstandiasanexer
ciseofepist
olar
yjuri
sdict
ion.
44

I
nopposi t
ion,
publicrespondentsar
guethatt
heResidentMari
neMammal shavenostandi
ng
becauseSecti
on1, Rule3oft heRul
esofCourtr
equir
espart
iestoanact
iontobeeit
hernatur
al
orjur
idi
calpersons,v
iz.:
chanrobl
esvi
rt
ual
lawl
ibr
ary

Secti
on1.Whomaybepar t
ies;pl
aint
iffanddef endant
.-Onlynat uralorjuri
dical
persons,or
enti
ti
esauthori
zedbylawmaybepar t
iesinaci vi
lacti
on.Thet erm" pl
ainti
ffmayr ef
ertothe
clai
mingparty,t
hecounter
-clai
mant ,
thecross-claimant,ort
het hir
d( f
our t
h,etc.
)-
part
yplainti
ff
.
Theterm"defendant"mayrefertotheor i
ginaldefendi
ngpar t
y,thedef endantinacounterclai
m,
thecross-
defendant
,orthethir
d(fourth,et
c.)-
partydefendant.

Thepubli
crespondentsalsocontestt
heappl
icabi
li
tyofOposa,poi
nti
ngoutt
hatt
hepet
it
ioner
s
ther
einwereal
lnaturalpersons,
albei
tsomeofthem werest
il
lunborn.
45

AsregardstheStewards,thepublicrespondentsli
kewisechall
engetheircl
aim oflegalst
anding
onthegroundthattheyarer epr
esentinganimals,whichcannotbeparti
est oanact i
on.
Moreover,t
hepubli
cr espondentsarguet hatt
heSt ewardsarenottherealpart
ies-i
n-i
nter
estfor
thei
rfai
l
uretoshowhowt heystandtobebenef it
edorinjur
edbyt hedecisi
onint hi
scase.46

I
nvoki
ngthealt
eregoprinci
pleinpol
it
icall
aw,t
hepubli
crespondent
sclai
mthatabsentany
pr
oofthatf
ormerPresi
dentArroyohaddisappr
ovedofthei
ractsi
nenteri
ngi
ntoand
i
mplementi
ngSC-46,suchactsremaintobeherown.47

Thepubli
crespondentscontendt
hatsi
ncepeti
ti
onersResi
dentMar i
neMammalsandStewards'
peti
ti
onwasnotbr oughti
nthenameofar eal
party
-i
n-i
nter
est
,itshouldbedi
smi
ssedf
orfai
lure
tostat
eacauseofact i
on.48

Theissueofwhetherornotanimalsorev eninanimateobjectsshoul
dbegivenlegalst
andi
ngin
acti
onsbefor
ecourtsoflawisnotnewi nthefiel
dofani malri
ghtsandenvi
ronmentall
aw.
Peti
ti
onersResi
dentMarineMammal sandStewardsci t
edthe1972UnitedStat
escaseSier
ra
Clubv.Roger
sC.B.Mor t
on,49wher einJusti
ceWi l
li
am O.Douglas,di
ssent
ingtothe
conventi
onal
thoughtonlegalstanding,opi
ned:chanrobl
esvir
tual
lawl
ibr
ary

Thecrit
ical
questionof"standing"wouldbesimpli
fiedandal soputneatlyi
nfocusifwe
fashi
onedaf eder
alrul
et hatall
owedenv i
ronmentalissuest obelit
igat
edbefor
efederal
agenci
esorf eder
alcourtsinthenameoft heinanimateobj ectabouttobedespoil
ed,def
aced,
orinv
adedbyr oadsandbul ldozersandwhereinj
uryist hesubjectofpubli
coutr
age,xxx.

Inani
mateobject
saresometi
mespar t
iesi
nliti
gat
ion.Ashiphasalegalpersonal
it
y,afi
cti
on
founduseful
formari
ti
mepurposes.Thecorporat
ionsole-acreat
ureofecclesi
asti
call
aw-is
anacceptabl
eadver
saryandl
argefort
unesrideonitscases.Theordi
narycorpor
ationi
sa
"per
son"forpur
posesoftheadjudi
cat
orypr
ocesses,
whet
heri
trepr
esent
spr
opr
iet
ary
,spi
ri
tual
,
aesthet
ic,
orchari
tabl
ecauses.

Soitshoul dbeasr espectsvall


eys,alpinemeadows, ri
v ers,lakes,estuari
es,beaches, ri
dges,
grovesoft rees, swampland,orevenai rthatfeelst hedest r
uct i
vepressur esofmoder n
technologyandmodem l i
fe.Theriv
er, f
orexampl e, i
sthel i
vingsymbol ofalltheli
feitsustai
ns
ornourishes—f i
sh,aquati
cinsects,waterouzel s,ot t
er,fisher ,
deer,el
k, bear,andallother
animals,includingman, whoar edependentoni torwhoenj oyitforit
ssi ght,i
tssound, ori
tsli
fe.
Ther i
veraspl ai
ntif
fspeaksfortheecol ogicalunitofl i
fet hati spartofit.Thosepeopl ewho
haveameani ngfulrel
ati
ontothatbodyofwat er—whet heritbeaf i
sherman, acanoei st,a
zoologist
, oral ogger—mustbeabl etospeakf ort hev alueswhi chtheriverrepresentsand
whichar et hr
eat enedwithdestr
uction.50(Citationsomi tted.)

Thepr imaryr easonanimal r


ightsadv ocat
esandenvir
onment ali
stsseektogi
veanimalsand
i
nani mateobj ectsstandi
ngisduet otheneedtocompl ywiththestr
ictr
equi
rementsinbr i
ngi
ng
asui ttocourt.Ourown1997Rul esofCourtdemandt hatpart
iestoasuitbeeit
hernaturalor
j
ur i
dicalpersons,orentit
iesauthorizedbylaw.Itf
urt
hernecessitat
estheacti
ontobebr oughtin
thenameoft herealpart
y-i
n-int
erest,eveniff
il
edbyar epr
esentati
ve,
viz.
:chanroblesvir
tuall
awli
brary

Rul
e3

Par
ti
est
oCi
vi
lAct
ions

Secti
on1.Whomaybepar t
ies;pl
aint
iffanddef endant
.-Onlynat uralorjuri
dical
persons,or
enti
ti
esauthori
zedbylawmaybepar t
iesinaci vi
lacti
on.Thet erm" pl
ainti
ffmayr ef
ertothe
clai
mingparty,t
hecounter
-clai
mant ,
thecross-claimant,ort
het hir
d( f
our t
h,etc.
)-
part
yplainti
ff
.
Theterm"defendant"mayrefertotheor i
ginaldefendi
ngpar t
y,thedef endantinacounterclai
m,
thecross-
defendant
,orthethir
d(fourth,et
c.)-
partydefendant.

Sec.2.Part
iesininterest.-Areal
par t
yininter
esti
st hepart
ywhost andst
obebenefit
edor
i
njuredbythejudgmenti nthesui
t,orthepartyent
it
ledtotheavail
softhesuit
.Unl
essother
wise
authori
zedbylawort heseRules,
ev er
yactionmustbepr osecut
edordefendedint
henameof
therealpar
tyininterest.

Sec.3.Repr esentat
ivesaspar t
ies.-Wher etheacti
onisallowedtobepr osecutedordef ended
byar epresentati
veorsomeoneact ingi naf i
duciar
ycapacity,t
hebenefici
aryshall bei ncludedi
n
theti
tleofthecaseandshal lbedeemedt obet herealpart
yinint
erest
.Ar epresent ativemaybe
atrusteeofanexpr esstrust,
aguar dian,anexecutororadmi ni
str
ator,
orapar tyaut hor i
zedby
l
awort heseRules.Anagentact i
ngi nhisownnameandf orthebenefi
tofanundi scl osed
pri
ncipalmaysueorbesuedwi thoutjoiningthepri
ncipalexceptwhenthecont racti nvolves
thi
ngsbel ongingtotheprinci
pal .

I
thadbeensuggest
edbyani
mal
right
sadv
ocat
esandenv
ironment
ali
stst
hatnotonl
ynat
ural
andj uri
dicalper sonsshoul dbegi v enlegalstandi
ngbecauseoft hediff
icult
yf orpersons, who
cannotshowt hatt heybyt hemsel v esarereal par
ties-in-
inter
ests, t
obr i
ngact i
onsin
representationoft heseani mal sori nani
mat eobject s.Forthisreason, manyenv i
ronment al
caseshav ebeendi smissedf orfailureofthepet i
ti
onert oshowt hathe/ shewoul dbedi rectl
y
i
njuredoraf fectedbyt heout comeoft hecase.Howev er,i
nourj urisdi
ction,l
ocusst andiin
environment al caseshasbeengi v enamor eliber
alizedappr oach.Whi ledev elopmentsin
Phil
ippinelegal t
heor yandj urisprudencehav enotpr ogressedasf arasJust iceDouglas's
paradigm ofl egal standi
ngf orinani mateobjects,thecur renttr
endmov estowar ds
simpl i
fi
cationofpr oceduresandf acil
it
ati
ngcour taccessi nenvironment alcases.

Recently
,theCourtpassedthel
andmar kRulesofProcedur
ef orEnv
ironment
alCases,
51whi
ch
al
lowf ora"ci
ti
zensuit,
"andpermitanyFil
i
pinocit
izentofi
leanacti
onbeforeourcour
tsf
or
vi
olat
ionsofourenv i
ronmental
laws:chanr
oblesv
irt
uall
awli
brary

SEC.5.Ci tizensui t.-AnyFili


pinociti
zeninrepresentati
onofot her
s,includingmi nor
sor
gener ati
onsy etunbor n,mayf i
leanact i
ontoenforcer i
ghtsorobli
gationsunderenv i
ronment al
l
aws.Upont hef i
lingofaci ti
zensui t
,thecourtshalli
ssueanor derwhi chshallcontai
nabr i
ef
descr i
pti
onoft hecauseofact ionandt herel
i
efspr ayedfor,r
equir
ingal lint
erestedpar t
iest o
mani festtheirinteresttointerveneinthecasewi t
hinfift
een(15)daysf r
om not i
cethereof .The
plainti
ffmaypubl ishtheor deroncei nanewspaperofagener alci
rculationinthePhilippinesor
furnishallaffectedbar angay scopiesofsaidorder.

Ci
ti
zensuit
sfi
l
edunderR.A.No.8749andR.
A.No.9003shal
lbegov
ernedbyt
hei
rrespect
ive
pr
ovi
sions.
52(Emphasi
sours.
)

Expl
aini
ngtherat
ional
efort
hisr
ule,theCourt,
intheAnnotat
ionst
otheRul
esofPr
ocedur
efor
Envi
ronmental
Cases,commented:chanr
oblesvi
rt
uall
awli
brar
y

Citi
zensui t.Tofurt
herencour aget heprotect
ionoft heenvi
ronment,theRulesenabl eli
ti
gant
s
enforcingenv i
ronment alri
ghtstof i
let
heircasesasci ti
zensuit
s.Thisprovisi
onl iberal
izes
standingf oral
lcasesf il
edenf or
cingenv i
ronment all
awsandcol l
apsesthet r
adi t
ional r
uleon
personal anddirectint
erest,onthepr i
nciplethathumansar estewardsofnat ure.The
terminologyoft hetextrefl
ectsthedoct ri
nefirstenunci
atedinOposav .Factoran, i
nsofarasit
refer
st omi norsandgener ationsyetunbor n.
53( Emphasissuppli
ed,cit
ati
onomi tted.)

Althought hispeti
ti
onwasf iledin2007,year
sbef oretheeff
ectiv
ityoftheRulesofProcedure
forEnv ironment alCases,i
thasbeenconsi stentl
yheldthatrul
esofpr ocedur
e"maybe
ret
r oact i
velyappli
edtoactionspendingandundet er
minedatthet i
meoft hei
rpassageandwi ll
notv iolateanyr i
ghtofaper sonwhomayf eelthatheisadversel
yaf f
ected,i
nasmuchast here
i
snov estedr i
ghtsinrul
esofpr ocedure.
"54

El
uci
dati
ngont
hisdoctr
ine,
theCourt,i
nSyst
emsFactor
sCorpor
ati
onv
.Nat
ional
Labor
Rel
ati
onsCommissi
on55heldthat
:chanr
obl
esvi
rt
ual
l
awli
brar
y
Remedial statut
esorst atutesrelati
ngtoremedi esormodesofpr ocedur e,whichdonotcr eate
newort akeawayv est edr i
ghts,butonlyoperateinfurther
anceoft her emedyorconf i
rmati
onof
ri
ghtsalreadyexisting, donotcomewi t
hinthelegalconceptionofar etroacti
vel aw,orthe
generalruleagainstr etroacti
veoper at
ionofstatutes.Stat
utesregulati
ngt hepr ocedureofthe
court
swi llbeconst ruedasappl i
cabletoactionspendingandundet erminedatt hetimeoft hei
r
passage.Pr ocedural lawsar eretroact
iveinthatsenseandt othatextent ,xxx.

Mor eover,evenbef or
et heRul esofPr ocedur eforEnv i
ronmental
Casesbecameef fecti
ve,this
Cour thadal readytakenaper missiveposit
ionont heissueofl
ocusst andi inenv i
ronment al
cases.I nOposa, weallowedt hesuittobebr oughtinthenameofgener ationsy etunborn
"basedont heconceptofi ntergenerati
onal responsibil
ityi
nsof
arast her i
ghtt oabal ancedand
healthfulecol ogyisconcer ned."56Furthermor e,wesai dthatt
herighttoabal ancedand
healthfulecol ogy,ari
ghtt hatdoesnotev enneedt obest at
edinourConst it
utionasi ti
s
assumedt oexi stfrom thei nceptionofhumanki nd,carri
eswit
hitthecor relat
ivedut ytorefrai
n
from impairingt heenvironment .57

Inlightoft hefor
egoing,theneedt ogi vetheResi dentMar ineMammal sl egalst
andi nghasbeen
elimi natedbyourRules,whichal l
owanyFi l
ipinoci ti
zen, asast ewar dofnat ur
e,tobr ingasuitt
o
enfor ceourenv i
ronmental l
aws.Itiswor thnot ingher et hattheSt ewardsar ejoi
nedasr eal
partiesi nthePeti
ti
onandnotj usti
nr epresent ati
onoft henamedcet aceanspeci es.The
Stewar ds,RamosandEi sma-Osor i
o, havi
ngshowni nt heirpetiti
onthatt heremaybepossi bl
e
violationsoflawsconcer ni
ngt hehabi t
atoft heResi dentMar i
neMammal s,ar
et herefore
decl aredtopossesst helegalstandingtof i
let hispetit
ion. chanRoblesv i
rtual
Lawlibrary

I
mpl
eadi
ngFor
merPr
esi
dentGl
ori
aMacapagal
-Ar
roy
oasanUnwi
l
li
ngCo-
Pet
it
ioner

Peti
tionersStewar
dsinG.R.No.180771i
mpleadedasanunwill
i
ngco-
pet
it
ionerf
ormer
Presi
dentGl ori
aMacapagal
-Arr
oyofort
hefol
lowingr
easons,
whichwe
quote:chanr
oblesv
irt
ual
lawl
ibr
ary

HerExcel lencyGlori
aMacapagal -Arroy o, al
sooflegal age, Fil
ipi
noandr esi dentofMalacañang
Palace, Mani l
aPhil
ippi
nes.Stewar dGl or i
aMacapagal -Arroy ohappenst obet heincumbent
Presidentoft hePhili
ppineI
slands.Shei spersonal
lyimpl eadedi nt hi
ssui tasanunwi l
li
ngco-
peti
tionerbyr easonofherexpr essdecl arati
onandunder takingundert her ecentl
ysigned
ASEANChar tert
opr ot
ectYourPet iti
oner s'habit
at,amongot hers.Sheismeant i
medomi nated
asanunwi l
l
ingco-peti
ti
onerduet ol ackofmat eri
altimei nseeki nghersi gnatureand
i
mpr imat urhereofandduet opossi blelegal complicationst hatmayher eafterari
sebyreasonof
herof f
icialrel
ati
onswi t
hpublicrespondent sundert heal t
eregopr inci
plei npoli
ti
cal
l
aw. 58cralawlawli
brary

Thi
sisi
ncor
rect
.

Sect
ion10,
Rul
e3oft
heRul
esofCour
tpr
ovi
des:
chanr
obl
esv
irt
ual
l
awl
i
brar
y
Sec.10.Unwill
ingco-
plai
nti
ff
.-I
ftheconsentofanypar
tywhoshoul
dbej oi
nedasplai
nti
ffcan
notbeobtained,hemaybemadeadef endantandthereasont
her
eforshall
bestat
edinthe
complaint
.

Undert heforegoi
ngrule,whent heconsentofapartywhoshouldbej oinedasapl ai
nti
ffcannot
beobt ained,
heorshemaybemadeapar t
ydefendanttothecase.Thiswillputtheunwi l
li
ng
partyunderthejuri
sdi
ct i
onoftheCour t
,whichcanpr oper
lyi
mpleadhi m orherthroughi t
s
processes.Theunwi l
l
ingpar t
y'snamecannotbesi mplyincl
udedinapet it
ion,withouthisorher
knowledgeandconsent ,assuchwouldbeadeni alofdueprocess.

Mor eover,
thereasonci t
edbyt hepet
it
ionersSt ewar dsf orincl
udingf
ormerPr esident
Macapagal -
Arroyoi ntheirpet i
ti
on,i
snotsufficienttoimpl eadherasanunwi l
lingco-peti
ti
oner .
I
mpl eadingthefor merPr esidentasanunwi l
lingco- petiti
oner ,
foranactshemadei nthe
performanceoft hef unctionsofheroffice,
iscont rarytot hepubli
cpol
icyagainstembr oil
ingthe
Presidenti
nsui t
s, "
toassur et heexer
ciseofPr esidential duti
esandfuncti
onsf reefrom any
hindranceordistracti
on, consideri
ngthatbeingt heChi efExecut i
veoftheGov ernmentisaj ob
that,asi
defrom requi r
ingal loftheof
fi
cehol der'sti
me, alsodemandsundi vi
dedat tent
ion."59

Theref
ore,
formerPresi
dentMacapagal
-Ar
royocannotbei
mpleadedasoneofthepeti
ti
oner
sin
thi
ssuit
.Thus,hernameisstr
ickenof
ftheti
tl
eofthi
scase.
chanRobl
esvi
rt
ual
Lawli
brary

Mai
nIssue:

Legal
i
tyofSer
viceCont
ractNo.46

Ser
viceCont
ractNo.46v
is-
a-v
is

Sect
ion2,
Art
icl
eXI
Ioft
he

1987Const
it
uti
on

Petit
ioner smai ntainthatSC- 46t r
ansgr essest heJur aRegaliaPr ov i
sionorpar agr aph1, Secti
on
2,ArticleXIIoft he1987Const i
tutionbecauseJAPEXi s100%Japanese- owned. 60Fur thermore,
theFIDECasser tsthatSC- 46cannotbeconsi deredasat echnical andf i
nanci alassi st
ance
agreementv alidlyexecut edunderpar agr aph4oft hesamepr ov i
sion.61Thepet i
t i
onerscl ai
m
thatLaBugal -B'l
aanTr i
bal Associ ation,Inc.v .Ramos62l ai
ddownt heguidel i
nesf orav ali
d
servi
cecont ract,oneofwhi chist hatt her emustexi stagener al l
awf oroil explor ati
onbef orea
servi
cecont ractmaybeent eredint obyt heGov ernment .Thepet it
ionersposi tthatt heser vi
ce
contracti nLaBugal ispresumedt ohav ecompl iedwi ththerequisitesof( a)legisl ati
ve
enactmentofagener allawaf tertheef fectivi
tyoft he1987Const i
tuti
on( suchasRepubl icAct
No.7942, orthePhi li
ppineMi ningLawof1995, gov erni
ngmi ningcont r
act s)and( b)presidenti
al
noti
ficat i
on.Thepet i
ti
oner sthusal leget hatther ulinginLaBugal ,whichi nvolvedmi ning
contract sunderRepubl icActNo.7942, doesnotappl yinthi
scase. 63Thepet iti
oner salsoar gue
thatPr esidentialDecreeNo.87ort heOi l Explorati
onandDev elopmentActof1972cannot
l
egallyj usti
fySC- 46asi tisdeemedt ohav ebeenr epeal edbyt he1987Const itutionand
subsequentlaws,whichenunciatenewpolici
esconcerni
ngtheenvi
ronment.
64Inadditi
on,
peti
ti
onersinG.R.No.180771cl aimthatparagr
aphs2and3ofSect i
on2, Ar
ti
cleXI
Iofthe1987
Consti
tuti
onmandat et heexcl
usiveuseandenjoymentbytheFil
i
pinosofournatur
al
resour
ces,65andpar agraph4doesnotspeakofser v
icecontr
actsbutofFTAAsorFinanci
al
Technical
AssistanceAgr eements.
66

Thepubl i
crespondent sagaincontrover
tthepet i
ti
oner s'cl
aimsandasseveratethatSC-46does
notvi
olateSect ion2,Arti
cleXI
Iofthe1987Const ituti
on.Theyholdt
hatSC-46doesnotf all
underthecov er
ageofpar agr
aph1buti nstead,underpar agraph4ofSecti
on2, Arti
cleXIIofthe
1987Const ituti
ononFTAAs.Theyal soinsi
stthatpar agraphs2and3, whi
chr efert
ot hegrant
ofexclusi
vef ishingri
ghttoFil
ipi
nos,arenotapplicabletoSC- 46asthecontractdoesnotgr ant
excl
usivefishingr i
ghtst
oJAPEXnordoesi tot
herwi seimpingeontheFIDEC' sri
ghtto
pref
erenti
al useofcommunal mar i
neandf ishi
ngr esources.67

Rul
i
ngoft
heCour
t

Ont
hel
egal
i
tyofSer
viceCont
ractNo.46

v
is-
a-v
isSect
ion2,
Art
icl
eXI
Ioft
he1987Const
it
uti
on

Thepet
it
ionersinsi
stt
hatSC-46i
snullandvoi
df orhavi
ngviol
atedSect
ion2,
Art
icl
eXI
Ioft
he
1987Consti
tuti
on,whi
chreadsasfol
l
ows:chanroblesv
irt
ual
l
awlibr
ary

Secti
on2.Al l l
andsoft hepubl i
cdomai n, waters, miner als,coal ,petroleum, andot hermi neral
oil
s,allforcesofpot entialenergy,fisheries,forest sort imber ,wildl
if
e, fl
or aandf auna, andot her
naturalresour cesareownedbyt heSt ate.Wi tht heex cept ionofagr iculturall
ands, allother
naturalresour cesshallnotbeal ienated.Theexpl orati
on, dev elopment ,
andut ili
zationofnat ural
resourcesshal lbeunderthef ullcont r
ol andsuper visi
onoft heSt ate.TheSt at
emaydi rectl
y
undertakesuchact ivi
ti
es,ori tmayent erintoco- product ion, jointvent ure,orproduct ion-sharing
agreement swi t
hFili
pinocitizens,orcor porationsorassoci ationsatl eastsixtypercent um of
whosecapi talisownedbysuchci tizens.Suchagr eement smaybef oraper i
odnotexceedi ng
twenty-fi
vey ears,renewabl efornotmor et hant went y
-fivey ear s,andundersucht er msand
conditi
onsasmaybepr ov
idedbyl aw.I ncasesofwat err i
ght sf ori
rrigation,wat ersuppl y,
fi
sheries,ori ndustr
ialusesot herthant hedev elopmentofwat erpower ,benefici
al usemaybe
themeasur eandl imitofthegr ant.

TheStat
eshallpr
otectt
henati
on'
smari
newealt
hini
tsar
chipel
agi
cwater
s,t
err
itor
ialsea,and
excl
usi
veeconomiczone,andr
eserv
eit
suseandenj
oymentexcl
usi
vel
ytoFi
li
pinocit
izens.

TheCongressmay,byl
aw,all
owsmall
-scal
eutil
i
zati
onofnaturalr
esour
cesbyFil
ipi
nocit
izens,
aswell
ascooperati
vef
ishfar
ming,
wit
hpr i
ori
tytosubsi
stencefi
sher
menandfishworker
si n
ri
ver
s,l
akes,bay
s,andl
agoons.

ThePresi
dentmayenteri
ntoagreementswithforei
gn-ownedcorpor
ati
onsinv
olv
ingeit
her
techni
cal
orfi
nanci
alassi
stanceforl
arge-
scaleexpl
orati
on,dev
elopment,
anduti
li
zat
ionof
miner al
s,petr
oleum, andothermineral
oil
saccordi
ngtothegeneralt
ermsandcondit
ions
providedbylaw, basedonr ealcont
ri
buti
onstotheeconomicgrowthandgener
alwelf
areoft
he
count ry
.Insuchagr eements,t
heStateshall
promotethedevel
opmentanduseoflocal
scientif
icandtechnicalresour
ces.

ThePresi
dentshall
noti
fyt
heCongressofeverycont
ractent
eredint
oinaccor
dancewi
tht
his
prov
isi
on,wi
thi
nt hi
rt
ydaysfr
om i
tsexecut
ion.(Emphasesours.
)

ThisCourthaspr eviouslysett
ledthei ssueofwhet herserv
icecontractsar estil
lall
owedunder
the1987Const it
ution.InLaBugal ,wehel dthatt
hedeleti
onoft hewor ds" servi
cecont r
acts"i
n
the1987Const it
utiondi dnotamountt oabanont hem perse.Infact,i
nt hatdecision,we
quotedinlength,por t
ionsofthedel i
ber ati
onsofthemember softheConst i
tuti
onal
Commi ssion(ConCom)t oshowt hati ndel i
berat
ingonparagraph4, Secti
on2, Art
icleXII
,they
wereactuall
yreferringtoservicecont ractsasunderstoodinthe1973Const i
tut
ion,albei
twith
safet
ymeasur est oeliminateormi nimi zetheabusesprevalentduri
ngt hemar t
iall
awr egi
me, t
o
wit:
chanrobl
esv i
rt
ual l
awlibr
ary

Summat
ionoft
heConCom Del
i
ber
ati
ons

Att
hispoint
,wesum upt
hemat
ter
sest
abl
i
shed,
basedonacar
eful
readi
ngoft
heConCom
del
i
berat
ions,asf
oll
ows:

Intheirdel
i
berati
onsonwhatwast
obecomeparagr
aph4, t
hefr
amersusedt
het er
m servi
ce
contractsi
nrefer
ringt
oagr
eement
sxxxinvol
vi
ngeithert
echni
cal
orfi
nanci
alassi
stance.

Theyspokeofser
vicecont
ract
sast
heconceptwasunder
stoodi
nthe1973Const
it
uti
on.

Itwasobvi
ousf
rom t
hei
rdi
scussi
onst
hatt
heywer
enotaboutt
obanorer
adi
cat
eser
vice
contr
act
s.

Instead,theywer epl
ainlycrafti
ngprovisi
onstoputi nplacesaf eguardst hatwouldeli
minateor
mi nimizetheabusespr evalentduri
ngt hemari
tallawr egime.Inbr i
ef,theywer egoingtopermit
serv i
cecontractswit
hf oreigncorporati
onsascont ractors,butwithsaf etymeasurestoprevent
abuses, asanexceptiont ot hegeneralnor
m establishedi nthefi
rstpar agraphofSecti
on2of
ArticleXII
.Thisprovi
sionr eservesorli
mitstoFi
lipinocitizensandcor por ati
onsatl
east60
per centofwhichisownedbysuchci ti
zens—theexpl orati
on,developmentandut i
l
izat
ionof
nat uralr
esources.

Thisprovi
sionwaspromptedbyt
heper
ceiv
edinsuf
fi
ciencyofFil
i
pinocapit
alandt
hef
eltneed
forfor
eigninvest
mentsi
ntheEDUofminer
alsandpet
roleum r
esources.

Theframersforthemostpar tdebat
edaboutt
hesor tofsafeguardst hatwoul
dbeconsidered
adequateandreasonabl
e.Butsomeoft hem,hav i
ngmor e"radical"l
eanings,wantedtoban
serv
icecontr
actsalt
ogether;fort
hem,t
heprovisionwoul dpermitalienstoexploi
tandbenefi
t
fr
om thenati
on'snatur
alresources,
whi
chtheyf eltshoul
dber eser v
edonl yf
orFil
ipi
nos.
I
ntheexplanat
ionoftheirv
ot es,
theindividualcommi ssi
onerswereheardbytheenti
rebody
.
Theysoundedoffthei
rindi
vidualopinions,openlyenunci
atedthei
rphil
osophi
es,andsuppor
ted
orat
tackedtheprovi
sionswithfervor.Everyone'
sv i
ewpointwasheard.

Inthefinal
vot
ing,t
heArt
icl
eontheNationalEconomyandPat ri
mony—i ncl
udi
ngpar agr
aph4
all
owingservi
cecontr
act
swithfor
eigncorporat
ionsasanex cept
iont
othegeneralnormin
paragraph1ofSecti
on2ofthesamearticl
e—wasr esoundi
nglyappr
ovedbyav ot
eof32t o7,
with2abstent
ions.

Agr
eement
sInv
olv
ingTechni
cal
OrFi
nanci
alAssi
stanceAr
eSer
viceCont
ract
swi
thSaf
eguar
ds

From t hef oregoi ng,wearei mpelledt oconcl udethatthephr aseagr eement sinv olv
ingei ther
technical orf i
nanci alassi
stance,refer r
edtoi nparagraph4, areinf actser vi
cecont racts.But
unli
ket hoseoft he1973v ari
ety,t
henewonesar ebetweenf or
eigncor porationsact ingas
cont r
act orsont heonehand; andont heother ,t
hegov ernmentaspr incipalor"owner "ofthe
wor ks.Int henewser vi
cecont r
act s,theforeigncontractorsprov i
decapi tal,t
echnol ogyand
technical know- how, andmanager ialexpertiseinthecr eati
onandoper ati
onofl arge-scale
mining/ extractiveent erpr
ises;andt hegov ernment ,t
hroughi tsagenci es(DENR, MGB) ,acti
vel
y
exercisescont rol andsupervisi
onov ertheent ir
eoperat i
on.68cral
awl awl i
brary

Insummar izi
ngthematter
sdiscussedintheConCom, weestabl
ishedthatpar agr
aph4,wi
ththe
safeguardsinpl
ace,i
stheexcepti
ont oparagr
aph1,Secti
on2ofAr ti
cleXII.Thefoll
owi
ngare
thesafeguardst
hisCourtenumeratedinLaBugal:
chanrobl
esvi
rt
uall
awl i
brary

Suchservi
cecont
ractsmaybeent
eredint
oonl
ywithr
especttominer
als,
petr
oleum andot
her
mineraloi
l
s.Thegrantt
her
eofi
ssubjectt
osev
eral
safeguar
ds,amongwhicharethese
requi
rements:

(1)Theser
vicecontr
actshal
lbecraftedinaccordancewit
hagenerall
awthatwil
lsetstandar
d
orunif
ormterms,condi
ti
onsandr equir
ements,presumabl
ytoattai
nacer
tainuni
formit
yin
provi
si
onsandav oi
dthepossibl
einserti
onoftermsdisadvant
ageoustot
hecountry.

(2)ThePresidentshall
bethesignatoryforthegov
ernmentbecause,supposedl
ybeforean
agreementispresentedtothePresidentforsi
gnat
ure,
itwi
llhavebeenv et
tedsever
alti
mes
overatdi
ffer
entlevelstoensur
et hatitconfor
mstolawandcanwi thst
andpublicscr
utiny
.

(3)Withi
nthir
tydaysoft heexecutedagr
eement,
thePresi
dentshallr
epor
titt
oCongr esst
o
givethatbr
anchofgov ernmentanopportuni
tyt
olookovertheagreementandint
erposeti
mel
y
objecti
ons,
ifany.
69cralawlawli
brary

Adheri
ngtotheafor
ementi
onedguidel
ines,
thisCour
tfi
ndsthatSC-46i
sindeednul
landv
oid
fornoncompl
iancewit
htherequi
rementsofthe1987Const
itut
ion.

1.TheGener
alLawonOi
lExpl
orat
ion
Thedi spositi
on,explor
ati
on,devel
opment ,exploitati
on,andutil
izat
ionofi ndigenouspetrol
eum
i
nt hePhi li
ppinesaregovernedbyPresidenti
alDecr eeNo.87ort heOi lExplorati
onand
Dev elopmentActof1972.Thi swasenact edbyt henPresidentFerdinandMar costopromote
thedi scov eryandproducti
onofindigenouspet roleum t
hrought heut i
li
zati
onofgov er
nment
and/ orl ocalorforei
gnpri
vateresourcestoy i
eldt hemaximum benef i
ttot heFili
pinopeopl
eand
ther ev enuest othePhi
li
ppineGovernment .
70

Contrarytot
hepetit
ioners'argument
,Presidenti
alDecr
eeNo.87,al
thoughenact
edin1972,
beforetheadopti
onoft he1987Constit
ution,remainst
obeav al
i
dlawunl essot
her
wise
repealed,
towit:
chanroblesvi
rtual
l
awl
ibr
ary

ARTI
CLEXVI
II-TRANSI
TORYPROVI
SIONS

Secti
on3.Al lexi
sti
nglaws, decrees,execut
iveor
ders,
procl
amat i
ons,
lett
ersofi
nstr
ucti
ons,
and
otherexecutiv
eissuancesnoti nconsist
entwitht
hisConsti
tut
ionshal
lremainoper
ati
veunti
l
amended, repeal
ed,orrevoked.

Ift
herewer eanyintent
iontorepeal Presi
dent i
alDecreeNo.87,i
twoul
dhavebeendone
expresslybyCongress.Forinstance,RepublicActNo.7160, mor
epopul
arl
yknownastheLocal
GovernmentCodeof1991, expresslyrepealedanumberofl aws,i
ncl
udi
ngaspeci
fi
cprovi
sion
i
nPr esident
ial
DecreeNo.87, v
iz.:
chanroblesvir
tual
lawli
brar
y

SECTI
ON534.RepealingClause.—(a)BatasPambansaBlg.337,ot
herwi
seknownasthe"Local
Gover
nmentCode,"Execut
iveOrderNo.112(1987)
,andExecuti
veOrderNo.319(
1988)ar
e
her
ebyrepeal
ed.

(
b)Pr
esident
ial
DecreeNos.684,1191,
1508andsuchotherdecrees,
orders,
instr
ucti
ons,
memorandaandissuancesr
elat
edtoorconcer
ningt
hebarangayareherebyrepeal
ed.

(c)Thepr ovi
sionsofSecti
ons2,3,
and4ofRepubl icActNo.1939r egardi
nghospit
alfund;
Secti
on3, a(
3)andb( 2)ofRepubl
icActNo.5447r egardingtheSpeci
alEducat
ionFund;
Presidenti
alDecreeNo.144asamendedbyPr esi
dentialDecreeNos.559and1741;President
ial
DecreeNo.231asamended; Presi
denti
alDecreeNo.436asamendedbyPr esi
dent
ialDecree
No.558; andPr esi
dent
ial
DecreeNos.381,436,464,477, 526,632,
752,and1136arehereby
repealedandrenderedofnoforceandeffect
.

(
d)Pr
esi
dent
ial
Decr
eeNo.1594i
sher
ebyr
epeal
edi
nsof
arasi
tgov
ernsl
ocal
l
y-f
undedpr
oject
s.

(e)Thefoll
owingprovi
sionsar
eherebyrepeal
edoramendedi nsofarast heyareinconsistent
withtheprovi
sionsoft
hisCode:Secti
ons2,16and29ofPr esidenti
alDecr eeNo.704; Section
12ofPr esi
denti
alDecr
eeNo.87,asamended; Sect
ions52,53,66,67, 68,69,70,71,72,73, and
74ofPr esi
denti
alDecr
eeNo.463,asamended; andSecti
on16ofPr esidenti
alDecreeNo.972,
asamended, and

(
f)Al
lgener
alandspeci
all
aws,
act
s,ci
tychar
ter
s,decr
ees,
execut
iveor
der
s,pr
ocl
amat
ionsand
admini
str
ati
veregul
ati
ons,orpartorpart
sther
eofwhichar
einconsi
stentwi
thanyoft
he
prov
isi
onsofthi
sCodear eherebyrepeal
edormodif
iedaccor
dingl
y.(
Emphasissuppl
i
ed.
)

ThisCour tcouldnotsimplyassumet hatwhil


ePr esi
denti
alDecreeNo.87hadnoty etbeen
expresslyrepealed,i
thadbeeni mpli
edlyr
epealed.Aswehel dinVill
areñav .TheCommi ssi
on
onAudi t,
71" [i
]mpli
edrepeal
sarenotlight
lypresumed."I
tisaset tl
edrulethatwhenl awsar ei
n
confli
ctwi t
honeanot her,ev
eryeffor
tmustbeexer t
edtoreconcil
ethem.InRepubl i
coft he
Phil
ippinesv .MarcopperMiningCorporat
ion,
72wesai d:
chanroblesvi
rtual
l
awl ibr
ary

Thet wolawsmustbeabsol utel


yincompat i
ble,andaclearf i
ndingther
eofmustsur face,before
theinfer
enceofi mpliedrepeal maybedr awn.Ther uleisexpressedinthemaxi m,int erpr
etare
etconcordareleqibusestopt imusinterpr
etendi,i
.e.
,everystatutemustbesoi nterpretedand
broughtint
oaccor dwi thot herlawsast oform auniform system ofjur
ispr
udence.The
fundamentisthatt helegislatureshouldbepr esumedt ohav eknownt heexist
inglawsont he
subjectandnothav eenact edconf li
ctingstat
utes.Hence, all
doubtsmustber esolv edagai nst
anyimpliedrepeal,andall effort
sshoul dbeexer t
edinor dertoharmonizeandgi veef fecttoall
l
awsont hesubject.(Cit
ationomi t
ted.)

Mor eover,incaseswherethestatut
eseemst obeinconfl
ictwit
ht heConstit
ution,
buta
constructionthati
tisinhar
monywi ththeConsti
tut
ionisalsopossibl
e,thatconstr
ucti
onshoul
d
bepr eferr
ed.73ThisCourt,
inPangandamanv .Commi ssi
ononEl ecti
ons74expoundingonthis
point,pronounced:chanr
obl
esvi
rtual
lawli
brar
y

Iti
sabasicpreceptinstat
utor
yconstructi
ont hatastat
uteshoul
dbeint
erpret
edinharmony
wit
htheConstit
utionandthatthespir
it
,ratherthanthelet
teroft
helawdeter
minesit
s
const
ruct
ion;
forthatreason,astat
utemustber eadaccordi
ngtoit
sspi
ri
tandintent
,xxx.
(Ci
tat
ionomit
ted.)

Consequentl
y,wefindnomeriti
npet i
ti
oners'
content
ionthatSC-
46i sprohi
bit
edonthegr
ound
thatt
hereisnogenerallawpr
escri
bingthestandardorunif
ormterms,condit
ions,
and
requi
rementsforserv
icecont
ract
sinvolvi
ngoilexpl
orat
ionandextract
ion.

Butnotemustbemadeatt hispoi
ntthatwhil
ePresi
denti
alDecreeNo.87mayser veasthe
generall
awuponwhi chaser vi
cecontr
actforpet
rol
eum expl
orati
onandextract
ionmaybe
authori
zed,aswill
bediscussedbel
ow, t
heexploi
tat
ionanduti
li
zati
onofthisenergyr
esour
cein
thepresentcasemaybeal l
owedonlythroughalawpassedbyCongr ess,
sincetheTañonStr
ait
i
saNI PAS75ar ea.

2.Pr
esi
dentwasnott
hesi
gnat
oryt
oSC-
46andt
hesamewasnotsubmi
tt
edt
oCongr
ess

WhiletheCour
tf i
ndst
hatPresi
dent
ial
DecreeNo.87issuffi
cienttosati
sfyt
herequi
rementofa
generall
aw,t
heabsenceofthetwoothercondi
ti
ons,t
hatthePr esi
dentbeasignat
orytoSC-
46,
andthatCongressbenot
if
iedofsuchcontr
act,
render
sitnullandvoid.

AsSC-
46wasexecut
edi
n2004,
itst
ermsshoul
dhav
econf
ormednotonl
ytot
hepr
ovi
sionsof
Pr
esi
dent
ialDecreeNo.87,butal
sotot
hoseoft
he1987Const
it
uti
on.TheCi
vi
lCode
pr
ovi
des:
chanroblesv
irt
ual
l
awlibr
ary

ARTICLE1306.Thecont
ract
ingpar
tiesmayest abli
shsuchst
ipul
ati
ons,cl
auses,t
ermsand
condi
ti
onsastheymaydeem convenient,pr
ovi
dedt heyar
enotcontr
arytolaw,moral
s,good
cust
oms,publi
corder
,orpubli
cpol
icy.(I
tali
csours.
)

I
nHei
rsofSanMi
guel
v.Cour
tofAppeal
s,
76t
hisCour
thel
dthat
:chanr
obl
esv
irt
ual
l
awl
i
brar
y

I
tisbasicthatt
helawisdeemedwrit
tenintoever
ycontract
.Alt
houghacontractist
hel
aw
betweenthepart
ies,
thepr
ovisi
onsofpositi
vel
awwhi chregul
atecont
ract
saredeemedwr i
tt
en
ther
einandshalll
imi
tandgovernt
herel
ationsbetweenthepart
ies,
xxx.(Cit
ationsomi
tt
ed.
)

Paragraph4, Sect
ion2, Art
icl
eXI Iofthe1987Const it
uti
onr equi
resthatthePr esi
denthimself
enterintoanyservicecontractfortheexplorat
ionofpetroleum.SC-46appear edtohav ebeen
enteredintoandsignedonl ybyt heDOEt hroughit
st henSecretary,
VicenteS.Perez,Jr.
,
contrarytothesaidconst i
tut
ionalrequi
rement .Moreover,publ
icrespondentshav enei
ther
shownnoral l
egedt hatCongresswassubsequent lynotif
iedoftheexecut i
onofsuchcont ract
.

Publicr espondents'
impli
edargumentthatbasedonthe"al
teregopri
ncipl
e,
"theiractsareal
so
thatoft henPr esi
dentMacapagal-
Arr
oyo's,
cannotappl
yinthi
scase.InJosonv .Torres,
77we
explainedt heconceptofthealt
eregopri
ncipl
eorthedoctr
ineofqual
ifi
edpoli
ti
cal agencyand
i
tslimi tinthi
swi se:
chanr
oblesvi
rt
ual
lawl
ibrar
y

Undert hisdoct ri
ne,whichrecognizest heest abli
shmentofasi ngleexecut ive,al
lexecuti
veand
admi ni
st r
ativeor ganizati
onsar eadjunct soft heExecutiveDepartment ,theheadsoft hevari
ous
executivedepar tmentsar eassist
ant sandagent soft heChiefExecut i
ve, and,exceptincases
wher etheChi efExecut i
veisrequiredbyt heConst i
tuti
onorl awtoacti nper sonort he
exigenciesoft hesituati
ondemandt hatheactper sonall
y,t
hemul ti
fari
ousexecut iveand
admi ni
st r
ativef uncti
onsoft heChi efExecut iveareper f
ormedbyandt hrought heexecuti
ve
depar t
ment s, andt heactsoft heSecr etariesofsuchdepar tment s,performedandpr omulgated
i
nt her egularcour seofbusi ness,are,unlessdi sapprovedorr epr
obat edbyt heChiefExecuti
ve
presumpt ivelyt heactsoft heChiefExecut iv
e.(Emphasi sours,citat
ionomi t
ted.)

Whi l
etherequirement si nexecut i
ngser vi
cecontr
actsinpar agr aph4, Secti
on2ofAr ticl
eXIIof
the1987Const i
tuti
onseem l ikemer ef ormali
ti
es,t
hey ,inreali
ty,takeonamuchbi ggerr ol
e.As
wehav eexplainedi nLaBugal ,theyar ethesafeguardsputi npl acebyt heframer soft he
Constit
uti
ont o"eliminat eormi nimizet heabusespr evalentdur i
ngt hemar ti
allawr egime."
78
Thus,theyarenotj ustmer eformal i
ties,whichwil
lonlyr enderacont r
actunenf orceablebutnot
void,i
fnotcompl iedwi t
h.Theyar er equi r
ementsplaced, notjustinanor dinarystatute,buti
n
thefundament allaw, thenon-obser vanceofwhi chwi l
l null
if
yt hecont r
act.Elucidati
ngont he
conceptofa" const i
tution,
"thisCour t, i
nManilaPri
nceHot elv.Gov ernmentSer vi
ceInsurance
System,79held:chanr oblesvi
rtuallawlibrary
Aconst i
tut i
oni sasy stem off undament allawsf orthegov er nanceandadmi nistr
ationofa
nati
on.Itissupr eme, i
mper ious, absoluteandunal t
erableexceptbyt heaut horit
yfrom whi chi
t
emanat es.Ithasbeendef inedast hefundament alandpar amountl awoft henat i
on.I t
prescri
best heper manentf ramewor kofasy st em ofgov ernment ,assignst othedi ff
erent
department st heirrespectivepower sanddut ies,andest ablishescer tainfixedpr i
ncipleson
whichgov ernmenti sfounded.Thef undament alconceptioni not herwor dsi sthatitisa
supremel awt owhi chall otherl awsmustconf orm andi naccor dancewi t
hwhi chal lpri
vate
ri
ghtsmustbedet erminedandal lpublicaut hor i
tyadmi nistered.Undert hedoct ri
neof
constit
utional supremacy ,ifal aworcont ractv i
olat
esanynor m oftheconst it
uti
ont hatlawor
contractwhet herpromul gat edbyt helegislativeorbyt heexecut i
vebr anchorent eredi nt
oby
pri
vateper sonsf orpri
vatepur posesi snul landv oidandwi t houtanyf orceandef fect.Thus,
si
ncet heConst ituti
onist hef undament al,par amountandsupr emel awoft henat i
on, i
tis
deemedwr i
tteni neveryst atuteandcont r
act .(Emphasi sour s.
)

AsthisCourthashel
di nLaBugal,ourConsti
tut
ionrequi
resthatthePresi
denthimsel
fbethe
si
gnatoryofserv
iceagreementswithfor
eign-
ownedcor por
ationsinv
olvi
ngtheexplor
ati
on,
devel
opment ,
andutil
i
zationofourmineral
s,petr
oleum,andot hermi
neraloi
ls.Thi
spower
cannotbetakenli
ghtl
y.

Inthi
scase, t
hepubl icrespondent shavef ai
ledtoshowt hatthePr esi
denthadanypar t i
cipati
on
i
nSC- 46.Thei rargumentt hattheiractsareact ual
lyt
heact soft henPr esi
dentMacapagal -
Arr
oy o,absentpr oofofherdi sapproval,
mustf ai
lasther equi
rementt hatthePr esidenther self
enterintotheseki ndsofcont ractsisembodi ednotjustinanyor dinar
yst atute,butint he
Constituti
onitself.Theseser v i
cecont r
actsinv ol
vi
ngtheexpl oit
ation,
dev elopment ,and
uti
l
izationofournat uralresour cesareofpar amountinteresttothepresentandf uture
generations.Hence, safeguar dswer eputi nplacetoinsurethatthegui deli
nessetbyl awar e
met i
culouslyobser vedandl ikewi setoeradicatethecorrupti
ont hatmayeasi l
ypenet r
at e
department sandagenci esbyensur i
ngt hatthePresi
denthasaut hori
zedorappr ovedoft hese
servi
cecont ractsher sel
f.

Evenundertheprovi
sionsofPr
esi
dent
ialDecreeNo.87,i
tisrequi
redthatt
hePetrol
eum Boar
d,
nowt heDOE,obt
ainthePresi
dent
'
sapprovalfort
heexecutionofanycontr
actundersai
d
stat
ute,asshowninthefol
lowi
ngprov
isi
on:chanr
oblesv
irt
uall
awli
brar
y

SECTION5.Execut ionofcontr actauthori


zedinthisAct.-Ever
ycontractherei
nauthor
izedshall,
subj
ectt otheapprovalofthePr esident,
beexecut edbythePetrol
eum Boardcr eat
edi
nt hisAct,
aft
erduepubl icnoti
cepre-qualifi
cati
onandpubl i
cbi ddi
ngorconcludedthroughnegoti
ations.I
n
casebidsar erequest
edori frequestednobi dissubmi t
tedorthebidssubmi t
tedarer
ejectedby
thePetroleum Boardforbeingdi sadvant
ageoust otheGov er
nment,thecontractmaybe
concl
udedt hr
oughnegotiati
on.

I
nopeni
ngcontractar
easandinsel
ect
ingt
hebestof
ferf
orpet
roleum operat
ions,anyoft
he
f
oll
owi
ngal
ternati
veprocedur
esmayberesor
tedt
obythePetr
oleum Board,subjectt
opri
or
appr
oval
oft
hePr
esi
dent[
.]

Eveni fwewer ei
nclinedtor elaxther equi rementi nLaBugal t
oharmonizethe1987Const it
uti
on
withtheaforement i
onedpr ov i
sionofPr esident i
al DecreeNo.87,i
tmustbeshownt hatthe
gov er
nmentagencyorsubor dinateofficial hasbeenaut hori
zedbythePresidentt
oent eri
nto
suchser vi
cecontractforthegov ernment .Ot herwise,i
tshouldbeatleastshownt hatthe
Presidentsubsequentlyappr ovedofsuchcont ractexplici
tl
y.Noneofthesecircumst ancesis
evidentinthecaseatbar .
chanRobl esvirtual Lawlibrar
y

Ser
viceCont
ractNo.46v
is-
a-v
isOt
herLaws

Petit
ioner sinG.R.No.180771cl ai
mt hatSC- 46v i
olatesSecti
on27ofRepubl i
cAct.No.9147or
theWi ldli
feResour cesConser v at i
onandPr ot ect
ionAct ,whi
chbansal lmarineexplor
ationand
exploit
at i
onofoi landgasdeposi t
s.Theyalsoav ert hatSecti
on14ofRepubl i
cActNo.7586or
theNat ionalIntegratedProtect edAr easSyst em Actof1992( NIPASAct),whichall
owst he
explorationofpr otectedareasf orthepurposeofi nformati
on-gather
ing,hasbeenrepealedby
Section27ofRepubl icActNo.9147.Thesai dpet i
ti
onersfurt
herclaimthatSC- 46i
sanat hema
toRepubl i
cActNo.8550ort hePhi li
ppi
neFi sheriesCodeof1998, whichprotectst
herightsof
thefisher f
olkinthepr efer
ent i
al useofmuni ci palwat er
s,wit
ht heexcepti
onbei ngl
imit
edonl yt
o
researchandsur veyact i
vi
ties.80

TheFI DEC, f
oritspar t,arguest hatt oav ai loft heexcept i
onsunderSect i
on14oft heNI PASAct,
thegat her i
ngofi nformat ionmustbei naccor dancewi t
haDENR- approv edpr ogr am, andthe
expl oi
t ationandut ili
zationofener gyr esour cesmustbepur suanttoagener al l
awpassedby
Congr essexpr esslyf orthatpur pose.Si ncet her eisnei theraDENR- approv edpr ogram nora
gener al l
awpassedbyCongr ess, thesei smi csur veysandoi ldril
li
ngoper ationswer ealldone
i
llegally .81TheFI DECl i
kewi secont endst hatSC- 46i nfr
ingesoni t
sr i
ghtt ot hepr eferenti
aluse
ofthecommunal f
ishingwat ersasi ti
sdeni edf r
eeaccesswi thinthepr ohi bit
edzone, in
violati
onnotonl yoft heFi sheriesCodebutal sooft he1987Const i
tuti
onal provisionson
subsi st encef i
sherfolkandsoci al justi
ce. 82Fur thermor e,theFIDECbel iev est hatthepr ovi
sions
i
nPr esi dentialDecreeNo.87, whi chal lowof fshor edr il
l
ingev eninmuni cipal wat ers,shouldbe
deemedt ohav ebeenr ender edinoper ativ ebyt hepr ov i
sionsofRepubl i
cActNo.8550and
Republ icActNo.7160, whi chrei t
er atet hesoci al justicepr ovi
sionsoft heConst it
ution.83

Thepubli
crespondent
sinvoketherulesonstatutor
yconst
ruct
ionandarguethatSection14of
theNIPASActisamor epart
icul
arprovisi
onandcannotbedeemedt ohavebeenr epealedby
themoregeneralpr
ohibi
ti
oninSection27ofRepubl i
cActNo.9147.TheyaverthatSection14,
underwhi
chSC- 46fal
l
s,shouldi
nsteadber egardedasanexempti
ontoSection27.84

Addressingthecl
aim ofpeti
ti
onersinG. R.No.180771thattherewasav iol
ati
onofSect ion27
ofRepubl i
cActNo.9147,thepubli
cr espondentsasser
tthatwhatt hesecti
onpr ohibi
tsi
st he
expl
orationofmineral
s,whichasdef i
nedi nthePhi
li
ppi
neMi ningActof1995, excludeenergy
materi
alssuchascoal ,
petrol
eum, naturalgas,r
adi
oact
ivemat eri
alsandgeother malenergy.
Thus,sinceSC-46invol
vesoilandgasexpl orat
ion,
Secti
on27doesnotappl y.
85
Thepublicrespondent sdefendt hevali
dit
yofSC- 46andi nsistthatitdoesnotgr antexclusive
fi
shingri
ghtst oJAPEX; hence, i
tdoesnotv iol
atetheruleonpr ef
erentialuseofmuni cipal
waters.Moreov er
,theyall
eget hatJAPEXhasnotbannedf ishingint heprojectarea,contrar
yto
theFIDEC'sclaim.Thepubl i
cr espondentsalsocontesttheat tri
buti
onoft hedecli
ningf i
shcatch
totheseismicsur veysandav erthattheall
egat i
onisunfounded.Theycl aimt hataccordingto
theBureauofFi sheri
esandAquat icResources'fi
shcatchdat a, t
her educedfishcatchst art
edin
the1970sduet odestruct
ivefishingpract
ices.86

Rul
i
ngoft
heCour
t

Ont
hel
egal
i
tyofSer
viceCont
ractNo.46v
is-
a-v
isOt
herLaws

Althoughwehav eal readyest ablishedabov ethatSC-46i snullandvoidforbei ngv i


olati
veoft he
1987Const i
tut
ion,itisourdut ytost i
ll
ruleont helegali
tyofSC- 46vi
s-a-vi
sot herpertinentlaws,
toserv easagui def ortheGov ernmentwhenexecut i
ngser vi
cecontractsinvolvi
ngnotonl ythe
TañonSt r
ait,butalsoot hersimi l
arareas.Whi let
hepet it
ionersall
egethatSC- 46isi nv i
olati
on
ofsev erallaws, i
ncludingi nternationalones,thei
rargument sfocuspr i
marilyont hepr otected
statusoft heTañonSt rait
,thust hisCourtwi l
lconcentrateont hosel
awst hatper tai
npar ticul
arl
y
totheTañonSt rai
tasapr otectedseascape.

TheTañonSt r
aitisanar rowpassageofwat erboundedbyt hei sl
andsofCebui ntheEastand
Negrosi nt heWest .Itharborsar ichbiodiversit
yofmar i
neli
f e,incl
udingendanger edspeciesof
dolphinsandwhal es.Fort hisreason, f
ormerPr esidentFidel V.Ramosdecl aredtheTañonSt rait
asapr ot
ect edseascapei n1998byv ir
tueofPr oclamat i
onNo.1234-Decl aringtheTañonSt rai
t
sit
uatedi nt hePr ovincesofCebu, Negr osOcci dent alandNegr osOrientalasaPr otect
edAr ea
pursuantt ot heNI PASActandshal lbeknownasTañonSt r
ai tProtectedSeascape.Dur i
ng
formerPr esi dentJosephE.Est rada'stime, heal soconst i
tutedt heTañonSt r
aitCommi ssionv i
a
Execut i
veOr derNo.76t oensur et heopti
mum andsust ai
neduseoft heresourcesinthatarea
withoutthr eateningi t
smar i
nelife.Hef ol
lowedt hi
swi t
hExecut i
veOr derNo.177,87wher einhe
i
ncludedt hemay orofNegr osOcci dentalMuni cipalit
y/Cit
yasamemberoft heTañonSt rait
Commi ssion, torepresentt heLGUsconcer ned.Thi sCommi ssion,howev er,wassubsequent ly
aboli
shedi n2002byt henPr esidentGlori
aMacapagal -
Arroy
o, v i
aExecut i
veOr derNo.72.88

Truet otheconst i
tuti
onal policythatthe"Stateshal lpr otectandadv ancet her i
ghtoft hepeopl e
toabal ancedandheal thfulecologyi naccordwi t
ht her hythm andhar monyofnat ure,"89
Congr essenact edt heNI PASActt osecuretheper pet ual existenceofal l nativeplantsand
animal sthrought heest ablishmentofacompr ehensi vesy stem ofi ntegr atedpr otectedar eas.
Thesear easpossesscommonecol ogi
calvaluest hatwer eincor por atedi ntoahol isti
cpl an
represent at
iveofournat uralher i
tage.Thesy stem encompassesout standi nglyremar kable
areasandbi ologicall
yimpor t
antpubl i
clandst hatar ehabi t
atsofr ar eandendanger edspeci es
ofplant sandani mals,biogeogr aphiczonesandr elatedecosy st
ems, whet herterrestri
al,
wetland, ormar ine.90Itclassifi
esandadmi nistersal lthedesi gnat edpr otectedar east o
maintainessent i
alecologi calprocessesandl i
fe-
suppor tsy stems, topr eser vegenet i
cdiv ersi
ty,
toensuresust
ainabl
euseofr esour
cesfoundtherei
n,andtomaint
aint
heirnat
ural
condit
ionst
o
thegreat
estext
entpossibl
e.91Thefol
lowingcategori
esofpr
otect
edareaswereestabl
i
shed
undertheNIPASAct:chanr
oblesvi
rt
ual
lawli
brar
y

St
ri
ctnat
urer
eser
ve;

Nat
ural
par
k;

Nat
ural
monument
;

Wi
l
dli
fesanct
uar
y;

Pr
otect
edl
andscapesandseascapes;

Resour
cer
eser
ve;

Nat
ural
biot
icar
eas;
and

Othercategor
iesest
abl
i
shedbylaw,convent
ionsori
nter
nat
ional
agr
eement
swhi
cht
he
Phil
ippi
neGov er
nmenti
sasignator
y.92

UnderSecti
on4oft heNIPASAct,aprot
ectedareareferstoport
ionsofl
andandwater
,set
asi
deduet othei
runiquephysi
calandbi
ologi
calsigni
ficance,
managedtoenhancebi
ologi
cal
di
versi
tyandprotect
edagainsthumanexploi
tat
ion.

TheTañonSt rai
t,pursuantt oPr oclamati
onNo.1234, wassetasi deanddecl aredapr otected
areaundert hecat egoryofPr otectedSeascape.TheNI PASActdef i
nesaPr otectedSeascapet o
beanar eaofnat ionalsi gnif
icancechar acteri
zedbyt hehar moni ousinter
actionofmanandl and
whileprovi
dingoppor t
uni ti
esf orpubl i
cenjoymentt hroughr ecreati
onandt ourism withinthe
normal l
if
esty l
eandeconomi cact i
vit
yofthisareas;93t husamanagementpl anf oreachar ea
mustbedesi gnedt oprot ectandenhancet heper manentpr eservati
onofit
snat ural
conditi
ons.94Consi stentwi tht hisendeavoristherequi r
ementt hatanEnv i
ronment alImpact
Assessment( EI
A)bemadepr i
ort oundert
akinganyact ivi
tyoutsidethescopeoft he
managementpl an.Unl essanECCundert heEIAsy stem i sobtained,noacti
v i
tyinconsistent
withthegoal softheNI PASActshal lbei
mpl emented. 95

TheEnv i
ronment al I
mpactSt atementSy stem ( EISS)wasest ablishedin1978underPr esi
denti
al
DecreeNo.1586.I tprohibi
tsanyperson, par t
nershiporcor por ationfrom undert
akingor
operat
inganydecl aredenv ir
onment all
ycriticalprojectorareaswi t
houtfir
stsecuringanECC
i
ssuedbyt hePr esidentorhi sdulyauthori
z edr epresentati
ve.96Pur suanttotheEISS,which
call
edforthepropermanagementofenv ironment all
ycrit
ical areas,97Proclamati
onNo.214698
wasenact ed,i
dent if
yingtheareasandt ypesofpr oj
ectstobeconsi deredasenvironmentall
y
cri
ti
calandwi t
hint hescopeoft heEISS,whi leDENRAdmi nistrativeOrderNo.2003- 30provi
ded
fori
tsImpl ementingRul esandRegul ati
ons( I
RR) .

DENRAdmi
nist
rat
iveOr
derNo.2003-
30def
inesanenv
ironment
all
ycr
it
ical
areaas"
anar
ea
deli
neatedasenv i
ronmentall
ysensiti
vesucht hatsignifi
cantenv i
ronment alimpactsare
expectedifcer
tai
nt ypesofproposedpr ojectsorprogr amsar elocat ed,dev el
oped, or
i
mpl ementedinit"
;99thus,bef
oreapr oject,whichis" anyact i
vi
ty,regardlessofscal eor
magnitude,whichmayhav esignifi
cantimpactont heenv i
ronment,"100i sunder t
akeninit
,such
proj
ectmustunder goanEIAt oev al
uateandpr edi
ctthel ikel
yimpact sofal l i
tsstagesonthe
envi
ronment .
101AnEI Aisdescribedindet ai
l asfoll
ows: chanrobl
esv i
rt
uallawlibr
ary

h.

Environmental I
mpactAssessment( EIA)-pr
ocessthatinvol
vesev al
uatingandpr edi
ctingthe
l
ikelyimpactsofapr oject(incl
udingcumul ati
veimpacts)ontheenv ir
onmentdur i
ng
const r
ucti
on,commi ssioning,operationandabandonment .I
talsoincludesdesigning
appropriatepreventi
v e,mitigat
ingandenhancementmeasur esaddr essingtheseconsequences
toprotecttheenv ir
onmentandt hecommuni t
y'
swelfare.Theprocessi sundertakenby, among
others,theprojectproponentand/ orEIAConsult
ant,EMB, aRev i
ewCommi ttee,aff
ected
communi t
iesandot herst akeholders.102

UnderProcl
amati
onNo.2146, t
heTañonStrai
tisanenvir
onment all
ycri
ti
calarea,
hav i
ngbeen
decl
aredasaprotect
edareain1998;t
heref
ore,anyact
ivi
tyoutsidethescopeofits
managementplanmayonlybei mpl
ementedpursuantt
oanECCsecur edafterundergoingan
EIAtodeter
minetheeff
ectsofsuchact
ivi
tyonitsecol
ogical
sy stem.

Thepubl
icrespondent
sarguethatt
heyhadcompli
edwit
hthepr
ocedur
esinobt
aini
ngan
ECC103andt hatSC-
46fal
lsundert
heexcept
ionsi
nSect
ion14oft
heNIPASAct,duet
othe
fol
l
owingreasons:

1)TheTañonSt
rai
tisnotast
ri
ctnat
urer
eser
veornat
ural
par
k;

2)Expl
orat
ioni
sonl
yfort
hepur
poseofgat
her
ingi
nfor
mat
iononpossi
bleener
gyr
esour
ces;
and

3)Measuresar
eundertakent
oensur
ethatt
heexpl
orat
ioni
sbei
ngdonewi
tht
hel
eastdamage
tosur
roundi
ngareas.
104

Wedonotagr
eewi
tht
hear
gument
srai
sedbyt
hepubl
i
crespondent
s.

Sect
ions12and14oft
heNI
PASActr
ead:
chanr
obl
esv
irt
ual
l
awl
i
brar
y

SECTION12.Envir
onmentalImpactAssessment .-Pr
oposalsforact
ivi
ti
eswhichareout
sidethe
scopeofthemanagementpl anforpr
otectedareasshal
lbesubjecttoanenvir
onmental
impact
assessmentasrequi
redbylawbef or
et heyar
eadopted,andtheresult
sther
eofshall
betaken
i
ntoconsider
ati
oninthedecisi
on-makingprocess.

Noact
ual i
mpl
ementati
onofsuchacti
vi
ti
esshallbeal
l
owedwithoutt
herequir
edEnvir
onment
al
Compl
ianceCer
ti
fi
cate(ECC)undert
hePhil
i
ppineEnvi
ronment
alImpactAssessment(
EIA)
syst
em.Ini
nstanceswher esuchacti
vi
ti
esareall
owedtobeunder taken,t
heproponentshal
l
pl
anandcarrythem outi
nsuchmanneraswi l
lmini
mizeanyadv er
seef f
ectsandtake
prev
ent
iveandremedialacti
onwhenappr opr
iat
e.Theproponentshallbeli
abl
eforanydamage
duetol
ackofcautionorindi
scret
ion.

SECTI ON14.Sur veyf orEnergyResources.-Consistentwiththepol ici


esdeclaredinSect i
on2
hereof,protectedar eas,exceptstr
ictnatur
ereser v
esandnat ur
al parks,maybesubj ectedto
explorati
ononl yfort hepurposeofgat heri
nginformat i
ononener gyr esourcesandonl yifsuch
acti
vityiscarri
edoutwi t
htheleastdamaget osur roundingareas.Sur veysshallbeconducted
onlyinaccor dancewi thaprogram approvedbyt heDENR, andther esultofsuchsurveysshall
bemadeav ail
abletot hepubl i
candsubmi tt
edtot hePr esi
dentforr ecommendat i
onto
Congr ess.Anyexpl oitati
onandut i
li
zationofenergyr esourcesfoundwi t
hinNIPASareasshal l
beal l
owedonl ythr oughal awpassedbyCongr ess.

I
tistruethattherestr
icti
onsfoundundertheNI PASActar enotwithoutexcepti
ons.However
,
whil
eanex plorat
iondonef orthepurposeofsurvey i
ngforener
gyresourcesisall
owedunder
Secti
on14oft heNI PASAct ,t
hisdoesnotmeant hatiti
sexemptfrom therequi
rementto
undergoanEI AunderSect i
on12.InSottov.Sotto,105thi
sCourtexplai
nedwhyast atut
eshould
beconstr
uedasawhol e:
chanrobl
esvir
tual
lawl
ibrary

Ast atut
eispassedasawhol eandnoti npar tsorsect ionsandi sani mat edbyonegener al
pur poseandi ntent.Consequent lyeachpar torsect i
onshoul dbeconst ruedi nconnect i
onwi t
h
ev eryotherpartorsect ionandsoast oproduceahar moni ouswhol e.Itisnotpr opertoconf i
ne
theat t
enti
ontot heonesect i
ont obeconst rued.Itisal waysanunsaf ewayofconst r
uinga
statut eorcontr
actt odi vi
deitbyapr ocessofet ymol ogicaldissecti
on, intosepar atewords,and
thenappl ytoeach, t
hussepar at edf rom itscont ext,somepar ti
culardef ini
tiongiv enby
l
exi cographers,andt henr econst ructt heinstrumentupont hebasi soft hesedef initi
ons.An
i
nst rumentmustal way sbeconst r uedasawhol e,andt hepar ti
cularmeani ngt obeat t
achedt o
anywor dorphrasei susual lytobeascer tainedfrom t hecont ext,t
henat ur
eoft hesubj ect
treat edofandthepur poseori ntent ionoft hepar t
ieswhoexecut edt hecont r
act, orofthebody
whi chenact edorframedt hest atut eorconst i
tut
ion,xxx.

Surv
ey i
ngforenergyresour
cesunderSecti
on14isnotanexemptionfrom complyi
ngwitht
he
EIArequir
ementinSection12;inst
ead,
Secti
on14provi
desforaddit
ional
requisi
tesbef
oreany
expl
orati
onforenergyresourcesmaybedoneinprot
ectedareas.

Therati
onal
eforsuchaddi
tional
requi
rement
sar
eincor
por
atedi
nSect
ion2oft
heNI
PASAct
,to
wit
:chanr
obl
esvi
rtual
l
awli
brary

SECTI ON2.Declar
ationofPol icy-Cogni zantoft heprof
oundimpactofman' sactiv
iti
esonall
component softhenaturalenv i
ronmentpar t
icularl
ytheeff
ectofincreasi
ngpopulati
on,
resourceexpl
oit
ati
onandi ndustri
aladv ancementamdr ecogni
zingthecrit
ical
impor t
anceof
protecti
ngandmai nt
aini
ngt henat ur
al biologicalandphysi
caldi
v er
siti
esoftheenvir
onment
notablyonareaswithbiologicall
yuniquef eaturestosustai
nhumanl if
eanddev el
opment,as
wellasplantandani mal l
if
e,iti
sherebydeclaredthepoli
cyoft heStatetosecurefortheFi
li
pino
peopleofpr esentandfuturegenerati
onstheper pet
ualexi
stenceofal lnati
vepl
antsand
ani
mal st
hr ought heestabl
ishmentofacompr ehensi
vesystem ofintegratedpr
otect
edareas
wit
hinthecl assi
ficat
ionofnat i
onalpar
kaspr ovidedfori
ntheConst i
tution.

Iti
sherebyr ecognizedt hatthesear eas,
al t
houghdistincti
nf eatur
es, possesscommon
ecologi
calvaluest hatmaybei ncorporatedintoaholisti
cpl anrepresentati
veofournatural
heri
tage;t
hatef fecti
veadmi ni
str
ationoft hisareai
spossi bleonlythroughcooper ati
onamong
nati
onalgov ernment ,
localgov ernmentandconcer nedpr ivateorganizati
ons;thattheuseand
enj
oy mentoft hesepr otectedareasmustbeconsi stentwitht heprincipl
esofbiologi
caldi
versi
ty
andsustainabledev elopment .

Tot hisend,thereisherebyestabl
ishedaNat i
onal I
ntegratedProtectedAreasSystem (NIPAS),
whichshal lencompassout st
andinglyremarkableareasandbi ological
lyimport
antpubli
cl ands
thatar ehabit
atsofr ar
eandendanger edspeciesofpl antsandanimal s,bi
ogeographi
czones
andr elatedecosystems,whetherterr
estr
ial
,wetlandormar i
ne,allofwhichshallbedesignated
as"pr otect
edareas."

Thepubl i
crespondentsthemsel
vesadmi ttedt hatJAPEXonl ystart
edt osecureanECCpr iort
o
thesecondsub- phaseofSC-46,
whi chrequir
edt hedr i
l
lingofanoi lexpl
orati
onwell.Thi
smeans
thatwhent heseismicsurvey
swer edonei ntheTañonSt rai
t,nosuchenv i
ronmentalimpact
evaluati
onwasdone.Unl esssei
smi csurveysar epartofthemanagementpl anoftheTañon
Strai
t,suchsurveysweredonainv i
olat
ionofSect ion12oft heNI PASActandSect ion4of
Presidenti
alDecreeNo.1586,whichprovides:chanroblesvir
tuall
awlibrar
y

Section4.Pr esident ialProclamati


onofEnv i
ronment al
lyCr i
ti
cal AreasandPr oj
ects.-The
Presidentoft hePhi l
ippinesmay ,onhi sowni nit
iat
iveoruponr ecommendat ionoft heNat ional
Environment al Protect i
onCouncil,bypr ocl
amat i
ondecl arecertainpr oject
s, undertakingsor
areasi nt hecount ryasenv i
ronment al
lycri
ti
cal.Noper son,partnershiporcor porationshal l
under takeoroper at eanysuchdecl aredenvir
onment allycri
ti
cal projectorar eawi thoutf i
rst
securinganEnv i
ronment alComplianceCer t
ifi
cateissuedbyt hePr esidentorhi sdul yauthorized
represent ati
ve.Fort hepr opermanagementofsai dcri
ticalproj
ectorar ea,thePr esidentmayby
hisprocl amat ionr eor ganizesuchgov ernmentof fi
ces,agencies, i
nstituti
ons, corporationsor
i
nstrument al
itiesi ncludingt her
e-ali
gnmentofgov ernmentper sonnel ,andt heirspeci f
ic
functionsandr esponsi bil
iti
es.

Forthesamepur poseasabov e,theMi ni


st r
yofHumanSet tlement sshall:(
a)preparetheproper
l
andorwat erusepatter
nf orsai dcrit
icalproject(
s)orarea(s); (
b)establi
shambi ent
environmentalquali
tyst
andar ds; (c)developapr ogram ofenv ironmentalenhancementor
protecti
vemeasur esagainstcal amitousf act
or ssuchasear thquakes,floods,waterer
osionand
others,and(d)perfor
m suchot herfunctionsasmaybedi rect edbythePr esidentfr
om ti
met o
ti
me.

Ther
espondent
s'subsequentcompl
i
ancewi
tht
heEI
SSf
ort
hesecondsub-
phaseofSC-
46
cannotandwill
notcurethi
sviol
ati
on.Thef
oll
owingpenal
ti
esar
epr
ovi
dedf
orunder
Presi
denti
alDecreeNo.1586andtheNIPASAct.

Sect
ion9ofPresi
denti
alDecr
eeNo.1586prov
idesf
ort
hepenal
tyi
nvol
vi
ngv
iol
ati
onsoft
he
ECCrequi
rement:
chanr
oblesv
irt
ual
l
awli
brar
y

Section9.Penal t
yf orViolat
ion.-Anyperson,corporat
ionorpartnershi
pf oundvi
olat
ingSecti
on
4oft hisDecr ee,ort hetermsandcondi ti
onsintheissuanceoftheEnv i
ronmentalCompli
ance
Certi
ficate,oroft hest andards,r
ulesandregulati
onsissuedbytheNat i
onalEnvi
ronmental
ProtectionCounci lpur suanttothisDecr
eeshal l
bepuni shedbythesuspensi onorcancel
lat
ion
ofhis/itscertif
icatesand/ oraf i
neinanamountnott oexceedFiftyThousandPesos
(P50,000.00)f orev eryviol
ati
ont her
eof,
atthediscreti
onoft heNationalEnvir
onmental
ProtectionCounci l.(Emphasi ssuppli
ed.
)

Violat
ionsoftheNIPASActent
ail
sthef
oll
owi
ngf
inesand/
ori
mpr
isonmentunderSect
ion
21:chanrobl
esvi
rt
uall
awl
ibr
ary

SECTI ON21.Penal ties.-Whoev erviol


atest hisActoranyr ulesandr egul ationsi ssuedbyt he
Depar tmentpur suantt ot hisActorwhoev eri sfoundgui l
tybyacompet entcour tofjust iceof
anyoft heof f
ensesi nt hepr ecedingsect i
onshal lbef inedi ntheamountofnotl esst hanFi ve
thousandpesos( P5, 000)normor et hanFi vehundr edt housandpesos( P500, 000) ,exclusiv eof
thev alueoft hethi ngdamagedori mprisonmentf ornotl esst hanone( 1)y earbutnotmor ethan
six(6)y ears,orbot h, asdet er minedbyt hecour t:
Pr ovided, that,i
fthear ear equi res
rehabilit
ationorr estor ationasdet erminedbyt hecour t, t
heof fendershal lber equiredtor estore
orcompensat efort her estor ati
ont othedamages: Prov i
ded, fur
ther,t
hatcour tshal lordert he
evicti
onoft heof f
enderf rom t helandandt hef orfeitureinf avoroftheGov ernmentofal l
miner als,ti
mberoranyspeci escol l
ectedorr emov edincl udingallequipment , dev i
cesand
fi
rearmsusedi nconnect i
ont herewi t
h,andanyconst r
uct i
onori mpr ovementmadet her eonby
theof fender.Iftheof fenderi sanassoci ati
onorcor por ati
on, thepresi
dentormanagershal lbe
directl
yr esponsiblef ort heactofhi sempl oy eesandl abor ers:Provi
ded, f
inally,thatt heDENR
mayi mposeadmi nistrat i
v efinesandpenal tiesconsi stentwi t
hthisAct .(Emphasessuppl ied.)

Mor eov er,


SC-46wasnotexecut edf orthemer epur poseofgat heringinfor
mat i
onont he
possibleener gyr esourcesintheTañonSt rai
tasital sopr ovi
desf orthepar t
ies'r
ightsand
obligationsrelatingtoextracti
onandpet roleum product i
onshoul doilincommer ci
al quantit
ies
bef oundt oexi stinthearea.Whi lePr esidentialDecr eeNo.87mayser veasthegener allaw
uponwhi chaser vi
cecont r
actf orpet rol
eum expl orat i
onandext r
act i
onmaybeaut horized,the
exploitati
onandut il
izat
ionoft hisener gyr esourceint hepr esentcasemaybeal l
owedonl y
throughal awpassedbyCongr ess, sincet heTañonSt r
aitisaNI PASar ea.
106Sincet hereisno
suchl awspeci f i
call
yallowingoi lexplorati
onand/ orext r
actionint heTañonSt rait,
noener gy
resourceexpl oitati
onandut i
l
izationmaybedonei nsaidpr otectedseascape.

I
nviewoftheforegoi
ngpremi
sesandconcl
usi
ons,
itisnolongernecessar
ytodi
scusst
he
ot
heri
ssuesraisedint
heseconsol
i
dat
edpeti
ti
ons.
cral
awred
WHEREFORE, thePeti
ti
onsinG.R.Nos.180771and181527ar
eGRANTED,Serv
iceCont
ractNo.
46isherebydeclar
edNULLANDVOI Dforvi
olat
ingt
he1987Const
it
uti
on,
Republi
cActNo.7586,
andPresident
ial
DecreeNo.1586.

SOORDERED.
chanr
obl
esv
irt
ual
l
awl
i
brar
y

G.
R.No.113375 May5,
1994

KILOSBAYAN,
INCORPORATED, JOVITOR.SALONGA,CIRI
LOA.RIGOS,ERMECAMBA,EMILI
O
C.CAPULONG,JR.,
JOSET.APOLO, EPHRAIM TENDERO,FERNANDOSANTIAGO,
JOSE
ABCEDE,CHRI
STINETAN, FELIPEL.GOZON, RAFAELG.FERNANDO,RAOULV.VI
CTORINO,
JOSECUNANAN, QUINTI
NS.DOROMAL, SEN.FREDDI
EWEBB, SEN.WI
GBERTOTAÑADA,and
REP.JOKERP.ARROYO, 
petit
ioners,
 

v
s.

TEOFISTOGUINGONA,JR.,
inhi
scapacit
yasExecutiv
eSecretary,Off
iceofthePresi
dent;
RENATOCORONA, i
nhiscapaci
tyasAssist
antExecuti
veSecretaryandChairmanoft he
Presi
dent
ial
revi
ewCommi t
teeontheLott
o,Off
iceofthePresident;
PHILIPPINECHARI TY
SWEEPSTAKESOFFICE;andPHILI
PPINEGAMI NGMANAGEMENTCORPORATI ON, r
espondent
s.

Jov
itoR.Sal
onga,
Fer
nandoSant
iago,
Emi
l
ioC.Capul
ong,
Jr.andFel
i
peL.Gozonf
orpet
it
ioner
s.

Renat
oL.Cay
etanoandEl
eazarB.Rey
esf
orPGMC.

Gamal
i
elG.Bongco,
OscarKar
aanandJedi
deohSi
ncer
ofori
nter
venor
s.

DAVI
DE,
JR.
,
 J.
:

Thisisaspeci alci
vi
lacti
onforpr ohibi
ti
onandi nj
uncti
on,withapr ayerf
orat emporar
y
rest
rainingorderandprel
iminaryinjunct
ion,whichseekstoprohibitandrestr
ainthe
i
mpl ement at
ionofthe"ContractofLease"executedbyt hePhili
ppineCharit
ySweepstakes
Offi
ce( PCSO)andt hePhil
ippineGami ngManagementCor porati
on( PGMC)inconnectionwi
th
theon-linelott
erysyst
em, al
soknownas" l
ott
o."

Petit
ionerKil
osbayan,I
ncorporated( KI
LOSBAYAN)av er sthatiti
sanon- stockdomest i
c
corporati
oncomposedofci vic-
spi r
it
edciti
zens,pastors, pri
ests,nuns,andl ayleaderswhoar e
commi ttedtothecauseoftruth,justi
ce,andnationalrenewal .Ther estoft hepet i
tioners,except
SenatorsFreddieWebbandWi gber t
oTañadaandRepr esentati
veJokerP.Ar royo,aresui ngin
thei
rcapaciti
esasmember soft heBoar dofTr usteesofKI LOSBAYANandast axpayersand
concernedciti
zens.Senat
orsWebbandTañadaandRepr esentativeArroy oaresui ngint heir
capaciti
esasmember sofCongressandast axpay er
sandconcer nedcitizensoft hePhi li
ppines.

Thepleadingsoft
hepar
ti
esdi
scl
oset
hef
act
ual
ant
ecedent
swhi
cht
ri
gger
edof
fthef
il
ingof
thi
spetit
ion.
Pur suantt oSect ion1oft hechar teroft hePCSO( R.A.No.1169, asamendedbyB. P.Bl g.42)
whi chgr ant si ttheaut horityt ohol dandconduct" char i
tysweepst akesr aces, l
ot teriesandot her
simi l
aract i
vities,"thePCSOdeci dedt oest ablishanon-l inel otterysy stem f ort hepur poseof
i
ncr easingi tsr ev enuebaseanddi versifyingitssour cesoff unds.Somet imebef oreMar ch1993,
afterlear ningt hatt hePCSOwasi nterestedi noper atinganon- li
nel otterysy stem, theBer j
aya
GroupBer had, "amul ti
national companyandoneoft het enl argestpubl iccompani esi n
Mal aysia,"long" engagedi n, amongot her s,successf ullot teryoper ati
onsi nAsi a, runni ngbot h
Lot t
oandDi gi tgames, t
hrui tssubsi diary, Spor t
sTot oMal aysia,"wi thits" affi
liate, the
International Tot alizatorSy st ems, Inc.,...anAmer icanpubl iccompanyengagedi nt he
i
nter national saleorpr ovisionofcomput ersy stems, softwar es, t
er mi nals, t
rainingandot her
technical serv i
cest ot hegami ngindust r
y,""becamei nterest edt oof feritsser vicesand
resour cest oPCSO. "Asani nitialstep, Ber j
ayaGr oupBer had( throughi tsi ndividual nomi nees)
organi zedwi thsomeFi l
ipinoi nvestorsi nMar ch1993aPhi l
ippinecor por ati
onknownast he
Phi l
ippineGami ngManagementCor por ation(PGMC) , whi ch" wasi ntendedt obet hemedi um
throughwhi cht het echnical andmanagementser vi
cesr equi redf ort hepr oj
ectwoul dbeof fered
anddel i
v eredt oPCSO. "
 1

Befor
eAugust1993,t
hePCSOformal
lyi
ssuedaRequestforPr
oposal(RFP)fort
heLease
Contr
actofanon-
li
nelot
ter
ysy
stem f
orthePCSO.
 2 
Relev
antprovi
sionsoftheRFParethe
fol
l
owing:

1. EXECUTI
VESUMMARY

xxx xxx xxx

1.2. PCSOi sseeki


ngasui tablecontr
actorwhi
chshall
build,
atit
sownexpense, allt
he
faci
li
ti
es( '
Faci
l
iti
es'
)neededtooper ateandmaintai
nanationwideon-l
i
nelott
erysystem.PCSO
shal
lleasetheFacil
it
iesforaf i
xedpercent
ageofquart
erl
ygrossrecei
pts.Al
lrecei
ptsfrom
ti
cketsalesshall
beturnedov erdir
ectl
ytoPCSO.Allcapi
tal,
operat
ingexpensesandexpansion
expensesandr i
sksshallbefortheexclusi
veaccountoft
heLessor.

xxx xxx xxx

1.
4. Thel
easeshal
lbef
oraper
iodnotexceedi
ngf
if
teen(
15)y
ear
s.

1.5. TheLessori sexpect


edtosubmi tacomprehensi
venationwi
delott
erydevel
opmentplan
("
DevelopmentPlan"
)whichwill
incl
udet hegame,t
hemar ket
ingofthegames,andthelogi
sti
cs
toint
roducethegamest oal
lthecit
iesandmunici
pali
ti
esofthecountrywit
hinfi
ve(5)y
ears.

xxx xxx xxx

1.7. TheLessorshal lbeselectedbasedonitst


echni
calexper
tise,har
dwareandsoft
war e
capabil
ity
,maintenancesuppor t
,andf i
nanci
alr
esour
ces.TheDevelopmentPlanshal
lhavea
substanti
albear
ingont hechoiceoft heLessor
.TheLessorshal
lbeadomest i
ccorpor
ation,
withatleastsi
xtypercent(60%)ofi t
ssharesownedbyFil
ipi
noshar ehol
der
s.
xxx xxx xxx

TheOffi
ceofthePr esi
dent,
theNationalDisasterContr
olCoordi
nati
ngCouncil
,thePhil
ippi
ne
Nati
onalPol
i
ce, andtheNational
Bur eauofInvesti
gati
onshallbeauthor
izedt
ouset he
nat
ionwi
detelecommuni cat
ionssystem oftheFacili
ti
esFreeofCharge.

1.
8. Uponexpirati
onofthel
ease,
theFaci
l
iti
esshal
lbeownedbyPCSOwi
thoutany
addi
ti
onal
consi
derat
ion.
 3

xxx xxx xxx

2.
2. OBJECTI
VES

Theobj
ect
ivesofPCSOi
nleasi
ngt
heFaci
l
iti
esf
rom apr
ivat
eent
it
yar
easf
oll
ows:

xxx xxx xxx

2.
2.2.Enabl
ePCSOt
ooper
ateanat
ionwi
deon-
li
neLot
ter
ysy
stem atnoexpenseorr
iskt
othe
gover
nment.

xxx xxx xxx

2.
4. DUTI
ESANDRESPONSI
BILI
TIESOFTHELESSOR

xxx xxx xxx

2.
4.2.THELESSOR

TheProponentisexpectedtofur
nishandmai ntai
ntheFaci
l
iti
es,i
ncludingt
hepersonnelneeded
tooper
atet hecomputers,t
hecommuni cat
ionsnetworkandsalesoffi
cesunderabuil
d-l
ease
basi
s.Thepr i
nti
ngofticket
sshall
beundertakenunderthesupervi
sionandcontr
olofPCSO.
TheFacil
i
tiesshall
enablePCSOt ocomputerizetheent
ir
egami ngsystem.

ThePr oponenti
sexpectedtof or
mulateanddesi gnconsumer -or
ient
edMast erGamesPl an
suit
edt othemarketpl
ace,especi
all
ygearedt oFili
pinogaminghabi t
sandpr ef
erences.I
n
additi
on,theMasterGamesPl anisexpectedtoincludeaPr oductPlanforeachgameand
explai
nhoweachwi l
lbeintr
oducedi nt
othemar ket.Thiswill
beani ntegr
alpartofthe
DevelopmentPlanwhichPCSOwi ll
requir
efrom thePr oponent.

xxx xxx xxx

ThePr
oponenti
sexpectedt
opr
ovi
deupgr
adest
omoder
nizet
heent
ir
egami
ngsy
stem ov
ert
he
l
if
eoft
heleasecont
ract
.

ThePr
oponenti
sexpect
edt
opr
ovi
det
echnol
ogyt
ransf
ert
oPCSOt
echni
cal
per
sonnel

4

7. GENERALGUI
DELI
NESFORPROPONENTS
xxx xxx xxx

Final
l
y,t
heProponentmustbeabletostandtheaci
dtestofprov
ingthatitisanent
it
yablet
o
takeonther
oleofresponsi
blemai
ntai
neroftheon-
li
nelott
erysyst
em, andabletoachi
eve
PSCO'sgoal
offormalizi
nganon-
li
nelotter
ysyst
em toachi
eveit
smandat edobj
ect
ive.
 5

xxx xxx xxx

16. DEFI
NITI
ONOFTERMS

Facil
i
ties:All
capit
al equipment,comput ers,t
erminal
s,software,nati
onwidetelecommunicat
ion
network,ti
cketsalesoffices,
furnishi
ngs,andf i
xtur
es;pri
ntingcosts;costofsalari
esandwages;
advert
isingandpromot ionexpenses; maintenancecosts;expansionandr epl
acementcosts;
securi
tyandi nsur
ance, andallotherrel
atedexpensesneededt ooperatenati
onwi deon-
li
ne
l
otter
ysy stem.6

Consideri
ngtheabovecit
izenshi
pr equirement
,thePGMCclai
msthatt
heBerj
ayaGr
oup
"
under t
ooktoreduceit
sequitystakesi nPGMCt o40%,
"bysel
li
ng35%outoft
heori
ginal
75%
for
eignstockhol
dingst
olocal i
nvestors.

On15August1993,
PGMCsubmi
tt
edi
tsbi
dtot
hePCSO.
7

Thebi dswer eevaluat


edbytheSpeci
alPre-
Quali
ficati
onBidsandAwardsCommi ttee(SPBAC)
fortheon-linelot
teryandi
tsBi
dReportwast her
eaftersubmit
tedt
otheOfficeofthe
President.
 8 Thesubmissi
onwasprecededbycompl aintsbyt
heCommi t
tee'sChairper
son,Dr
.
MitaPar dodeTav era. 
9

On21Oct ober1993, t
heOf fi
ceofthePresidentannouncedthatithadgiv
ent herespondent
PGMCt hego-signaltooperatethecountry'
son-l
inelotter
ysyst
em andthatthecor responding
i
mpl ementi
ngcont r
actwoul dbesubmittednotlaterthan8November1993" forfinalclearance
andapprovalbytheChi efExecuti
ve.

10 Thisannouncementwaspubl i
shedint heMani la
Standar
d,Phil
ippi
neDai lyI
nquirer
,andtheMani l
aTimeson29Oct ober1993. 11

On4Nov ember1993, KI
LOSBAYANsentanopenlet
tert
oPresi
dent
ial
FidelV.Ramosstr
ongly
opposi
ngtheset t
inguptot
heon-l
i
nelot
ter
ysystem ont
hebasi
sofseriousmoralandet
hical
consi
derat
ions.
 12

Atthemeeti
ngoftheCommi tt
eeonGamesandAmusement
softheSenat
eon12Nov ember
1993,KI
LOSBAYANr ei
ter
atedit
svi
gor
ousopposi
ti
ont
otheon-
li
nel
ott
eryonaccountofi
ts
i
mmor al
it
yandil
l
egalit
y. 
13

On19Nov ember1993,themediareportedthatdespi
tetheoppositi
on,"
Malacañangwi l
lpush
t
hroughwi t
htheoper
ationofanon-l
inelott
erysystem nat
ionwide"andt
hatitisactual
lyt
he
r
espondentPCSOwhi chwil
loperat
et helot
terywhil
ethewi nni
ngcorpor
atebiddersaremerel
y
"
lessor
s."
 14
On1December1993, KILOSBAYANr equestedcopiesofal ldocumentsper t
aini
ngtothelott
ery
awardfrom Executiv
eSecr etar
yTeof i
stoGuingona,Jr.Inhi
sanswerof17December1993, the
Execut
iveSecretaryinf
or medKILOSBAYANt hattherequesteddocument swouldbedul y
tr
ansmittedbeforetheendoft hemont h. 
15.Howev er,onthatsamedat e,anagreement
denominatedas" ContractofLease"wasf i
nall
yexecutedbyr espondentPCSOandr espondent
PGMC.  
16 ThePresident,perthepressstatementissuedbyt heOffi
ceoft hePresi
dent,
approv
edi ton20December1993. 17

I
nv i
ewoftheirmater
ial
i
tyandr
elev
ance,
wequot
ethef
oll
owi
ngsal
i
entpr
ovi
sionsoft
he
Contr
actofLease:

1. DEFI
NITI
ONS

Thef
oll
owi
ngwor
dsandt
ermsshal
lhav
ethef
oll
owi
ngr
espect
ivemeani
ngs:

1.1 Rent alFee—Amountt obepai


dbyPCSOt otheLESSORascompensationf
ort
he
ful
fi
ll
mentoft
heobl igat
ionsoft
heLESSORunderthi
sContract
,incl
udi
ng,
butnotl
i
mit
edtot
he
l
easeoftheFacil
i
ties.

xxx xxx xxx

1.3 Faci l
it
ies—Al lcapi
talequipment,computer
s,terminals,
soft
ware( i
ncludi
ngsour ce
codesfortheOn-LineLotteryappli
cationsoft
warefortheterminal
s,t
elecommuni cat
ionsand
centr
alsystems),
technology ,
int
ellect
ualproper
tyr
ights,t
elecommunicationsnetwork,and
fur
nishi
ngsandf i
xtures.

1.4 Mai ntenanceandOt herCost s—Al lcostsandexpensesrelati


ngt opr
int
ing,manpower,
sal
ar i
esandwages, advert
isi
ngandpr omot ion,maint
enance,expansionandreplacement,
securit
yandi nsurance,andallotherrelatedexpensesneededt ooper ateanOn-LineLot
ter
y
System, whichshallbefortheaccountoft heLESSOR.Al lexpensesrelati
ngtothesetti
ng-
up,
operati
onandmai ntenanceofticketsal esof f
icesofdeal
ersandr et
ailer
sshall
bebor neby
PCSO' sdealersandr et
ail
ers.

1.
5 Dev elopmentPlan—Thedet ai
l
edplanofallgames,themarket
ingthereof,numberof
pl
ayers,
v al
ueofwinningsandthelogi
sti
csrequi
redtoint
roducet
hegames, includingt
he
MasterGamesPl anasapprovedbyPCSO, att
achedheret
oasAnnex" A",
modi fi
edasnecessary
bytheprovi
sionsofthisCont
ract
.

xxx xxx xxx

1.8 Escr owDeposi t—Theproposaldeposi


tint
hesum ofThr
eeHundredMil
l
ionPesos
(P300,
000,000.00)submit
tedbytheLESSORt oPCSOpur
suanttother
equir
ementsoft
he
RequestforProposals.

2. SUBJECTMATTEROFTHELEASE
TheLESSORshallbui
l
d,fur
nishandmaintai
natit
sownexpenseandriskt
heFacil
i
tiesf
orthe
On-
LineLot
terySy
stem ofPCSOintheTerri
tor
yonanexcl
usi
vebasis.TheLESSORshallbear
al
lMaint
enanceandOtherCostsasdef
inedherei
n.

xxx xxx xxx

3. RENTALFEE

Forandinconsiderat
ionoftheperformancebyt heLESSORofi t
sobli
gati
onsherei
n,PCSOshal
l
payLESSORaf ixedRentalFeeequal t
of ourpoi
ntninepercent(
4.9%)ofgr
ossrecei
ptsfr
om
ti
cketsal
es,payablenetoftaxesrequir
edbyl awtobewi t
hheld,
onasemi -
monthlybasi
s.
Goodwill
,fr
anchiseandsimilarf
eesshall bel
ongtoPCSO.

4. LEASEPERI
OD

Theperi
odofthel
easeshallcommenceninety(90)daysfr
om thedateofeff
ecti
vit
yofthi
s
Contr
actandshal
lrunf
oraper i
odofei
ght(8)yearsther
eaft
er,
unlesssoonerter
minat
edin
accor
dancewitht
hisCont
ract.

5. RIGHTSANDOBLI
GATI
ONSOFPCSOASOPERATOROFTHEON-
LINELOTTERY
SYSTEM

PCSOshal
lbet
hesol
eandi
ndi
vi
dual
oper
atoroft
heOn-
LineLot
ter
ySy
stem.Consequent
ly:

5.1 PCSOshal l
havesoler
esponsi
bil
i
tytodecidewhethert
oimplement,f
ull
yorpart
ial
l
y,t
he
Mast erGamesPl anoftheLESSOR.PCSOshallhavethesoler
esponsi
bil
it
ytodeter
minethe
ti
mef ori
ntr
oducingnewgamest othemarket
.TheMast erGamesPlanincl
udedinAnnex"A"
hereofisherebyapprovedbyPCSO.

5.2 PCSOshal lhav econtr


olov
errevenuesandrecei
ptsofwhat
evernatur
efrom theOn-
Line
Lotter
ySystem.Afterpay ingt
heRentalFeetotheLESSOR,PCSOshal
lhaveexclusi
ve
responsi
bil
i
tytodet erminetheRevenueAll
ocati
onPlan;Pr
ovi
ded,t
hatthesameshall be
consist
entwithther equir
ementofR.A.No.1169,asamended,whi
chfi
xesapr i
zefundoffi
ft
y
fi
vepercent(55%)ont heaver
age.

5.3 PCSOshal lhaveexcl


usiv
econtrol
overt
hepr
int
ingoft
icket
s,i
ncl
udi
ngbutnotl
i
mit
edt
o
thedesi
gn,
text
,andcontent
sthereof
.

5.4 PCSOshal lhavesoleresponsibi


li
tyov ert
heappoi ntmentofdealer
sorretai
lers
throughoutthecountr
y.PCSOshal lappointthedealersandr etai
ler
sinati
melymannerwi t
hdue
regardtotheimplementat
iontimetableoft heOn-Li
neLot terySystem.Nothi
nghereinshal
l
precludetheLESSORf r
om recommendi ngdeal ersorretai
lersforappoi
ntmentbyPCSO, which
shallactonsaidrecommendat i
onwi t
hinf ort
y-ei
ght(48)hour s.

5.
5 PCSOshal ldesi
gnatethenecessaryper
sonnelt
omoni t
orandaudi
tthedail
y
per
for
manceoft
heOn-LineLott
erySystem.Forthi
spurpose,
PCSOdesigneesshal
lbegi
ven,
f
reeofcharge,
sui
tabl
eandadequatespace,f
urni
tur
eandfi
xtures,
inall
offi
cesoftheLESSOR,
i
ncl
udingbutnotl
i
mi t
edtoi
tsheadquart
ers,
alter
nat
esit
e,r
egionalandareaoff
ices.

5.
6 PCSOshallhavet
her esponsi
bil
i
tytoresol
ve,
andexclusi
vejur
isdi
cti
onover,
all
mat t
ers
i
nvol
vi
ngt
heoper
ati
onoftheOn-LineLott
erySyst
em notot
herwisepr
ov i
dedint
hisContr
act.

5.7 PCSOshal lpromul


gat
epr
ocedur
alandcoor
dinat
ingr
ulesgov
erni
ngal
lact
ivi
ti
esr
elat
ing
totheOn-
LineLot
ter
ySyst
em.

5.
8 PCSOwi llberesponsi
blef
ort hepaymentofpri
zemonies,commissi
onstoagent
sand
deal
ers,
andt
axesandl evi
es(i
fany)chargeabl
etotheoper
atoroftheOn-Li
neLott
erySy
stem.
TheLESSORwil
lbearallot
herMaintenanceandOtherCost
s,exceptasprov
idedi
nSecti
on1.4.

5.
9 PCSOshal
lassi
stt
heLESSORi
nthef
oll
owi
ng:

5.
9.1 Wor
kper
mit
sfort
heLESSOR'
sst
aff
;

5.
9.2 Appr
oval
sfori
mpor
tat
ionoft
heFaci
l
iti
es;

5.
9.3 Appr
oval
sandconsent
sfort
heOn-
LineLot
ter
ySy
stem;
and

5.9.4 Busi
nessandpremisesli
censesf
oral
lof
fi
cesoft
heLESSORandl
i
censesf
ort
he
telecommuni
cati
onsnetwork.

5.10 I ntheev entt


hatPCSOshal l
pre-termi
natethisCont r
actorsuspendt heoperat
ionofthe
On- Li
neLot ter
ySy st
em, inbreachoft hisContractandt hr
oughnof aultoftheLESSOR, PCSO
shal l
prompt ly
,andinanyev entnotlaterthansixt
y( 60)days,r
eimbursetheLESSORt heamount
ofitstotalinvestmentcostassoci atedwi t
htheOn- LineLott
erySystem, i
ncludi
ngbutnotlimi
ted
tothecostoft heFacil
i
ties,andf urt
hercompensat et heLESSORf orlossofexpectednetprofi
t
aftertax,comput edovert heunexpiredterm ofthelease.

6. DUTI
ESANDRESPONSI
BILI
TIESOFTHELESSOR

TheLESSORi soneofnotmor ethanthree( 3)lessorsofsimilarf


acil
i
tiesforthenati
onwideOn-
Li
neLot t
erySystem ofPCSO.Itisunderstoodt hattheri
ghtsoft heLESSORar epri
maril
ythose
ofalessoroftheFacil
it
ies,andconsequent ly
,allri
ghtsinv
ol v
ingthebusinessaspectsofthe
useoftheFacili
ti
esarewi t
hinthejur
isdi
ctionofPCSO.Dur ingtheterm ofthelease,t
heLESSOR
shal
l
.

6.
1 Mai nt
ainandpreserv
eitscor
porateexi
stence,
right
sandpr
ivi
l
eges,
andconducti
ts
busi
nessi
nanorderl
y,eff
ici
ent
,andcustomarymanner.

6.
2 Mai
ntai
ninsur
ancecov
eragewi
thi
nsur
ersaccept
abl
etoPCSOonal
lFaci
l
iti
es.

6.3 Compl ywi


thal
llaws,stat
ues,r
ulesandr
egul
ati
ons,
order
sanddi
rect
ives,
obl
i
gat
ions
anddut
iesbywhi
chi
tislegal
lybound.
6.4 Dul ypayanddischar
geallt
axes,assessment
sandgover
nmentchar
gesnowand
hereaf
teri
mposedofwhatevernat
urethatmaybelegal
l
ylev
ieduponi
t.

6.5 Keepal lt
heFaci
li
ti
esinfail
safeconditi
onand,ifnecessary,upgrade,r
epl
aceand
i
mprovet heFaci
li
ti
esfr
om ti
met oti
measnewt echnologydev el
ops,inordertomaketheOn-
Li
neLotterySyst
em morecost-
eff
ecti
veand/ orcompetit
ive,andasmayber equi
redbyPCSO
shal
lnotimposesuchrequi
rementsunreasonablynorarbi
trari
ly.

6.
6 Pr ovi
dePCSOwi
thmanagementt
ermi
nal
swhi
chwi
l
lal
l
owr
eal
-t
imemoni
tor
ingoft
he
On-
LineLot
ter
ySyst
em.

6.
7 Uponef fecti
vi
tyofthi
sContract,commencethetrai
ningofPCSOandot herl
ocal
per
sonnelandthetr
ansferoft
echnologyandexpert
ise,
sucht hatatt
heendoft het
erm oft
his
Contr
act
,PCSOwi l
lbeabletoeff
ectivel
ytake-
overt
heFacil
iti
esandef f
ici
ent
lyoper
atetheOn-
Li
neLott
erySystem.

6.
8 Under t
akeapositi
veadv
ert
isi
ngandpr
omotionscampaignforbothinst
it
uti
onal
and
pr
oductl
i
neswit
houtengagi
ngi
nnegati
veadv
ert
isi
ngagainstot
herlessor
s.

6.
9 Bearal lexpensesandri
sksr
elat
ingt
otheFaci
l
iti
esi
ncl
udi
ng,
butnotl
i
mit
edt
o,
Maint
enanceandOt herCost
sand:

xxx xxx xxx

6.
10 Bearall
risksi
fther
evenuesf
rom t
icketsal
es,
onanannual
i
zedbasi
s,ar
einsuf
fi
cientt
o
payt
heent
ir
eprizemoney.

6.11 Be, andishereby, author


izedt
ocol
lectandret
ainf
orit
sownaccount ,asecuri
tydeposi
t
fr
om dealer
sandr et
ailers,inanamountdeter
minedwitht
heapprovalofPCSO,inrespectof
equipmentsuppli
edbyt heLESSOR.PCSO' sapprov
alshal
lnotbeunreasonabl
ywithheld.

xxx xxx xxx

6.
12 Compl
ywi
thpr
ocedur
alandcoor
dinat
ingr
ulesi
ssuedbyPCSO.

7. REPRESENTATI
ONSANDWARRANTI
ES

TheLESSORr
epr
esent
sandwar
rant
sthat
:

7.1 TheLESSORi scor por


ati
ondulyorganizedandexi sti
ngundert helawsoft
heRepubli
cof
thePhi l
i
ppines,atleastsixtyper
cent(60%)oft heout st
andingcapitalst
ockofwhichisowned
byFili
pinoshareholder s.Theminimum requi
redFi l
ipi
noequi t
ypartici
pati
onshal
lnotbe
i
mpai redthroughv oluntaryori
nvolunt
arytr
ansf er
,disposit
ion,orsaleofshar
esofstockbythe
presentstockholders.

7.
2 TheLESSORandi tsAf
fil
iat
eshavet
heful
lcorpor
ateandlegal
powerandauthor
it
yto
ownandoper
atet
hei
rpr
opert
iesandtocar
ryont
heirbusi
nessintheplacewher
esuch
pr
oper
ti
esar
enowormaybeconduct
ed....

7.3 TheLESSORhasorhasaccesstoal
lthefi
nancingandf
undi
ngr
equi
rement
stopr
ompt
ly
andef
fect
ivel
ycar
ryoutt
het
ermsoft
hisContr
act....

7.4 TheLESSORhasorhasaccesstoal
lthemanager
ial
andt
echni
cal
exper
ti
set
opr
ompt
ly
andef
fect
ivel
ycar
ryoutt
het
ermsoft
hisContr
act
....

xxx xxx xxx

10. TELECOMMUNI
CATI
ONSNETWORK

TheLESSORshal lest
abli
shat el
ecommunicati
onsnetworkthatwil
lconnectal
lmuni
cipal
i
ties
andciti
esintheTerri
tor
yinaccordancewit
h,attheLESSOR'
sopt i
on,ei
theroft
heLESSOR's
proposal
s(oracombi nat
ionsofbothsuchproposal
s)att
achedheretoasAnnex"B,
"andunder
thefol
lowi
ngPCSOschedul e:

xxx xxx xxx

PCSOmay ,atitsopt ion,requi retheLESSORt oest abli


shthet elecommuni cat ionsnet workin
accor dancewi tht heabov eTi met ablei nprovinceswher etheLESSORhasnoty etinstal
led
terminals.Pr ovided,thatsuchpr ov i
nceshav eexistingnodes.Onceamuni cipal i
tyorci t
yis
servi
cedbyl andl inesofal i
censedpubl i
ctelephonecompany ,
andsuchl i
nesar econnect edto
Met r
oMani la,thent heobl i
gat ionoft heLESSORt oconnectsuchmuni cipali
tyorci tythrougha
tel
ecommuni cationsnet wor kshal lceasewi t
hrespectt osuchmuni cipalit
yorci ty.Thev oice
facil
i
tywi llcovert hef ourofficesoft heOf ficeoft hePr esi
dent ,National Disast erCont rol
Coordi nati
ngCounci l,Phil
ippineNat i
onal Policeandt heNat i
onal BureauofI nv estigati
on,and
eachci tyandmuni ci
palit
yi nt heTer r
itoryexceptMet roMani la,andt hoseci tiesand
muni cipal
iti
eswhi chhav eeasyt el ephoneaccessf rom thesef ourof f
ices.Voi cecal lsfr
om t he
fourof fi
cesshal l
bet ransmi ttedv iar adioorVSATt otheremot emuni cipalit
ieswhi chwillbe
connect edt othisv oicefacili
tyt hroughwi rednet wor korbyr adio.Thef acil
it
yshal l bedesigned
tohandl efourpr i
vateconv ersat i
onsatanyonet ime.

xxx xxx xxx

13. STOCKDI
SPERSALPLAN

Withi
nt wo(2)yearsfr
om theeff
ect
ivi
tyofthi
sContr
act
,theLESSORshall
causei
tsel
ftobe
l
ist
edi nthelocalst
ockexchangeandofferatl
eastt
went
yfiveper
cent(
25%)ofit
sequit
ytothe
publi
c.

14. NON-
COMPETI
TION

TheLESSORshal lnot,
dir
ectl
yori
ndi
rect
ly,under
takeanyact
ivi
tyorbusi
nessi
ncompet i
ti
on
wit
horadv er
setot heOn-Li
neLot
ter
ySystem ofPCSOunlessitobt
ainst
helat
ter
'spr
iorwrit
ten
consentt
hereto.
15. HOLDHARMLESSCLAUSE

15.1 TheLESSORshal latalltimesprotectanddefend,atit


scostandexpense, PCSOf r
om
andagainstanyandal l
li
abil
i
tiesandcl ai
msf ordamagesand/ orsui
tsf
ororbyr easonofany
deathsof,oranyinj
uryorinj
uriestoanyper sonorpersons,ordamagest opropert
yofanykind
whatsoever,causedbytheLESSOR, i
tssubcontr
actors,i
tsauthori
zedagentsorempl oy
ees,
fr
om anycauseorcauseswhat soev
er.

15.
2 TheLESSORher ebycovenantsandagreestoindemnif
yandhol dPCSOharmlessfr
om
al
ll
iabil
it
ies,
char
ges,expenses(incl
udi
ngreasonablecounselfees)andcost
sonaccountofor
byreasonofanysuchdeathordeat hs,
inj
uryorinj
uri
es,l
iabi
l
iti
es,clai
ms,sui
tsorl
ossescaused
bytheLESSOR'sfaul
tornegli
gence.

15.3 TheLESSORshal latal


lti
mesprot
ectanddefend,atit
sowncostandexpense,
itst
it
let
o
thefaci
l
iti
esandPCSO' sint
erestt
her
einf
rom andagainstanyandal
lcl
aimsfort
hedurati
onof
theContractunt
ilt
ransf
ertoPCSOofownershipoftheservi
ceabl
eFaci
li
ti
es.

16. SECURI
TY

16.1 Toensurefait
hful
compli
ancebyt
heLESSORwit
htheter
msoftheContr
act,
the
LESSORshal
lsecureaPerfor
manceBondfr
om ar
eput
ablei
nsur
ancecompanyorcompani
es
accept
abl
etoPCSO.

16.2 ThePer formanceBondshal lbeintheinit


ialamountofThreeHundr edMil
li
onPesos
(P300,000,000.
00),toit
sU.S.doll
arequival
ent,andshallberenewedtocov ert
hedurationofthe
Contract.Howev er
,thePerf
ormanceBondshal lbereducedproport
ionat
elytothepercentageof
unencumber edt er
minalsi
nstal
led;Provi
ded,t
hatt hePerfor
manceBondshal li
nnocasebel ess
thanOneHundr edFift
yMill
ionPesos( P150,
000,000.00)
.

16.
3 TheLESSORmayati
tsopt
ionmai
ntai
nit
sEscr
owDeposi
tast
hePer
for
manceBond....

17. PENALTI
ES

17.
1 Exceptasmaybepr ovi
dedinSect i
on17. 2,shouldtheLESSORf ai
ltotakeremedial
measur eswit
hinseven(7)days,andrecti
fyt hebreachwi t
hint
hir
ty(30)days,from wri
tt
en
not
icebyPCSOofanywi l
ful
lorgrosslynegli
gentv i
olat
ionofthemat er
ialt
ermsandcondi t
ions
ofthisContr
act,al
lunencumberedFaci l
it
iesshallautomatical
l
ybecomet hepr opert
yofPCSO
wit
houtconsiderat
ionandwi t
houtneedf orfurthernoti
ceordemandbyPCSO.The
PerformanceBondshallli
kewisebef orf
eitedinfavorofPCSO.

17.2 Shoul dt heLESSORfai


ltocompl ywiththet
ermsoft heTimetabl
espr ov
idedinSect i
on9
and10, itshallbesubjectt
oanini
tialPenal
tyofTwentyThousandPesos( P20,
000.00),percit
y
ormuni cipali
typereverymont
hofdel ay;Pr
ov i
ded,t
hatthePenaltyshal
li
ncrease,everyninety
(90)days, bytheamountofTwent yThousandPesos( P20,000.
00)percit
yormuni cipal
ityper
mont h,
whi lstshallf
ail
uret
ocompl ypersi
sts.Thepenal
tyshallbedeductedbyPCSOf rom the
r
ent
alf
ee.

xxx xxx xxx

20. OWNERSHI
POFTHEFACI
LITI
ES

Afterexpi
rat
ionoftheterm ofthel
easeasprovidedinSecti
on4,theFacil
it
iesdirectl
yrequi
red
fortheOn-Li
neLotter
ySy st
em mentionedinSecti
on1.3shallaut
omaticall
ybelongi nful
l
owner shi
ptoPCSOwi thoutanyfur
therconsi
derati
onotherthant
heRent alFeesalreadypai
d
duringtheef
fect
ivi
tyofthelease.

21. TERMI
NATI
ONOFTHELEASE

PCSOmayter
minat
et hi
sCont
ractf
oranybr
eachoft
hemat
eri
alpr
ovi
sionsoft
hisCont
ract
,
i
ncl
udi
ngt
hefoll
owing:

21.1 TheLESSORi sinsol


ventorbankr
uptorunabl
etopayitsdebts,
stopsorsuspendsor
threat
enstost
oporsuspendpay mentofal
loramateri
alpar
tofitsdebts,orpr
oposesor
makesageneralassi
gnmentoranar r
angementorcomposit
ionswithorforthebenef
itofi
ts
credit
ors;
or

21.2 Anor derismadeoranef f


ecti
veresol
uti
onpassedf
ort
hewindi
ngupordissol
uti
onof
theLESSORorwheni tceasesorthr
eatenstoceaset
ocarr
yonal
loramater
ial
partofit
s
operat
ionsorbusiness;or

21.3 Anymateri
alstat
ement,repr
esentat
ionorwar
rant
ymadeorf
urni
shedbyt
heLESSOR
provedt
obemateri
all
yfalseormisl
eading;

saidter
minati
ontotakeeff
ectuponreceiptofwri
tt
ennoti
ceofter
minat
ionbyt
heLESSORand
fai
lur
etotakeremedialact
ionwi
thi
nsev en(7)daysandcureorr
emedythesamewit
hint
hir
ty
(30)daysfr
om noti
ce.

Anysuspensi
on,
cancel
l
ati
onortermi
nati
onoft hi
sCont
ractshal
lnotr
eli
evet
heLESSORofany
l
iabi
l
ityt
hatmayhaveal
readyaccr
uedhereunder.

xxx     xxx     xxx

Consider
ingthedeni
albytheOf fi
ceoft hePr
esidentofitsprotestandthestatementof
Assist
antExecut
iveSecr
etaryRenatoCor onathat"onlyacourti nj
unct
ioncanstop
Malacañang,
"andtheimmi nentimplementat
ionoft heContractofLeasei nFebr
uary1994,
KI
LOSBAYAN, wi
thit
sco-petit
ioner
s, f
il
edon28Januar y1994t hispet
it
ion.

I
nsuppor
toft
hepet
it
ion,
thepet
it
ioner
scl
aimt
hat
:

...XXTHEOFFICEOFTHEPRESI DENT,ACTI
NGTHROUGHRESPONDENTSEXECUTI
VE
SECRETARYAND/ORASSISTANTEXECUTIVESECRETARYFORLEGALAFFAI
RS,
ANDTHE
PCSOGRAVELYABUSE[D]THEIRDISCRETI
ONAND/ORFUNCTIONSTANTAMOUNTTOLACK
OFJURISDI
CTI
ONAND/ORAUTHORI TYINRESPECTI
VELY:(
A)APPROVINGTHEAWARDOF
THECONTRACTTO,AND(B)ENTERINGINTOTHESO-CALLED"CONTRACTOFLEASE"WI
TH,
RESPONDENTPGMCFORTHEI NSTALLATI
ON,ESTABLI
SHMENTANDOPERATI ONOFTHEON
-
LINELOTTERYANDTELECOMMUNICATIONSYSTEMSREQUI REDAND/ORAUTHORI
ZED
UNDERTHESAIDCONTRACT,CONSIDERI
NGTHAT:

a) UnderSecti
on1oftheChart
erofthePCSO,thePCSOi
sprohibi
tedf
rom hol
dingand
conducti
nglot
teri
es"i
ncol
l
aborati
on,associ
ati
onorjoi
ntv
ent
urewit
hanyperson,associ
ati
on,
companyorentit
y";

b) UnderActNo.3846andest
abl
ishedjur
ispr
udence,aCongressi
onal
franchisei
srequi
red
bef
oreanyper
sonmaybeal l
owedtoestabl
i
shandoper at
esaidtel
ecommuni cat
ionssyst
em;

c) UnderSection11, Ar
ti
cleXI
IoftheConst
it
uti
on,al
essthan60%Fil
i
pino-
ownedand/or
cont
rol
l
edcorporation,li
ket
hePGMC, i
sdisqual
if
iedfr
om oper
ati
ngapubl
icser
vice,
li
ket
he
sai
dtel
ecommuni cationssy
stem;and

d) RespondentPGMCi snotaut
hor
izedbyit
schar
terandundert
heForei
gnI
nvest
mentAct
(R.
A.No.7042)t
oinst
all
,establ
i
shandoperat
etheon-
li
nelot
toandtel
ecommuni
cati
ons
syst
ems.18

Petit
ionerssubmitt hatt hePCSOcannotv ali
dl yent eri
nt otheassailedCont ractofLeasewi th
thePGMCbecausei tisanar rangementwher eint hePCSOwoul dhol dandconductt heon-li
ne
l
ot t
erysystem in"collabor at
ion"or" association"wi ththePGMC, i
nv iolati
onofSect i
on1( B)of
R.A.No.1169, asamendedbyB. P.Bl g.42,whi chpr ohibitsthePCSOf rom holdingand
conductingcharit
ysweepst akesr aces, l
otter
ies, andot hersi mil
aract i
viti
es"incol l
aborati
on,
associati
onorjointv ent urewi t
hanyper son,associ ati
on, companyorent it
y,foreignor
domest i
c."Evengr anting arguendo  thataleaseoff aci
liti
esisnotwi thinthecont empl at
ionof
"coll
aborati
on"or"associ ati
on,"ananal ysi
s,howev er,oftheCont ractofLeasecl earl
yshows
thatthereisa"collabor ati
on, associat i
on,orjointv enturebet weenrespondent sPCSOand
PGMCi ntheholdingoft heOn- LineLot t
erySy stem, "andt hattherearet ermsandcondi ti
onsof
theCont r
act"showi ngt hatrespondentPGMCi stheact uallott
ooper atorandnotr espondent
PCSO. "
19

Thepet iti
oner sal sopoi ntoutt hatparagraph10oft heCont ractofLeaser equir
esoraut hori
zes
PGMCt oest abl ishat elecommuni cati
onsnet workthatwi l
lconnectal lthemuni ci
pali
ti
esand
cit
iesint het errit
or y
.Howev er, PGMCcannotdot hatbecausei thasnof ranchi
sef r
om Congr ess
toconst ruct,inst al
l,establi
sh, oroperatethenetworkpur suantt oSection1ofActNo.3846, as
amended.Mor eov er,PGMCi sa75%f orei
gn-ownedorcont roll
edcor porati
onandcannot ,
therefore,begr antedaf r
anchi seforthatpurposebecauseofSect i
on11, Arti
cleXIIofthe1987
Const i
tution.Fur thermor e,since" t
hesubscr i
bedforeigncapi t
al"ofthePGMC" comest oabout
75%, asshownbypar agraphEI GHTofitsArticl
esofI ncorporati
on,"itcannotlawfull
yent eri
nto
thecont ractinquest ionbecauseal lf
ormsofgambl ing—andl otteryisoneoft hem —ar e
i
ncludedi nt heso- call
edf oreigninvestmentsnegativelistundert heForeignInvestmentsAct
(
R.A.No.7042)wher
eonl
yupt
o40%f
orei
gncapi
tal
isal
l
owed.
 20

Final
ly
, t
hepet
it
ioner
sinsi
stt
hattheArti
clesofIncor
por
ati
onofPGMCdonotaut
hor
izei
tto
establ
ishandoperat
eanon-l
i
nelotter
yandt el
ecommunicat
ionssy
stems.
21

Accordingl
y, t
hepet i
ti
onerspraythatwei ssueat emporaryr
estrai
ningorderandawr i
tof
preli
minaryinjunct
ioncommandi ngt herespondent soranypersonactingi
nt hei
rplacesorupon
theiri
nstr
uct i
onstoceaseanddesi stfrom implement i
ngthechall
engedCont ractofLeaseand,
afterheari
ngt hemer i
tsofthepetit
ion,t
hatwer enderj
udgmentdecl ar
ingtheContractofLease
voidandwi thouteffectandmakingt heinjunct
ionpermanent. 
22

Wer
equi
redt
her
espondent
stocommentont
hepet
it
ion.

InitsCommentf iledon1Mar ch1994, pr ivaterespondentPGMCasser tsthat"(


1)[it
]ismer el
y
ani ndependentcont r
actorforapi eceofwor k,(i
.e.,t
hebui l
dingandmai nt
enanceofal ottery
syst em t
obeusedbyPCSOi ntheoper at ionofi tslott
er yf
ranchi se);and( 2)assuch
i
ndependentcont ractor,PGMCi snotaco- operatoroft helotteryf ranchi sewi t
hPCSO, noris
PCSOshar i
ngi t
sf ranchise,'
incol l
aboration, associationorj ointv entur e'withPGMC—assuch
statutoryl
imi tati
oni sv i
ewedf rom thecont ext ,
intent,andspi ri
tofRepubl icAct1169, as
amendedbyBat asPambansa42. "I
tfurthercl aimst hatasani ndependentcont r
actorfora
pieceofwor k,itisnei t
herengagedi n"gambl ing"norin" publicser v i
ce"r el
ativetothe
telecommuni cationsnet work,whi chthepet i
tionersev enconsi derasan" i
ndispensable
requi r
ement "ofanon- li
nelotterysystem.Fi nally
, i
tstatesthatt heexecut i
onand
i
mpl ementationoft hecontractdoesnotv i
olatetheConst ituti
onandt helaws; t
hatthei ssueon
the" morali
ty"oft hel ott
eryfranchisegr ant edt othePCSOi spol iti
cal andnotj udici
alorlegal ,
whi chshouldbev entil
atedinanot herfor um; andt hatt he"petitionersdonotappeart ohav ethe
l
egal st
andingorr eal i
nter
estint hesubj ectcont ractandi nobt ainingt her el
ief
ssought ." 
23

Inthei rCommentf il
edbyt heOf ficeoft heSol i
citorGener al,publicrespondent sExecut i
ve
Secr etaryTeof istoGui ngona, Jr.,Assist antExecut iveSecr etaryRenat oCor ona, andthePCSO
mai ntaint hatthecont r
actofl easei nquest iondoesnotv i
olat eSect i
on1ofR. A.No.1169, as
amendedbyB. P.Bl g.42, andt hatthepet it
ioner '
sinterpretationoft hephr ase" i
ncol l
aboration,
associ ationorj ointventur e"i
nSect i
on1i s" mucht oonar row, str
ainedandut terlydevoidof
l
ogi c"f orit"i
gnor esther ealit
yt hatPCSO, asacor porateent ity,
isvest edwi t
ht hebasicand
essent ialprerogat i
vet oent erintoall kindsoft ransact i
onsorcont r
actsasmaybenecessar yf or
theat tainmentofi tspurposesandobj ect i
ves."Whatt hePCSOchar ter"seekst opr ohi
bitist hat
arrangementaki ntoa" j
ointv enture"orpar tnershipwher et hereis" communi tyofinterestint he
busi ness, shari
ngofpr ofitsandl osses, andamut ualri
ghtofcont rol,
"achar act eri
sti
cwhi ch
doesnotobt aini nacont ractofl ease. "Wi t
hr espectt othechal l
engedCont ractofLease, t
he
"rol
eofPGMCi slimitedtot hatofal essoroft hef acil
it
ies"fort heon- l
inelotterysy st
em; in
"stri
ctt echnical andlegal sense, "saidcont r
act" canbecat egor i
zedasacont ractforapi eceof
wor kasdef inedi nArticl
es1467, 1713and1644oft heCi vilCode."

Theyf
urt
hercl
aimt
hatt
heest
abl
i
shmentoft
het
elecommuni
cat
ionssy
stem st
ipul
atedi
nthe
ContractofLeasedoesnotr equireacongr essi
onalf
ranchisebecausePGMCwi ll
notoper atea
publi
cut i
l
ity
;mor eover,PGMC's"establi
shmentofat el
ecommuni cationssystem isnotintended
toestabli
shat elecommunicat
ionsbusi ness,
"andithasbeenhel dt hatwher ethefaci
li
tiesar e
operated"notforbusinesspur
posesbutf orit
sownuse,"al egi
slati
v efranchi
seisnotr equired
beforeacertif
icateofpubli
cconv eni
encecanbegr anted.
 24 Evengr anting 
arguendo t
hat
PGMCi sapubl i
cut i
l
ity
,pur
suantt o 
AlbanoS.

Reyes,
 25 
"i
tcanest ablishatelecommuni cati
onssystem evenwithoutal egi
slat
ivefranchise
becausenotev erypubl icut
il
ityisrequir
edt osecureal egisl
ati
vefranchi
sebef oreitcould
establ
i
sh,mai ntain,
andoper at etheservice";and,i
nanycase, "PGMC' sestabl
ishmentoft he
tel
ecommuni cationssy stem stipul
atedini t
scontractofleasewithPCSOf al
lswithinthe
excepti
onsunderSect ion1ofActNo.3846wher ealegislat
ivefr
anchiseisnotnecessar yfor
theestabl
i
shmentofr adiost
at i
ons."

Theyalsoar guethatthecontr
actdoesnotvi
olat
etheForeignInvest
mentActof1991; thatt he
Art
icl
esofI ncorporati
onofPGMCaut hori
zei
ttoenterint
ot heContr
actofLease;andthatt he
i
ssuesof" wisdom, mor al
i
tyandpropr
iet
yofactsoftheexecuti
vedepart
mentar ebeyondt he
ambitofjudicialr
eview."

Fi
nall
y,thepubl
icr
espondentsal
l
egethatt
hepeti
ti
onershavenostandi
ngt
omaint
aint
he
i
nstantsuit
,ci
ti
ngourresol
uti
onin 
Val
montevs.Phi
li
ppineChar
it
ySweepst
akesOf
fi
ce.
 26

Severalpart
iesfi
ledmot
ionst ointer
veneaspet i
ti
onersi
nthi
scase, 
27 butonlythemot i
onof
SenatorsAlbert
oRomulo,ArturoTolenti
no,Fr
anciscoTatad,
Glor
iaMacapagal -Arr
oyo,Vicent
e
SottoII
I,JohnOsmeña,RamonRev i
l
la,
andJoseLi na 
28 
wasgranted,andt herespondentswere
requir
edt ocommentontheirpet i
ti
oninint
erventi
on,whi
chthepubli
cr espondentsandPGMC
did.

I
nthemeant i
me,thepeti
tionersf
il
edwiththeSecurit
iesandExchangeCommi ssi
onon29
March1994apet i
ti
onagainstPGMCf orthenull
i
ficati
onofthelat
ter
'sGener
al I
nfor
mat
ion
Sheet
s.Thatcase,however,hasnobeari
nginthi
spet it
ion.

On11Apr il1994,wehear dt heparti


esinor
alargument s.Ther
eaft
er,
wer esolvedtoconsi
der
themattersubmi ttedforresolutionandpendi
ngr esol
utionofthemajori
ssuesi nthi
scase,t
o
i
ssueat empor aryr est
rai
ningor dercommandingt herespondent
soranyper sonacti
nginthei
r
placeorupont heirinstr
uctionstoceaseanddesi stfr
om implementi
ngthechal l
engedContr
act
ofLease.

Inthedeliberat
iononthiscaseon26Apr i
l1994,wer esolv edtoconsideronlyt heseissues: (
a)
thelocusstandi 
ofthepeti
ti
oners,and(b)thelegalit
yandv ali
dit
yoftheCont ractofLeasei nt he
l
ightofSect i
on1ofR. A.No.1169, asamendedbyB. P.Blg.42, whichprohibi
tst hePCSOf rom
holdingandconduct i
nglott
eri
es"incoll
aborati
on, associationorjoi
ntventurewi thanyper son,
associati
on,companyorent i
ty,whetherdomest i
corf oreign."Onthefir
stissue, sevenJust i
ces
votedt osustai
nthe l
ocusstandi 
ofthepetit
ioners,whil
esi xv ot
ednott o.Ont hesecondi ssue,
thesevenJusticeswer eoftheopini
ont hattheCont
ractofLeasev i
olat
est heexcepti
onto
Secti
on1(B)ofR. A.No.1169, asamendedbyB. P.Blg.42,andis,
theref
ore, i
nval
idandcontr
ary
tolaw.ThesixJust i
cesstatedthattheywishedtoexpressnoopinionthereoninviewofthei
r
standonthefirstissue.TheChiefJusti
cetooknopar tbecauseoneoft heDirect
orsofthe
PCSOi shi
sbr other-i
n-l
aw.

Thi
scasewast
henassi
gnedt
othi
s ponent
e f
ort
hewr
it
ingoft
heopi
nionoft
heCour
t.

Thepr eli
minar yissueont he l
ocusstandi ofthepetit
ioner sshould,i
ndeed, beresolvedintheir
favor .Apar ty'
sst andingbeforethi
sCour tisapr ocedural t
echnicali
tywhi chi tmay ,i
nthe
exer ciseofitsdi screti
on,setasi
deinv i
ewoft heimpor tanceoft heissuesr aised.Inthe
l
andmar k 
Emer gencyPower sCases, 
29 thisCour tbrushedasi dethistechni cal
itybecause"the
transcendent alimpor t
ancetothepublicoft hesecasesdemandst hatt heybeset t
ledprompt l
y
anddef ini
tely,brushingaside,i
fwemust , techni
calit
iesofpr ocedure.(Av el
inov s.Cuenco, 
G.R.
No.L- 2821)."Insofarastaxpayers'
suitsar econcerned, thi
sCour thaddecl ar edthatit"
isnot
dev oidofdi scretionast owhetherornoti tshouldbeent ert
ained,

30 ort hatit"enjoysanopen
discr eti
ont oent ert
ainthesameornot ." 
31  I
n DeLaLl anav s.Alba,
 32 t
hisCour tdeclar
ed:

1. Thear gumentast ot hel ackofst andi ngofpet iti


oner si seasi lyr esol v ed.Asf arasJudge
delaLl anaisconcer ned, hecer tainl yf allswi thint hepr inci plesetf or thi nJust iceLaur el's
opinionin Peopl ev s.Ver a [65Phi l
.56( 1937) ]
.Thus: "Theunchal lengedr ul ei st hatt heper son
whoi mpugnst hev alidityofast atut emusthav eaper sonal andsubst ant i
al int eresti nt hecase
sucht hathehassust ained, orwi llsust ain, dir
ecti njur yasar esul tofi tsenf orcement[ Ibid, 89].
Theot herpet i
tioner sasmember soft hebarandof ficersoft hecour tcannotbeconsi der edas
devoidof" anyper sonal andsubst ant i
al interest "ont hemat ter.Ther ei sr elev ancet ot his
excerptfrom asepar at eopi nionin  Aqui no, Jr.v .Commi ssiononEl ect i
ons  [L-40004, Januar y31,
1975, 62SCRA275] : "
Thent herei st heat t
ackont hest andi ngofpet itioner s,asv indicat ingat
mostwhatt heyconsi derapubl icr ightandnotpr ot ect i
ngt hei rr i
ght sasi ndi vidual s.Thi si sto
conjurethespect eroft hepubl i
cr ightdogmaasani nhi bitiont opar ti
esi ntentonkeepi ngpubl i
c
offi
cialsstayingont hepat hofconst itutional i
sm.Aswassowel lputbyJaf fe; "Thepr otect i
onof
pri
vat eri
ghtsi sanessent ial const ituentofpubl ici nt erestand, conv ersel y ,wi thoutawel l-
orderedst atetherecoul dbenoenf or cementofpr iv ater ight s.Pr ivat eandpubl icinter est sare,
bothi nasubst ant iveandpr ocedur al sense, aspect soft het otal i
tyoft hel egal order .
"Mor eover,
petit
ionershav econv incingl yshownt hati nthei rcapaci tyast axpay ers, thei rst andi ngt osue
hasbeenampl ydemonst rat ed.Ther ewoul dbear et reatf rom t hel i
ber al appr oachf ol l
owed
i
n Pascual v
.Secr et aryofPubl i
cWor ks, for eshadowedbyt hev erydeci sionof  Peopl ev .
Vera wher ethedoct rinewasf ir
stf ul l
ydi scussed, i
fweactdi fferent l
ynow.Idonott hinkwear e
preparedt otaket hatst ep.Respondent s, howev er ,woul dhar dbackt ot heAmer i
canSupr eme
Courtdoct ri
nei n Mel l
onv .Fr othingham, wi ththei rcl aimt hatwhatpet i
tioner spossess" i
san
i
nterestwhi chi sshar edi ncommonbyot herpeopl eandi scompar ativel ysomi nut eand
i
ndet erminateast oaf fordanybasi sandassur ancet hatt hej udi cial processcanactoni t."That
i
st ospeaki nt hel anguageofaby goneer a, eveni nt heUni tedSt at es.ForasChi efJust i
ce
War r
encl earlypoi ntedouti nt hel at ercaseof  Flastv .Cohen, t
hebar ri
ert hussetupi fnot
br
eachedhasdef
ini
tel
ybeenl
ower
ed.

I
n Kapati
ranngmgaNaglil
i
ngkodsaPamahal
aanngPil
i
pinas,
Inc.vs.Tan,
33 
reit
erat
ed
i
n Bascov s.Phi
l
ippi
neAmusement
sandGamingCor
porati
on,
34 t
hisCourtst
ated:

Objecti
onstot axpayer
s'suitsforlackofsuffici
entpersonali
tystandingorinterestare,however
,
i
nt hemai nproceduralmatters.Consider
ingthei mportancetothepubl i
coft hecasesatbar ,
andinkeepingwi ththeCour t
'sduty,underthe1987Const i
tut
ion,todeterminewhet herornot
theotherbranchesofgov ernmenthav ekeptt hemselveswithinthelimit
soft heConst i
tuti
on
andthel awsandt hattheyhav enotabusedt hediscr
etiongiventot hem,thi
sCour thasbrushed
asidetechni
calit
iesofprocedur eandhast akencognizanceoft hesepetit
ions.

andin 
Associat
ionofSmal
lLandowner
sint
hePhi
l
ippi
nes,
Inc.v
s.Secr
etar
yofAgr
ari
an
Refor
m,35 
itdeclar
ed:

Withparticul
arregardt
ot herequir
ementofpr operpar t
yasappl iedinthecasesbef or
eus,we
holdthatthesamei ssati
sfiedbythepet it
ionersandi ntervenor
sbecauseeachoft hem has
sustai
nedori sindangerofsust ai
ningani mmedi at
ei nj
ur yasar esul
toftheact sormeasures
complainedof .[
ExParte 
Lev i
tt
,303US633] .
 Andev enif, st
ri
ctl
yspeaking,theyarenotcovered
bythedef ini
ti
on,i
tisst
ill
withinthewi dediscreti
onoft heCour ttowaivether equi
rementandso
remov etheimpedimenttoitsaddr essingandr esol
vingt heseri
ousconst i
tuti
onalquesti
ons
rai
sed.

Inthef i
rstEmer gencyPower sCases, ordinarycitizensandt axpayerswer eal
l
owedt oquesti
on
theconst i
tuti
onalit
yofsev eralexecuti
v eordersissuedbyPr esi
dentQui r
inoalt
houghtheywer e
i
nv oki
ngonl yani ndi
rectandgener alinter
estshar edincommonwi ththepubl i
c.TheCourt
dismissedt heobjecti
vet hattheywerenotpr operpar tiesandr ul
edthatt hetr
anscendental
i
mpor tancet othepubl i
coft hesecasesdemandst hatt heybesettl
edpr omptlyanddefini
tel
y,
brushingaside,ifwemust ,t
echnicali
tiesofpr ocedur e.Wehav esi
ncet henappliedt
his
exceptioninmanyot hercases.(Emphasi ssuppl ied)

I
n Dazav
s.Si
ngson,
 
36 
thi
sCour
toncemor
esai
d:

...Foranother,
wehav eearl
yasint heEmergencyPowersCasesthatwher eseri
ous
consti
tut
ionalquesti
onsareinv
olved,"t
het
ranscendent
alimport
ancetot hepublicofthese
casesdemandst hattheybesettl
edpromptlyanddefi
nit
ely
,br
ushingaside,ifwemust ,
techni
cal
iti
esofprocedure.
"Thesamepol i
cyhassincethenbeenconsistentlyf
ollowedbythe
Court,
asin Gonzalesvs.CommissiononElecti
ons 
[21SCRA774]...

TheFeder
al SupremeCourtoftheUni
tedStatesofAmer i
cahasalsoexpr essedit
sdi
scr
eti
onar
y
powertol
iberali
zetherul
eon l
ocusst
andi.In 
UnitedStat
esvs.FederalPower
Commissi
on  andVi
rgi
niaReaAssoci
ati
onv s.Federal
PowerCommi ssi
on,37 i
thel
d:

Wehol
dthatpet
it
ioner
shavestandi
ng.Di
ff
erencesofv i
ew,however
,precl
udeasingl
eopini
on
oft
heCour
tastobothpeti
ti
oners.I
twoul
dnotfurthercl
ari
fi
cati
onofthiscompl
i
catedspeci
alt
y
offederal
jur
isdi
cti
on,t
hesolut
ionofwhoseprobl
emsisinanyeventmoreorl
essdet
ermined
bythespecif
icci
rcumstancesofi
ndiv
idual
sit
uati
ons,
tosetoutt
hediver
gentgr
oundsin
supportofst
andingint
hesecases.

Inli
newi tht hel i
beral policyoft hisCour ton locusstandi, ordinaryt axpay ers,member sof
Congr ess, andev enassoci ati
onofpl anters,andnon- profitcivicor ganizationswer eal lowedt o
i
niti
ateandpr osecuteact ionsbef orethisCour ttoquest i
ont heconst it
utionali
tyorv alidityof
l
aws, act s, decisions, ruli
ngs, orordersofv ari
ousgov ernmentagenci esori nstr
ument alit
ies.
Amongsuchcaseswer ethoseassai l
ingtheconst i
tut
ional i
tyof( a)R. A.No.3836i nsof arasi t
all
owsr etirementgr atuityandcommut ati
onofv acati
onandsi ckleav etoSenat orsand
Represent ativesandt oel ectiveoffici
alsofbot hHousesofCongr ess;38 (b)Execut i
veOr derNo.
284,issuedbyPr esidentCor azonC.Aqui noon25Jul y1987, whichal lowedmember soft he
cabinet ,theirunder secr etar i
es,andassi st
antsecr etari
est ohol dot hergov ernmentof fi
cesor
positi
ons;  39 (c)theaut omat i
cappr opriat
ionf ordebtser viceint heGener al Appropriations
Act;
 40  (
d)R. A.No.7056ont heholdingofdesy nchronizedel ect i
ons; 41 (d)R.A.No.1869( the
charteroft hePhi li
ppi neAmusementandGami ngCorpor ation)ont hegr oundt hatitiscont rar
y
tomor als,publ i
cpol icy ,
andor der; 
42 and( f)R.A.No.6975, est abli
shingt hePhi li
ppineNat i
onal

Pol
i
ce.
 43

Othercaseswher ewehav efoll


owedal i
ber alpol i
cyr egar ding locusst andi  
includet hose
attackingthev alidityorl egali
tyof( a)anor deral lowingt hei mpor tat i
onofr i
cei nt helightoft he
prohibiti
oni mposedbyR. A.No.3452;  
44 (b)P. D.Nos.991and1033i nsof arast heypr oposed
amendment stot heConst i
tut
ionandP. D.No.1031i nsof arasi tdirectedt heCOMELECt o
super vi
se,cont r
ol ,hold, andconductt her eferendum- plebi scit
eon16Oct ober1976;  45 (c)the
biddingfort hesal eoft he3,179squar emet ersofl andatRoppongi ,Minat o-ku, Tokyo,
Japan;  
46 (d)theappr ov alwithouthear ingbyt heBoar dofI nvest ment soft heamended
applicati
onoft heBat aanPet rochemi cal Cor por ati
ont ot ransf erthesi teofi tsplantf rom Bat aan
toBat angasandt hev alidi
tyofsucht ransf erandt heshi ftoff eedst ockf r
om napht haonl yt o
napht haand/ orliquef i
edpet r
oleum gas;  47( e)thedeci sions, order s,ruli
ngs, andr esolutionsof
theExecut iveSecr et
ary ,SecretaryofFi nance, Commi ssi onerofI nt ernal Rev enue,
Commi ssionerofCust oms, andt heFi scal Incent i
vesRev iewBoar dexempt ingt heNat i
onal
PowerCor porationf rom i ndir
ecttaxanddut i
es; 48 (f)theor dersoft heEner gyRegul atoryBoar d
of5and6December1990ont hegr oundt hatt hehear ingsconduct edont hesecondpr ovisi
onal
i
ncr easeinoi lpricesdi dnotal l
owt hepet itionersubst ant ial cross-exami nat i
on;  
49 (g)Execut ive
OrderNo.478whi chlev iedaspeci al dutyofP0. 95perl iterorP151. 05perbar relofimpor t
ed
crudeoi landP1. 00perl i
terofimpor tedoi l product s; 
50  (h)r esolut ionsoft heCommi ssionon
Electionsconcer ni ngtheappor ti
onment ,bydi st ri
ct,oft henumberofel ectivemember s
of Sangguni ans;
 51  and( i
)memor andum or der sissuedbyaMay oraf fectingt heChi efofPol i
ce
ofPasayCi ty.
52

I
nthe1975caseof 
Aqui
nov s.Commi
ssi
ononElecti
ons,
 53 
thi
sCourt
,despi
tei
tsunequi
vocal
r
uli
ngthatt
hepet
it
ioner
stherei
nhadnoper
sonal
it
ytofil
ethepeti
ti
on,r
esolv
ednevert
hel
essto
passupont heissuesrai
sedbecauseoft hefar-r
eachingimpl i
cati
onsofthepet i
ti
on.Wedi dno
l
essi n 
DeGui avs.COMELEC 54 where,alt
houghwedecl aredthatDeGuia"doesnotappeart o
havelocusst andi

astandi
nginlaw, apersonalorsubstantiali
nter
est,
"webr ushedasidethe
proceduralinfi
rmity"
consider
ingtheimpor t
anceoft heissueinvolv
ed,concerningasitdoesthe
poli
ti
cal exerci
seofqualif
iedvoter
saffectedbyt heappor t
ionment,andpeti
tioneral
legi
ng
abuseofdi screti
onandv i
olat
ionoftheConst i
tutionbyrespondent."

Wef indt
hei nstantpet i
ti
ont obeoft r
anscendentalimpor t
ancet othepubl ic.Thei ssuesi t
rai
sedareofpar amountpubl i
cinterestandofacat egoryev enhi ghert hant hosei nvolvedin
manyoft heaf orecitedcases.Ther ami f
icat
ionsofsuchi ssuesi mmeasur ablyaf f
ectt hesocial
,
economic,andmor alwell
-beingoft hepeopleeveni ntheremot estbarangay soft hecount ry
andthecount er-producti
veandr etr
ogressiv
eeffectsoft heenv isionedon- li
nel otter
ysy st
em
areasstagger i
ngast hebill
ionsinpesosi ti
sexpect edtor aise.Thel egal standingt henoft he
peti
ti
onersdeser vesr ecogniti
onand, intheexerciseofitssounddi scretion, t
hisCour thereby
brushesasidet hepr oceduralbarri
erwhi chtherespondent stri
edt ot akeadv antageof .

Andnowont
hesubst
ant
ivei
ssue.

Secti
on1ofR.A.No.1169,asamendingbyB. P.Blg.42,pr
ohi
bit
sthePCSOf r
om holdi
ngand
conduct
ingl
ott
eri
es"i
ncollabor
ati
on,associ
ationorjointv
ent
urewithanyperson,
associ
ati
on,
companyorenti
ty,
whetherdomesti
corf or
eign."Secti
on1provi
des:

Sec.1. 
ThePhi l
ippi
neCharit
ySweepst akesOffi
ce.—ThePhi li
ppi
neCharit
ySweepst akesOff
ice,
herei
nafterdesi
gnatedtheOffi
ce,shallbetheprincipalgover
nmentagencyforrai
singand
provi
dingforfundsforheal
thprograms, medicalassistanceandserv
icesandcharit
iesof
nati
onalcharacter,
andassuchshal lhav et
hegener alpowersconfer
redinsect
ionthirt
eenof
ActNumber edOnet housandfourhundr edfi
ft
y-nine,asamended,andshallhav
et heauthor
it
y :

A. Toholdandconductchari
tysweepstakesraces,l
ott
eriesandothersi
mil
aracti
vi
ti
es,i
n
suchfr
equencyandmanner ,asshal
lbedetermined,andsubjecttosuchrul
esandregul
ati
ons
asshal
l bepr
omulgat
edbyt heBoardofDirector
s.

B. Subj
ectt otheappr ovaloftheMi nisterofHumanSet t
lement s,
toengagei nhealthand
welfare-
rel
atedi nv
estment s,pr
ogr ams,  
proj
ectsandact i
vit
ieswhichmaybepr ofi
t-ori
ented, 
by
i
tselforincollaborati
on,associati
onorj oi
ntventure 
wit
hanyper son,associat
ion, companyor
enti
ty,whetherdomest i
corf or
eign, exceptfortheacti
vit
iesment ionedinthepr eceding
paragraph (
A) ,f
orthepur poseofpr ovidingforpermanentandcont i
nuingsourcesoff undsfor
healthprograms, i
ncludingtheexpansi onofexi sti
ngones,medi calassist
anceandser vices,
and/orcharitablegrants:Provi
ded, Thatsuchi nvestmentwillnotcompet ewi t
ht hepr i
vate
sectorinareaswher einv est
ment sar eadequat easmaybedet erminedbyt heNat ional
Economi candDev el
opmentAut hority
.(emphasi ssuppli
ed)

Thel
anguageoft
hesecti
onisi
ndi
sputabl
ycleart
hatwit
hrespectt
oitsf
ranchi
seorpri
vi
lege
"t
ohol
dandconductchar
it
ysweepst
akesraces,l
ott
eri
esandothersi
mil
aracti
vi
ti
es,
"the
PCSO  cannot
 exerci
seit"
incol
laborat
ion,associ
ationorjointvent
ure"withanyotherpart
y.Thi
s
i
st heunequivocalmeaningandimpor tofthephrase"exceptfortheactiv
iti
esmentionedinthe
precedingparagraph(A),
"namely,
"chari
tysweepstakesr aces,l
ott
eriesandothersimil
ar
activ
iti
es."

B.P.Bl
g.42ori
ginatedf
rom Par
li
ament
aryBil
lNo.622,whi
chwascov er
edbyCommitteeReport
No.103asreportedoutbyt
heCommi t
teeonSocio-
EconomicPlanni
ngandDevel
opmentofthe
I
nteri
m Bat
asangPambansa.Theorigi
nalt
extofparagr
aphB,Secti
on1ofPar
li
amentaryBi
llNo.
622readsasfol
lows:

Toengageinanyandal li
nvest
mentsandr elat
edprofi
t-
orient
edpr oj
ect
sorpr ogr
amsand
act
ivi
ti
esbyitsel
forincoll
abor
ati
on,associati
onorjoi
ntventurewi t
hanyperson,associ
ati
on,
companyorent i
ty,
whetherdomesti
corforeign,f
orthemai npurposeofrai
singfundsforheal
th
andmedicalassist
anceandservi
cesandchar i
tabl
egrants. 
55

Duri
ngtheper
iodofcommi t
teeamendment s,
theCommitteeonSocio-Economi
cPlanni
ngand
Devel
opment,t
hroughAssemblymanRonaldoB.Zamora,i
ntroducedanamendmentby
subst
it
uti
ontothesaidpar
agraphBsuchthat,asamended,i
tshouldreadasfol
lows:

Subjecttotheapprovaloft heMi nisterofHumanSet tl


ement s,t
oengagei nhealth-oriented
i
nvestments,programs, projectsandact ivi
ti
eswhichmaybepr ofi
t-ori
ented,byitselforin
col
laborati
on,associati
on,orj oi
ntv enturewit
hanyper son,associ
ation,
companyorent i
ty,
whetherdomest icorforei
gn, forthepur poseofprovi
dingforpermanentandcont inuingsour ces
offundsforhealthprograms, i
ncludingtheexpansionofexisti
ngones, medicalassist anceand
ser
v i
cesand/ orchari
tablegr ant
s. 56

Befor
ethemotionofAssembl
ymanZamorafort
heappr
ovaloft
heamendmentcoul
dbeact
ed
upon,Assembl
ymanDav i
dei
ntr
oducedanamendmentt
otheamendment:

MR.DAVI
DE.

Mr
.Speaker
.

THESPEAKER.

Thegent
lemanf
rom Cebui
srecogni
zed.

MR.DAVI
DE.

MayIint
roduceanamendmenttothecommi t
teeamendment?Theamendmentwoul
dbeto
i
nsertaf
ter"
for
eign"i
ntheamendmentjustr
eadthef
oll
owing:EXCEPTFORTHEACTIVI
TYIN
LETTER(A)ABOVE.

Wheniti
sjoi
ntventureorincollaborat
ionwit
hanyent
it
ysuchcol
labor
ati
onorjoi
ntvent
ure
mustnoti
ncludeacti
vit
yacti
vitylett
er(a)whi
chist
heholdi
ngandconduct
ingofsweepst
akes
r
aces,l
ott
eri
esandot hersi
milaracts.
MR.ZAMORA.

Weacceptt
heamendment
,Mr
.Speaker
.

MR.DAVI
DE.

Thanky
ou,
Mr.Speaker
.

THESPEAKER.

Ist
hereanyobj
ect
iont
otheamendment
?(Si
l
ence)Theamendment
,asamended,
is
appr
oved. 
57

Fur
theramendment stoparagr
aphBwer ei nt
roducedandapproved.WhenAssembl yman
Zamor ar
eadthefinal
textofparagraphBasf urt
heramended,theearl
i
erapprovedamendment
ofAssemblymanDav i
debecame" EXCEPTFORTHEACTI VITI
ESMENTI ONEDI NPARAGRAPH
(A)
";andbyvir
tueoftheamendmenti ntr
oducedbyAssembl ymanEmmanuel Pelaez,t
hewor
d
PRECEDINGwasi nsert
edbeforePARAGRAPH.Assembl ymanPel aezi
ntr
oducedot her
amendments.Thereaft
er,t
henewpar agraphBwasappr oved.
 58

Thi
sisnowpar
agr
aphB,
Sect
ion1ofR.
A.No.1169,
asamendedbyB.
P.Bl
g.42.

Noi nterpretati
onoft hesai dprovisi
ontor elaxorcircumv entt heprohibiti
oncanbeal l
owed
si
ncet hepr iv
il
eget oholdorconductchar i
tysweepst akesr aces,lott
er i
es,orothersi milar
acti
v i
tiesisaf ranchisegrantedbyt helegislatur
et othePCSO.I tisaset tl
edr ul
et hat"i
nal l
grantsbyt hegov ernmentt oindivi
dual
sorcor porati
onsofr ights,pri
vi
legesandf ranchises,the
wordsar et obet akenmostst ronglyagainstthegr antee...
.[o]newhocl ai
msaf ranchiseor
pri
vilegeinder ogati
onoft hecommonr ightsoft hepublicmustpr ovehi stit
letheretobyagr ant
whichi sclear l
yanddef i
nit
elyexpressed,andhecannotenl argeitbyequi vocalordoubt ful
provisionsorbypr obableinferences.What everisnotunequi vocallygrantediswi thheld.Nothing
passesbymer eimplicat
ion."
 59

Inshor tthen, bytheexcept i


onexpli
citlymadei nparagr aphB,Section1ofi tschar t
er,t
hePCSO
cannotshar eitsfranchisewithanotherbywayofcol laborati
on,associationorjointventure.
Neithercani tassi gn,tr
ansfer,orl
easesuchf ranchise.Ithasbeensai dt hat"theri
ghtsand
privil
egesconf er
redunderaf r
anchisemay ,
wi thoutdoubt ,beassignedort ransferr
edwhent he
grantist othegr anteeandassi gns,ori sauthorizedbyst atut
e.Ont heot herhand, t
her i
ghtof
transferorassi gnmentmayber est
rictedbyst atuteort heconstit
ution,orbemadesubj ectto
theappr oval ofthegr antororagover nmental agency, suchasapubl i
cut i
li
ti
escommi ssion,
except i
ont hatanexi sti
ngr i
ghtofassi gnmentcannotbei mpair
edbysubsequent
l
egi sl
ation."
 60

Itmayalsobepoint
edoutthatthefr
anchi
segrantedtothePCSOtoholdandconductl
otter
ies
all
owsittoholdandconductaspeci
esofgambling.I
tisset
tl
edthat"ast
atutewhi
chauthori
zes
thecar
ryingonofagamblingacti
vi
tyorbusi
nessshouldbestr
ict
lyconst
ruedandever
y
r
easonabl
edoubtsor
esol
vedast
oli
mitt
hepower
sandr
ight
scl
aimedunderi
tsaut
hor
it
y."
 61

Doesthechal
lengedCont
ractofLeasevi
olat
eorcontravenetheexcept
ioninSect
ion1ofR.A.
No.1169,asamendedbyB.P.Blg.42,
whichprohi
bit
st hePCSOfrom holdi
ngandconduct
ing
l
ott
eri
es"i
ncol l
abor
ati
on,
associati
onorj
ointvent
urewi t
h"anot
her?

Weagr eewitht hepet i


ti
oner st hatitdoes, notwithst andingi tsdenomi nat ionordesi gnati
onasa
(Cont r
actofLease) .Wear enei therconv i
ncednormov edorf azedbyt hei nsi st
enceandf orceful
argument soft hePGMCt hati tdoesnotbecausei nr ealit
yitisonl yani ndependentcont ractor
forapi eceofwor k,i.e.
,thebui ldingandmai nt enanceofal otterysystem t obeusedbyt he
PCSOi ntheoper at
ionofi tsl otteryfranchise.Whet hert hecont ractinquest ionisoneofl easeor
whet herthePGMCi smer elyani ndependentcont ract orshoul dnotbedeci dedont hebasi sof
thetitl
eordesi gnat i
onoft hecont ractbutbyt heintentoft hepar ti
es,whi chmaybegat hered
from theprovisionsoft hecont ractitsel
f. 
Animushomi nisestani mascr ipti.Thei ntenti
onoft he
partyisthesoul ofthei nstrument .Inordert ogiv elifeoref fecttoani nst rument ,itisessential
tolookt otheint entionoft hei ndi v
idualwhoexecut edi t.
 62 And, pursuantt oAr t
icle1371oft he
Civi
lCode, "t
odet ermi nethei nt enti
onoft hecont ractingpar ti
es, t
heircont empor aneousand
subsequentact sshal lbepr incipallyconsider ed."Toputi tmor ebluntl
y, nooneshoul dbe
deceivedbyt het itl
eordesi gnat ionofacont ract.

Acar efulanalysisandev al
uationoft hepr ov i
sionsoft hecont ractandaconsi der ati
onoft he
contempor aneousact soft hePCSOandPGMCi ndubitablydi scloset hatt hecont ractisnoti n
reali
tyacont ractofl easeunderwhi cht hePGMCi smer el
yani ndependentcont ractorfora
pieceofwor k,butonewher ethest atutoril
ypr oscr i
bed collabor ation or associati
on, inthel east
,
or j
ointventure, atthemost ,existsbetweent hecont r
act i
ngpar ties. 
Col l
aboration isdefinedas
theact sofwor ki ngtoget herinaj oi
ntpr oject. 
63  Associati
on  meanst heactofanumberof
personsi nunitingt ogetherf orsomespeci alpurposeorbusi ness.  64 Jointventure isdefinedas
anassoci ati
onofper sonsorcompani esjoi nt
lyunder takingsomecommer ci
alent erpri
se;
gener al
lyallcont ri
buteasset sandshar eri
sks.I trequiresacommuni tyofi nt
eresti nthe
performanceoft hesubj ectmat ter,
ar ightt odirectandgov ernt hepol icyinconnect i
on
therewith,anddut y,whichmaybeal teredbyagr eementt oshar ebot hi npr of
itand

l
osses.
65

Thecontemporaneousact soft hePCSOandt hePGMCr evealthatthePCSOhadnei therfunds


ofit
sownnort heexper t
iset ooper ateandmanageanon- li
nelotter
ysystem, andt hatalthough
i
twishedtohav ethesystem, i
twoul dhav eit"atnoexpenseorr i
skstothegov er
nment ."
Becauseoftheseseriousconst raintsandunwi l
li
ngnesstobearexpensesandassumer isks,
the
PCSOwascandi denought ost atei nitsRFPt hatitisseekingfor"asuit
abl econtractorwhi ch
shal
lbuil
d,ati
tsownexpense, allthef acil
i
tiesneededt ooperateandmai ntai
n"thesy stem;
excl
usiv
elybear"all
capital
, oper atingexpensesandexpansi onexpensesandr i
sks";andsubmi t
"acomprehensi
v enati
onwi delot terydev el
opmentpl an...whichwilli
ncludet hegame, t
he
marketi
ngofthegames, andt hel ogisticstointroducethegamet oallt
heci ti
esand
muni cipal
iti
esofthecountrywithinfi
ve(5)y ears";andt hatt heoper ati
onoft heon- li
nelottery
system shoul dbe"atnoexpenseorr i
sktothegov ernment "—meani ng i
tsel
f,sincei ti
sa
gov er nment-ownedandcont rol
ledagency .The facili
ti
es referredt omeans" allcapi t
al
equi pment ,comput er
s,ter
minals,soft
ware, nat
ionwi det elecommuni cati
onsnet wor k,ti
cket
salesof fi
ces,fur
nishi
ngsandf i
xtures,pr
inti
ngcost s,costsofsal ari
esandwages, advert
ising
andpr omotionsexpenses,maintenancecost s,
expansi onandr eplacementcost s, securi
tyand
i
nsur ance,andallotherrel
atedexpensesneededt ooper ateanat ionwideon-li
nel otter
y
system. "

I
nshor t
,theonl ycont ri
but i
onthePCSOwoul dhav eisit
sf r
anchiseoraut horit
ytooper atethe
on-l
inelotterysy stem; withtherest,includingthe ri
sks ofthebusiness, bei
ngbor nebyt he
proponentorbi dder .Itcouldbef orthisr easont hatitwarnedthat" t
hepr oponentmustbeabl e
tostandt otheaci dt estofpr ovi
ngt hatitisanent i
tyabletotake onther oleofresponsible
maint ai
neroft heon- li
nel ott
erysystem. "ThePCSO, however,makesi tclearinitsRFPt hatthe
proponentcanpr oposeaper i
odoft hecont r
actwhi chshallnotexceedf i
ft
eeny ears,duri
ng
whicht i
mei tisassur edofa" rent
al"whi chshal lnotexceed12%ofgr ossreceipts.Asadmi tted
bythePGMC, uponl earni ngofthePCSO' sdeci si
on, t
heBer j
ayaGr oupBer had,withitsaffi
li
ates,
want edtoof ferits servicesandr esour ces tothePCSO.For t
hwith,itorganizedthePGMCas" a
medi um throughwhi cht he t
echnicalandmanagement  
servi
cesr equiredforthepr oj
ectwoul d
beof fer
edanddel i
v eredt oPCSO. " 
66

Undoubtedly ,t
hen, t
heBer j
ayaGr oupBer hadknewal lalongt hatinconnect ionwi t
hanon- l
ine
l
otterysystem, thePCSOhadnot hi ngbuti tsf r
anchise,whi chitsol emnlyguar anteedi thadi n
theGeneral Inf
or mati
onoft heRFP.  
67 Howsoev erviewedt hen,from t
hev er yinception,the
PCSOandt hePGMCmut ual l
yunder st
oodt hatanyar r
angementbet weent hem woul d
necessari
lyleav etothePGMCt he  technical,operati
ons, andmanagement  aspect soft heon-
l
inelott
erysy stem whilethePCSOwoul d,primaril
y,providet hef r
anchise.The
words Gami ng and Management  int hecor poratenameofr espondentPhi lippineGami ng
ManagementCor porat
ioncouldnothav ebeenconcei vedjustf oreuphemi sticpur poses.Of
course,t
heRFPcannotsubst itutef ort heCont ractofLeasewhi chwassubsequent lyexecut ed
bythePCSOandt hePGMC.Nev ertheless, theCont r
actofLeasei ncor
por atest hei
rintention
andunder standing.

Theso- call
edCont r
actofLeasei snot,therefore,whati tpurportstobe.I t
sdenomi nati
onas
suchisacr aft
ydevice,careful
lyconceived, t
opr ovideabui lt-
indefensei ntheev entthatt
he
agreementi squestionedasv i
olat
iveoft heexcept i
oni nSect i
on1( B)oft hePCSO' schart
er.The
acuit
yorski l
lofi
tsdraftsment oaccompl i
sht hatpur poseeasi l
ymani festsit
selfintheCont r
act
ofLease.I tisoutst
andingf orit
scar ef
ul andmet iculousdr aft
ingdesignedt ogiveani mmediate
i
mpr essiont hati
tisacont ractoflease.Yet ,
wov ent hereinarepr ovi
sionswhi chnegat eit
stit
le
andbet raythetrueintenti
onoft hepartiestobei nort ohav ea j
ointventure f
oraper i
odofeight
yearsint heoperati
onandmai ntenanceoft heon- li
nel ott
erysy stem.

Consi
stentwi
tht
heabov
eobser
vat
ionsont
heRFP,
thePCSOhasonl
yit
sfr
anchi
set
oof
fer
,
whi lethePGMCr epr esent sandwar rantst hatithasaccess  toal lmanager i
al andt echni cal
exper t
ise topr ompt l
yandef fect ivelycar ryoutt het ermsoft hecont ract .And, foraper i
odof
eighty ears,thePGMCi sunderobl i
gat i
ont okeepal lt
he  Faci l
ities insaf econdi tionandi f
necessar y ,
upgr ade, repl ace, andi mpr ov et hem f rom t imet ot i
measnewt echnol ogydev elops
tomaket heon- linelot ter ysy stem mor ecost -
effect i
veandcompet iti
v e; exclusi v elybearal l
cost sandexpensesr elat i
ngt ot hepr inting, manpower ,salariesandwages, adv ertisingand
promot i
on, mai ntenance, expansi onandr eplacement , secur i
tyandi nsur ance, andal l ot
her
relatedexpensesneededt ooper atet heon- li
nel otterysy stem; under takeaposi tiveadv er ti
sing
andpr omot ionscampai gnf orbot hinst i
tut ional andpr oductl ineswi thoutengagi ngi nnegat ive
adv er t
isingagai nstot herl essor s; beart hesal ariesandr el
at edcost sofski lledandqual ifi
ed
per sonnel f
or  admi nistr ativeandt echni cal  
oper ations; compl ywi th procedur al andcoor dinating
rules issuedbyt hePCSO; andt ot rainPCSOandot herl ocal per sonnel andt oef fectt het r
ansf er
oft echnol ogyandot herex pertise, sucht hatatt heendoft het er m oft hecont ract ,thePCSOwi ll
beabl et oef fectivelyt akeov ert heFaci l
itiesandef fi
cient l
yoper atet heon- li
nel ot terysy stem.
Thel attersimpl ymeanst hat ,i
ndeed, themanager s,techni ciansorempl oyeeswhoshal l
oper atet heon- l
inel otter ysy stem ar enotmanager s,techni ciansorempl oyeesoft hePCSO, but
oft hePGMCandt hati tisonl yaf tertheexpi rati
onoft hecont ractt hatt hePCSOwi lloperatet he
syst em.Af terei ghty ear s, thePCSOwoul daut omat i
callybecomet heowneroft heFaci li
ti
es
withoutanyot herf urtherconsi der ation.

Fortheser easons, too,thePGMChast hei ni


ti
alprer
ogat i
vetopr eparethedetai
ledpl anofal l
gamesandt hemar ket
ingthereof ,
anddet erminethenumberofpl ayers,v
alueofwi nnings,and
thelogisti
csr equiredtointroducet hegames, incl
udingtheMast erGamesPl an.Ofcour se,the
PCSOhast her eser vedauthoritytodisapprovethem. 68 And,whi l
ethePCSOhast hesol e
responsibil
ityov ertheappoi nt
mentofdeal er
sandr etai
lersthroughoutthecount r
y,thePGMC
may ,nevertheless,recommendf orappointmentdealersandr etail
erswhichshallbeact edupon
bythePCSOwi t
hinf or
ty-ei
ghthour sandcol l
ectandr et
ain,forit
sownaccount ,asecur i
ty
depositfrom deal ersandr etai
lersinrespectofequipmentsuppl i
edbyi t
.

Thi
sjoi
ntv
ent
urei
sfur
therest
abl
i
shedbyt
hef
oll
owi
ng:

(a) Renti sdefi


nedint heleasecont ractast heamountt obepai dt othePGMCas
compensat i
onfortheful
fil
lmentofi tsobl i
gati
onsundert hecont r
act ,i
ncluding,butnot
l
imi t
ed  t
otheleaseoftheFaci l
iti
es.Howev er,
this 
rent i
snotact uallyafixedamount .Alt
hough
i
tisst atedtobe4.9%ofgr ossreceiptsfrom ticketsales,payablenetoft axesr equiredbylawto
bewi thheld,i
tmaybedr ast i
call
yreducedor ,
inextremecases, nothingmaybedueor
demandabl eatal
lbecauset hePGMCbi ndsitselfto"bearallri
sksi ftherev enuefrom theti
cket
sales,onanannual i
zedbasi s,areinsuff
icienttopayt heent i
reprizemoney .
"Thisrisk-
beari
ng
provisionisunusuali
nal essor-l
esseer elati
onship,butinherentinaj oi
ntv entur
e.

(b) I ntheeventofpr
e-terminat
ionofthecontr
actbythePCSO, ori
tssuspensi
onof
oper
ati
onoftheon-li
nel
otterysyst
em inbreachofthecontractandt
hroughnofaul
tofthe
PGMC,thePCSObi ndsi
tself"t
opromptly
, andi
nanyev entnotlat
ert
hansixt
y(60)day
s,
rei
mbursetheLessortheamountofitstotal
investmentcostassoci
atedwit
ht heOn-Li
ne
Lott
erySyst
em, i
ncl
udingbutnotli
mitedtothecostoftheFacili
ti
es,
andfurthercompensate
theLESSORforlossofexpectednetprof
itaft
ertax,computedovert
heunexpi r
edter
m ofthe
l
ease."I
fthecontr
actwereindeedoneoflease,thepaymentoftheexpectedprofi
tsorrent
als
fort
heunexpir
edporti
onoft heter
m ofthecontractwouldbeenough.

(c) ThePGMCcannot" dir


ectl
yorindi
rect
lyunder
takeanyactivi
tyorbusinessincompet i
ti
on
withoradv er
setotheOn- LineLot
ter
ySy st
em ofPCSOunl essi
tobtainsthelatt
er'
spr i
orwrit
ten
consent."I
fthePGMCi sengagedi nthebusinessofl
easingequi
pmentandt echnologyforan
on-l
inelott
erysyst
em, wef ailt
oseeanyacceptabler
easonwhyi tshouldall
owar estri
cti
onon
thepursuitofsuchbusiness.

(d) ThePGMCshal lpr


ovi
det hePCSOt heaudi tedAnnualRepor tsentt
oitsstockholders,
andwithintwoy earsfr
om theef f
ecti
vi
tyoft hecont r
act,
causeitsel
ft obeli
stedinthelocal
stockexchangeandof feratleast25%ofi t
sequi tytothepubli
c.IfthePGMCi smer elyalessor,
thi
simposi t
ionisunreasonableandwhi msical,andcouldonlybet i
edupt othefactthatthe
PGMCwi ll
actuall
yoperateandmanaget hesy stem;hence,i
ncreasingpubli
cparti
cipati
onint he
corpor
ationwoul denhancepubl i
cint
erest.

(
e) ThePGMCshal lputupanEscr owDepositofP300,000,
000.00pursuanttot
he
r
equi
rement
soft
heRFP,whichitmay, ati
tsopt
ion,mai
ntainasit
si ni
ti
alper
formancebond
r
equi
redt
oensur
eit
sfai
thfulcompl
iancewi t
hthetermsofthecontract.

(f) ThePCSOshal ldesignat ethenecessar


yper sonneltomoni t
orandaudi tt hedai
ly
performanceoft heon- l
inelotterysystem;andpromul gate 
proceduralandcoor dinat
ing
rul
es gov erni
ngallacti
viti
esrelatingtotheon-l
inelotter
ysystem.Thef ir
stfurtherconfi
rmsthat
i
tist hePGMCwhi chwi lloperatet hesyst
em andt hePCSOmay ,fortheprotectionofit
sint
erest
,
moni torandauditthedai l
yper formanceofthesy stem.Thesecondadmi ts
the 
coor dinat
ing 
and cooperative powersandfunct i
onsofthepar t
ies.

(g) ThePCSOmayv al
idl
yterminatethecontr
acti
fthePGMCbecomesi nsol
ventor
bankr
uptorisunabl
etopayit
sdebt s,ori
fitst
opsorsuspendsort
hreat
enstostoporsuspend
paymentofall
oramateri
alpartofitsdebt
s.

Alloftheforegoingunmi stakablyconf ir
mt hei ndispensabler oleoft hePGMCi nt hepursuit,
operati
on,conduct ,andmanagementoft heOn- LineLot ter
ySy stem.Theyexhi bitand
demonst r
atet hepar t
ies'indivi
sibl
ecommuni tyofi nter
estint heconcept i
on, birt
handgr owthof
theon-li
nelottery,and,abov eall,i
ni tsprofi
ts, witheachhav ingar ighti
nt hef ormul at
ionand
i
mpl ementationofpol ici
esr elatedtot hebusi nessandshar ing, aswel l
,i
nt hel osses—wi ththe
PGMCbear i
ngt hegr eatestbur denbecauseofi tsassumpt ionofexpensesandr isks,
andt he
PCSOt heleast ,
becauseofi tsconf essedunwi ll
ingnesst obearexpensesandr i
sks.Inamanner
ofspeaking,eachi swedt ot heotherf orbetterorf orwor se.Int hef i
nalanalysis, however,i
nt he
l
ightofthePCSO' sRFPandt heabov ehighl
ight edprovisions, aswel lasthe" Hol dHarml ess
Clause"oftheCont r
actofLease, itisevensaf etoconcl udet hatt heactual 
lessor  i
nthiscasei s
t
hePCSOandthesubj
ectmatt
erther
eofi
sit
sfranchi
set
ohol
dandconductlot
ter
iessincei
tis,
i
nreal
it
y,t
hePGMCwhichoper
atesandmanagestheon-
li
nel
ott
erysy
stem f
oraperiodofei
ght
y
ears.

Wethusdeclar
ethatthechall
engedCont r
actofLeasevi
olat
estheexceptionprovi
dedf ori
n
par
agraphB,Sect
ion1ofR. A.No.1169,asamendedbyB. P.Bl
g.42,andis,ther
efore,
inval
idf
or
bei
ngcontr
arytolaw.Thi
sconcl usi
onrender
sunnecessaryfur
therdi
scussionont heother
i
ssuesrai
sedbythepetit
ioners.

WHEREFORE, theinstantpeti
ti
oni
sherebyGRANTEDandt hechall
engedCont
ractofLease
execut
edon17December1993byr espondentPhi
l
ippineChar
itySweepst
akesOffi
ce(PCSO)
andrespondentPhil
ippineGamingManagementCor porat
ion(
PGMC)i sher
ebyDECLARED
contr
arytolawandi nvali
d.

TheTempor
aryRest
rai
ningOr
deri
ssuedon11Apr
il1994i
sher
ebyMADEPERMANENT.

Nopr
onouncementast
ocost
s.

SOORDERED.

Regal
ado,
Romer
oandBel
l
osi
l
lo,
JJ.
,concur
.

Nar
vasa,
C.J.
,tooknopar
t.

G.
R.No.187714 Mar
ch8,
2011

AQUI
LINOQ.PI
MENTEL,
JR.
,MANUELB.VILLAR,
JOKERP.ARROYO,
FRANCI
SN.PANGI
LINAN,
PI
AS.CAYETANO,
andALANPETERS.CAYETANO,Pet
it
ioner
s,

v
s.

SENATECOMMI TTEEOFTHEWHOLEr
epr
esent
edbySENATEPRESI
DENTJUANPONCE
ENRI
LE,Respondent
s.

DECISION

CARPI
O,J.
:

TheCase

BeforetheCourtisapeti
t i
onforprohibi
tion1wi thprayerforissuanceofawr i
tofprel
imi nar
y
i
njuncti
onand/ort emporaryr
estr
ainingor derfi
ledbySenat orsAqui l
inoQ.Pi
ment el
,Jr.(Senat
or
Pi
ment el),
Manuel B.Vi
ll
ar(SenatorVill
ar),JokerP.Arroyo,FrancisN.Pangil
inan,
PiaS.
Cayetano,andAlanPeterS.Cayetano( petiti
oners)
.Pet i
ti
onersseekt oenj
ointheSenat e
Commi tteeoft
heWhol e( r
espondent)from conduct i
ngf urt
herhearingsonthecompl aintfi
l
ed
bySenat
orMari
aAnaConsuel
oA. S.Madr i
gal(
SenatorMadr
igal)agai
nstSenatorVil
l
arpur
suant
toSenat
eResol
uti
onNo.706(P.S.Resolut
ion706)ontheal
legeddoubleinser
tionof₱200
mil
li
onfort
heC-5RoadExt
ensionProjecti
nthe2008GeneralAppropri
ati
onsAct .

TheAnt
ecedent
s

On15Sept ember2008, Senat orPanf i


loLacson( Senat
orLacson)del iveredapr i
v i
l
egespeech
enti
tl
ed" KabanngBay an,Bant ayan!
"2I nhisprivil
egespeech, SenatorLacsoncal l
edat t
entionto
thecongr essionali
nser t
ioni nt he2008Gener al Appropr
iati
onsAct ,parti
cularl
yt he₱200mi l
li
on
appropriatedfortheconst ructionoft hePr esi
dentCar l
osP.Gar ciaAv enueExt ensionfrom
SucatLuzonExpr esswayt oSucatRoadi nPar añaqueCityincludingRight -
of-
Way( ROW) ,and
another₱200mi l
li
onappr opriatedfort heextensionofC- 5roadincludingROW.Senat orLacson
stat
edt hatC- 5iswhatwasf or merlycalledPresidentCarlosP.Gar ciaAv enueandt hatthe
secondappr opri
ati
oncov erst hesamest r
etch–f r
om SucatLuzonExpr esswayt oSucatRoadi n
ParañaqueCi t
y.SenatorLacsoni nquiredfrom DBM Secr etaryRolandoAnday a,Jr.aboutt he
doubleent ryandwasi nformedt hatitwasonaccountofacongr essional i
nserti
on.Senat or
Lacsonf urtherstatedthatwhenhef oll
owedt henar rowtraill
eadingtot hedoubl eent r
y,itledto
SenatorVi l
lar,t
hent heSenat ePr esi
dent .

On8October2008,
Senat
orMadr
igal
int
roducedP.
S.Resol
uti
on706,
3thef
ull
textofwhi
ch
r
eads:

WHEREASt heSenatePresi
denthasrepeat
edlyandpubl
i
cly"
advocat
ed"(si
c)theconst
ruct
ion
oftheC-5Road/Pres.C.
P.Gar
ciaAvenueExtensi
onli
nki
ngSucatRoadinParañaqueCi
tytothe
SouthLuzonExpressway;

WHEREASitwasdiscoveredt
hatt
herewasadoublei
nsert
ionof₱200mi
l
li
onf
ort
heC-
5Road
Ext
ensi
onproj
ecti
nt he2008Gener
alAppr
opr
iat
ionsAct;

WHEREASCommit
teeonFi
nanceChai
rSen.JuanPonceEnr
il
econfi
rmedthatt
hedoubl
e
i
nser
ti
onfort
heC-
5RoadExtensi
onPr
ojectwasmadebytheSenat
ePresi
dent;

WHEREASt
hisdoubl
einser
ti
oni
sonl
ythet
ipoft
hei
ceber
g;

WHEREASt hereisov er
whelmingev idencetoshowt hatt
heSenat ePresi
dent,f
rom t
hetimehe
wasmemberoft heHouseofRepr esentati
ves,
usedhisinfl
uenceont heexecuti
vetocauset
he
real
ignmentoft heC-5RoadExt ensionpr oj
ecttoensurethathisproper
ti
esinBarangaySan
Dionisi
o,ParañaqueCi t
yandBar angay sPulangLupaandMay unoUno,LasPiñaswouldbe
fi
nancial
lybenefitedbytheconstructionofthenewr oad;

WHEREASt hereisoverwhelmingevi
dencetoshowt hatt
heSenatePr
esi
dent
,thr
oughhis
cor
por
ations,negot
iatedthesaleofhi
sproperti
esasroadsr
ightofwayt
othegovernment,
the
samepropert
iesaffectedbytheproj
ectsheproposed;

WHEREASt
her
eisov
erwhel
mingev
idencet
oshowt
hatt
heSenat
ePr
esi
dentcausedt
hesal
eof
hisl
andholdi
ngstogov
ernmentasagrossl
yover
pri
cedcostpr
ejudi
cial
toot
herl
otowner
sin
thear
ea,thegover
nment,andt
heFi
li
pinopeopl
e;

WHEREASt herei
sov erwhel
mi ngevi
dencetoshowthattheSenat
ePresi
dent
,intheover
pri
ced
saleofanot
herpropert
y,usedhispowerandinfl
uencetoextor
tfr
om t
heori
ginal
landownerthe
profi
tmadefrom theoverpr
icebytheVil
larownedcorpor
ati
ons;

WHEREAStheseact
softheSenatePr
esi
dentar
eindirectv
iol
ati
onoftheConsti
tut
ion,
theAnti
-
Gr
aftandCor
ruptPr
act
icesAct
,theCodeofConductandEthi
calSt
andardsofPubl
icOffi
cer
s;

WHEREAStheSenatePresi
denthasvi
olat
edt
hepubl
i
ct r
ustoft
hepeopl
einor
dert
oser
vehi
s
per
sonal
int
erest
sther
ebysacri
fi
cingt
hepeopl
e’
swelf
are;

WHEREAStheil
l
egalandunethi
calconductoft
heSenatePresi
denthasbet
ray
edt
het
rustof
t
hepeopl
e,andbydoingsohasshamedt hePhil
ippi
neSenate;

WHEREASi ti
si ncumbentuponthemembersoftheSenat
enowtorecl
aimthepeopl
e’st
rust
andconf
idenceandshowt hatt
heil
l
egal
conductofanyofit
smember,ev
enofit
sleader
s,shal
l
notgounpunished;

WHEREFORE,BEITRESOLVEDASI TISHEREBYRESOLVED,TODI
RECTTHECOMMI TTEEON
ETHICSANDPRIVI
LEGESTOI NVESTIGATETHECONDUCTOFSENATEPRESI DENTMANUELB.
VILLAR,
JR.FORUSINGHISPOSITIONOFPOWERTOI NFLUENCEPUBLI
COFFI CI
ALSIN
RELOCATINGTHEC-5ROADEXTENSI ONPROJECTTODELIBERATELYPASSTHRUHI S
PROPERTIES,
ANDTONEGOTI ATETHEOVERPRICEDPURCHASEOFROADRI GHTSOFWAY
THRUSEVERALPROPERTIESALSOOWNEDBYHI SCORPORATIONSREDOUNDI NGI
NHUGE
PERSONALFINANCIALBENEFITSFORHIM TOTHEDETRIMENTOFTHEFILIPINOPEOPLE,
THEREBYRESULTINGINABLATANTCONFLI CTOFINTEREST.

Adopt
ed,

(
Sgd.
)

M.
A.MADRI
GAL4

Onevendat
e,P.
S.Resolut
ion706wasref
err
edt
otheCommit
teeonEt
hicsandPri
vi
leges
(
Ethi
csCommitt
ee)whichatthatt
imewascomposedoft
hef
oll
owi
ngmember s:

Sen.Pi
aS.Cay
etano-Chai
rper
son

Sen.Lor
enLegar
da-Memberi
nli
euofSen.Madr
igal

Sen.JokerAr
roy
o-Member

Sen.Al
anPet
erCay
etano-Member

Sen.Mi
ri
am Def
ensor
-Sant
iago-Member
Sen.Gr
egor
ioHonasan-Member

Sen.Panf
il
oLacson-I
nhi
bit
edandr
epl
acedbySen.Rodol
foBi
azon

On17Nov ember2008, Senat orJuanPonceEnr i


l
e( Senat orEnri
le)wasel ectedSenat ePr esident.
TheEt hicsCommi t
teewasr eorganizedwi ththeelecti
onofSenat orLacsonasChai rperson, and
SenatorsRi char dGordon, Gregor i
oHonasan, Lor
enLegar da,andMarRoxasasmember sf orthe
Majori
t y.On16December2008, Senat orLacsoninquiredwhet hert heMi nori
tywasr eadyt o
namet hei rrepr esentati
vest otheEt hicsCommi t
tee.5Af terconsul tati
onwi t
ht hemember sof
theMinor i
ty, SenatorPiment el i
nformedt hebodyt hatther ewoul dbenomemberf rom t he
Minori
t yint heEt hi
csCommi t t
ee.6On26Januar y2009, SenatorLacsonr ei
ter
atedhi sappeal t
o
theMinor i
tyt onomi natet hei
rr epresentati
vestotheEt hicsCommi ttee.
7Senat orPiment el
stat
edt hati ti sthestandoft heMi norit
ynottonomi nateanyoft hei rmember stotheEt hi cs
Commi ttee, buthepr omi sedt oconv eneacaucust odeter mineift heMi nori
ty’
sdeci sionont he
matterisf inal .
8Ther eafter,t
heSenat eadoptedtheRul esoft heSenat eCommi t
teeonEt hics
andPr ivi
leges( Commi t
teeRul es)whi chwaspubl ishedint heOf fi
ci alGazetteon23Mar ch
2009.9

On20Apr i
l2009, SenatorVil
l
ardeli
veredapr iv
il
egespeech10wher ehest atedthathewoul d
answert heaccusationsagainsthim onthefloorandnotbef oretheEt hi
csCommi ttee.On27
April2009,SenatorLacsondel i
ver
edanot herprivi
legespeech11wher ehest atedthatt heEthics
Commi t
teewasnotakangar oocour t
.Howev er,duetot heaccusati
ont hattheEthics
Commi t
teecouldnotactwi thfai
rnessonSenat orVi l
l
ar’scase,SenatorLacsonmov edthatthe
responsibi
li
tyoftheEt hicsCommi t
teebeunder takenbyt heSenate,actingasaCommi t
teeof
theWhol e.Themot ionwasappr ovedwithtenmember svoti
nginfavor ,
noneagai nst ,
andf i
ve
abstenti
ons.12

RespondentSenat eCommi t
teeoft heWhol econduct edi t
shear i
ngson4May2009, withel ev
en
Senat orspresent, andon7May2009, wit
hei ghtSenat orspr esent .Onbot hheari
ngs,peti
tioners
objectedt otheappl icati
onoft heRulesoft heEt hicsCommi tteetot heSenat eCommi t
teeoft he
Whol e.Inpar t
icular,peti
ti
oner squest i
onedt hedet er
mi nati
onoft hequor um.On11May2009,
petit
ioner sproposed11amendment stot heRulesoft heEt hicsCommi tt
eet hatwould
constitutetheRul esoft heSenat eCommi tt
eeoft heWhol e, outofwhi cht hr
eeamendment s
wereadopt ed.On14May2009, SenatorPi mentel rai
sedasani ssuet heneedt opubl
isht he
proposedamendedRul esoft heSenat eCommi tteeoft heWhol e.Onev endat e,r
espondent
proceededwi ththePr eli
mi naryInquiryonP. S.Resol uti
on706.On18May2009, t
heChai rman
submi ttedar eportont hePr eliminaryInquirywithadi rectivet oallSenat orstocomeupwi t
ha
decisionont hepr eliminaryrepor ton21May2009.On21May2009, r
espondentdecl aredthat
therewassubst ant i
alevidencet opr oceedwi ththeadj udicat oryhear i
ng.Thepr eli
minary
confer encewasseton26May2009.

Pet
it
ioner
scamet
othi
sCour
tforr
eli
ef,
rai
singt
hef
oll
owi
nggr
ounds:

1.Thet
ransf
eroft
hecompl
aintagai
nstSenat
orVi
l
larf
rom t
heEt
hicsCommi
tt
eet
otheSenat
e
Commi
tt
eeoft
heWhol
eisv
iol
ati
veofSenat
orVi
l
lar
’sconst
it
uti
onal
rightt
oequal
prot
ect
ion;

2.TheRul esadoptedbytheSenat
eCommi t
teeoftheWholef
ort heinvesti
gat
ionofthe
complaintfi
ledbySenatorMadri
galagai
nstSenatorVi
l
lari
sviolati
veofSenatorVil
l
ar’sr
ightt
o
dueprocessandoft hemajori
tyquorum r
equir
ementunderAr
t .VI,Sec.16(
2)ofthe
Consti
tuti
on; and

3.TheSenateCommitteeoftheWholeli
kewisevi
olatedt
heduepr ocesscl
auseoft
he
Consti
tut
ionwhenitr
efusedtopubl
ishtheRulesoftheSenat
eCommi tt
eeoftheWhol
einspi
te
ofit
sownpr ov
isi
on[which]r
equi
re[
s]it
seffect
ivi
tyuponpubl
icat
ion.13

I
nit
sComment
,respondentar
guest
hat
:

1.Theinstantpet
it
ionshoul
dbedismissedforfai
lur
etoj
oinorimpl
eadanindispensabl
epar
ty.
I
nt healt
ernati
ve,
theinst
antpet
it
ionshoul
dbear chi
vedunti
lsucht
imethatt
hesai d
i
ndispensablepar
tyhasbeenjoi
nedorimpleadedandaffor
dedtheoppor
tuni
tytobeheard;

2.Ther
ewasnogr
aveabuseofdi
scr
eti
onont
hepar
tofr
espondentCommi
tt
ee;

3.Peti
ti
oner
sarenotent
it
ledtoawr i
tofpr
ohibi
ti
onforf
ail
uret
opr
ovegr
aveabuseof
di
scret
iononthepar
tofrespondentCommitt
eeoftheWhole;

4.Thepr
inci
pleofsepar
ati
onofpower
smustbeuphel
d;

5.Theinst
antpet
it
ionmustbedismissedforbei
ngpr
emat
ure.Pet
it
ioner
sfai
l
edt
oobser
vet
he
doctr
ineorpr
imaryjur
isdi
cti
onorpr
iorresor
t;

6.I
tiswi
thi
nthepowerofCongr
esst
odi
sci
pli
nei
tsmember
sfordi
sor
der
lybehav
ior
;

7.Thedet
ermi
nat
ionofwhatconsti
tut
esdi
sor
der
lybehav
iori
sapol
i
tical
quest
ionwhi
ch
excl
usi
vel
yper
tai
nstoCongress;

8.TheInt
ernalRul
esoftheSenat
ear
enotsubj
ectt
ojudi
cial
rev
iewi
ntheabsenceofgr
ave
abuseofdiscr
eti
on;[
and]

9.TheRul
esoftheEt
hicsCommit
tee,
whichhavebeendulypubli
shedandadopt
ed[
,
]al
l
owt
he
adopt
ionofsuppl
ementar
yrul
est
ogover
nadjudicat
oryhear
ings.
14

TheI
ssues

Thei
ssuesf
ort
heCour
t’
sresol
uti
onar
ethef
oll
owi
ng:

1.Whet
herSenatorMadr
igal
,whof
il
edt
hecompl
aintagai
nstSenat
orVi
l
lar
,isani
ndi
spensabl
e
par
tyi
nthi
spetit
ion;

2.Whethert
hepet
it
ioni
spr
emat
uref
orf
ail
uret
oobser
vet
hedoct
ri
neofpr
imar
yjur
isdi
cti
onor
pri
orr
esort;
3.Whethert
het
ransf
erofthecompl
aintagai
nstSenatorVil
larf
rom t
heEt
hicsCommi t
teet
othe
Senat
eCommi t
teeoftheWholei
svi
olati
veofSenatorVil
lar
’sri
ghtt
oequalprot
ect
ion;

4.WhethertheadoptionoftheRul
esoftheEthi
csCommi tt
eeasRulesoftheSenateCommitt
ee
oftheWholeisav i
olati
veofSenat
orVil
l
ar’sri
ghttodueprocessandoft
hemaj or
it
yquorum
requi
rementunderArt.VI,
Secti
on16(
2)oft heConst
it
uti
on;and

5.Whetherpubl
i
cat
ionoft
heRul
esoft
heSenat
eCommi
tt
eeoft
heWhol
eisr
equi
redf
ort
hei
r
eff
ect
ivi
ty.

TheRul
i
ngoft
hisCour
t

I
ndi
spensabl
ePar
ty

Sect
ion7,
Rul
e3oft
he1997Rul
esofCi
vi
lPr
ocedur
epr
ovi
des:

SEC.7–Compulsor
yjoi
nderofi
ndi
spensablepar
ti
es.-Part
iesini
nter
estwithoutwhom nof
inal
det
ermi
nat
ioncanbehadofanacti
onshallbejoi
nedasplai
ntif
fsordefendants.

Thet
estt
odet
ermi
nei
fapar
tyi
sani
ndi
spensabl
epar
tyi
sasf
oll
ows:

Anindispensablepartyisapartywhohasani nt
erestint hecontr
over syorsubj ectmat terthata
fi
naladjudicat
ioncannotbemade, i
nhisabsence, wit
houti nj
uri
ngoraf fecti
ngt hati
nterest,a
part
ywhohasnotonl yaninter
estinthesubj ectmat t
eroft hecontrov ersy,butalsohasan
i
nterestofsuchnat ur
et hatafinal
decreecannotbemadewi thoutaff ecti
nghi sint
erestor
l
eavingthecont r
oversyinsuchacondi ti
ont hatitsfi
nal deter
minati
onmaybewhol l
y
i
nconsistentwithequityandgoodconsci ence.Ithasal sobeenconsi deredt hatan
i
ndispensablepartyisaper soninwhoseabsencet herecannotbeadet er
mi nati
onbet weent he
part
iesalreadybeforethecourtwhichisef f
ect i
ve,compl eteorequitable.Fur t
her,an
i
ndispensablepartyisonewhomustbei ncl
udedi nanact i
onbeforei tmaypr operl
ygof or
war d.

Aper sonwhoi snotani ndi spensablepar t


y,however,
ifhisi
nteresti
nt hecontrov
ersyorsubject
mat t
erisseparabl efr
om t heinter
estoft heotherpart
ies,sothatitwil
lnotnecessari
lybe
di
rectlyorinjur
iouslyaffectedbyadecr eewhichdoescompl etejusti
cebetweent hem.Also,a
personisnotani ndispensabl epartyifhispresencewoul dmerelypermitacompl et
ereli
ef
betweenhi m andt hosealreadypar t
iest otheacti
on,orifhehasnoi nter
estinthesubject
mat t
eroftheact ion.Iti
snotasuf f
icientreasontodeclareapersont obeani ndi
spensable
partythathispresencewi llavoidmul ti
pleli
ti
gati
on.15

Inthiscase,SenatorMadr igalisnotani ndispensablepar tyt


ot hepeti
ti
onbef oretheCourt.
Whi l
eitmaybet ruet hatshehasani nterestintheout comeoft hi
scaseast heauthorofP.S.
Resoluti
on706, theissuesi nthiscasear emat tersofjurisdi
cti
onandpr ocedureont hepartof
theSenat eCommi tteeoftheWhol ewhi chcanber esolvedwithoutaff
ectingSenat orMadri
gal’
s
i
nterest.ThenatureofSenat orMadrigal’sinter
estinthiscasei snotofthenat ur
et hatt
hiscase
couldnotber esolvedwi t
houtherpar ti
cipat i
on.1awphi1
Doct
ri
neofPr
imar
yJur
isdi
cti
on

Respondentasser t
sthatthedoctri
neofpr i
mar yj
uri
sdicti
on"simpl ycal
lsforthedeterminat
ion
ofadmi nist
rat
ivequesti
ons,whichareor di
nari
l
yquest i
onsoff act,byadministr
ati
veagencies
ratherthanbycourtsofjust
ice.
"16Ci t
ingPimentelv
.HRET, 17r espondentaversthatpri
mary
recourseofpetit
ioner
sshouldhav ebeent otheSenateandt hatthisCourtmustuphol dthe
separationofpowersbetweenthel egisl
ati
veandjudici
albranchesoft hegov er
nment .

Thedoct
ri
neofpr
imar
yjur
isdi
cti
ondoesnotappl
ytot
hiscase.TheCour
thasr
uled:

xxxI tmayoccurt hattheCourthasj uri


sdict
iont otakecogni zanceofapar t
icularcase,whi
ch
meanst hatthemat t
erinvolv
edisal sojudi
cial i
nchar acter.Howev er,i
fthecasei ssuchthatit
s
determi nat
ionr equir
est heexpert
ise,special
izedskillsandknowl edgeofthepr oper
admi nistr
ati
vebodi esbecauset echnicalmat ter
sori ntri
catequest ionsoffactareinvolv
ed,then
rel
iefmustf i
rstbeobt ainedinanadmi nist
rativeproceedi ngbef orearemedywi llbesuppli
edby
thecour tsevent hought hemat t
eriswi t
hinthepr operjurisdicti
onoft hecourt.xxx18

Theissuespr esentedheredonotrequi
retheexper
ti
se,speci
ali
zedski
l
lsandknowl edgeof
respondentfortheirresol
uti
on.Onthecontr
ary
,thei
ssuesherearepurel
ylegalquesti
onswhich
arewithi
nthecompet enceandjur
isdi
cti
onoftheCourt
,andnotanadmi ni
strat
iveagencyort
he
Senatetoresolve.19

Asr egardsr espondent’


si nvocat i
onofsepar ati
onofpower s,theCour tr
ei t
erat
esthat"t
he
i
nv i
olatedoct ri
neofsepar ationofpower samongt hel
egi sl
ati
v e,executiveorjudi
cialbranches
ofgov ernmentbynomeanspr escri
besf orabsoluteautonomyi nt hedischargebyeachoft hat
partoft hegov ernmentalpowerassi gnedt oitbythesov erei
gnpeopl e."
20Thus, i
thasbeenhel d
that"thepowerofj udi
cialreviewi snotsomuchpowerasi tis[a]dut yimposedont hi
sCour tby
theConst it
uti
onandt hatwewoul dber emissintheper f
or manceoft hatdut yi
fwedecl i
net o
l
ookbehi ndt hebarri
erssetbyt heprincipl
eofsepar at
ionofpower s.
"21TheCour t
,ther
ef or
e,is
notpr ecludedf r
om resolvingthel egalissuesrai
sedbyt hemer einv ocationbyrespondentof
thedoct rineofseparationofpower s.Ont hecontrary,
ther esolutionoft helegali
ssuesf all
s
withintheexcl usiv
ejuri
sdi ct
ionoft hisCour t
.

Tr
ansf
eroft
heCompl
aintf
rom t
heEt
hicsCommi
tt
ee

t
otheSenat
eCommi
tt
eeont
heWhol
e

Petit
ionersal
legethatt
hetransferofthecompl ai
ntagainstSenat orVil
lartotheSenate
Commi t
teeoftheWholev i
olat
eshi sconsti
tuti
onalr
ighttoequal protecti
on.Petit
ioner
sallege
thattheSenateCommi tt
eeoft heWhol ewasconst i
tutedsolelyforthepurposeofassumi ng
j
ur i
sdict
ionoverthecomplai
ntagai nstSenatorVi
ll
ar.Peti
tionersfurt
herallegethattheactwas
discri
minator
yandr emovedSenat orVil
lar
’srecour
seagai nstanyadv ersereportoftheEthics
Commi t
teetotheSenateasabody .

Wedonotagr
eewi
thpet
it
ioner
s.
Rev i
ewi ngt heev entsthatledt ot heconst i
tutionoft heSenateCommi t
teeoft heWhol e,the
Cour tnot esthatupont heelect i
onofSenat orEnr i
leasSenat ePr esidenton17Nov ember2008,
theEt hicsCommi t
teewasal sor eor ganized.Senat orLacson, whof ir
stcal ledtheSenat e’
s
attentiont otheal l
egedi r
regular i
tiescommi tt
edbySenat orVillar,wasel ect edasChai rper son.
On16December2008, whenSenat orLacsoni nquiredwhet hert heMi nor it
ywasr eadyt oname
theirrepresent ati
vestot heEt hicsCommi ttee, SenatorPiment elinf ormedt hebodyt hatther e
woul dbenomemberf rom theMi nori
tyintheEt hi
csCommi ttee.On26Januar y2009, Senat or
Lacsonr eiter
atedhi sappeal tot heMi nori
tyt onomi natetheirrepr esent at i
vest otheEt hics
Commi t
tee.Senat orPiment elinf ormedhi mt hatitisthestandoft heMi nor i
tynottonomi nat e
anyoft heirmember stot heEt hicsCommi ttee.Senat orPiment el promi sedt oconv eneacaucus
todet ermi neiftheMi nority
’sdeci si
onont hemat terisfi
nalbutt her ecor dsdi dnotshowt hata
caucuswasconv ened.

On20Apr i
l2009, Senat
orVill
ardeli
veredapr ivi
l
egespeechwher ehest at
edt hathewould
answert heaccusationsagainsthi
m ont hefloorandnotbefor et
heEthicsCommi t
tee.I
twas
becauseoft heaccusationthattheEthicsCommi t
teecouldnotactwithfair
nessonSenat or
Vil
lar
’scaset hatSenatorLacsonmov edt hattheresponsi
bil
ityoftheEthi
csCommi tt
eebe
undertakenbyt heSenateactingasaCommi t
teeoftheWhol e,whichmotionwasappr ovedwit
h
tenmember sv oti
ngi nfavor
,noneagai nst
,andf i
veabstenti
ons.

TheRul esoft heEt hicsCommi t


teepr ovidet hat" al
lmat tersrelat
ingtot heconduct ,
rights,
pri
v i
leges,saf ety ,digni t
y,i
ntegri
tyandr eputat i
onoft heSenat eandi t
sMember sshal lbeunder
theexcl usi
v ej urisdictionoft heSenat eCommi tteeonEt hi
csandPr i
vil
eges."22Howev er,i
nthis
case, t
her efusal oft heMi norit
ytonamei tsmember st otheEt hi
csCommi tteest all
edt he
i
nv esti
gat i
on.I nshor t, whi
leordinaril
yani nv esti
gat i
onaboutoneofi tsmember s’ al
leged
i
rregularorunet hical conductiswi thi
nt hej uri
sdi cti
onoft heEt hi
csCommi t
tee,theMi nori
ty
effecti
v el
ypr ev ent edi tfr
om pursuingt hei nvestigationwhent heyrefusedt onomi natetheir
member stot heEt hicsCommi tt
ee.Ev enSenat orVi ll
arcalledtheEt hi
csCommi tt
eeakangar oo
courtanddecl aredt hathewoul danswert heaccusat ionsagainsthim ont hef l
oorandnot
beforet heEt hicsCommi ttee.Gi
v entheci rcumst ances, t
hereferraloftheinvestigationt othe
Commi tteeoft heWhol ewasanext raordinar yremedyunder takenbyt heEt hicsCommi tt
eeand
approv edbyamaj orit
yoft hemember soft heSenat e.

Adopt
ionoft
heRul
esoft
heEt
hicsCommi
tt
ee

byt
heSenat
eCommi
tt
eeoft
heWhol
e

Pet
it
ionersal
leget
hatt
headopti
onoftheRulesoftheEthi
csCommitteebytheSenat
e
Commi t
teeoftheWhol
eisvi
olat
iveofSenat
orVil
l
ar’sr
ighttoduepr
ocess.

Wedonotagr
ee.

Agai
n,wer ei
ter
atethat,
consi
der
ingtheci
rcumst
ancesoft
hiscase,t
her
eferr
aloft
he
i
nvest
igat
ionbyt heEthi
csCommi t
teet
otheSenateCommit
teeoftheWholeisanext
raor
dinar
y
remedyt hatdoesnotvi
olateSenatorVill
ar’
sri
ghttodueprocess.I
nthesamemanner
,the
adoptionbytheSenateCommi tteeoftheWhol eoftheRul
esoft heEt
hicsCommi
tt
eedoesnot
viol
ateSenatorVil
l
ar’
srighttoduepr ocess.

TheConstit
uti
onal
rightoft
heSenat
etopromul
gat
eit
sownr
ulesofpr
oceedi
ngshasbeen
recogni
zedandaff
ir
medbyt hi
sCourt
.Thus:

Fir
st.Secti
on16(
3),Ar
ti
cleVIoft
hePhi
l
ippi
neConst
it
uti
onst
ates:
"EachHouseshal
ldet
ermi
ne
therulesofit
spr
oceedi
ngs."

Thisprovi
sionhasbeent r
adit
ional
lyconstruedasagr antofful
ldiscr
eti
onaryauthori
tyt
ot he
HouseofCongr essi
ntheformulat
ion,adoptionandpr omulgati
onofi t
sownr ul
es.Assuch, t
he
exerci
seoft hispoweri
sgenerall
yexemptf rom judi
cialsupervi
sionandint
erfer
ence,excepton
aclearshowi ngofsucharbi
tr
aryandimpr ovidentuseoft hepoweraswi l
lconsti
tuteadenialof
dueprocess.

xxx.Thei ssuepartakesofthenatur
eofapol i
ti
calquesti
onwhich, undert
heConstituti
on,ist
o
bedecidedbyt hepeopleinthei
rsovereigncapacity
,orinregar
dt owhi chf
ulldi
scr
eti
onar y
authori
tyhasbeendel egat
edtothelegislati
veorexecuti
vebranchoft hegovernment.Furt
her,
pursuanttohisconstit
uti
onalgr
antofv i
rtuall
yunrestr
ict
edauthorit
ytodetermineit
sownr ul
es,
theSenateisatli
bertytoalt
erormodifyt heserul
esatanyt i
mei tmayseef i
t,subj
ectonlytothe
i
mper ati
vesofquorum, voti
ngandpubl i
cation.
23

Theonlyl
imitat
iontothepowerofCongresst opr
omul gat
eit
sownrul
esistheobser
vanceof
quorum,vot
ing,andpubli
cat
ionwhenrequired.Aslongastheser
equi
rementsar
ecomplied
wit
h, t
heCourtwil
lnotint
erf
erewi
ththerightofCongresstoamendit
sownr ul
es.

Pr
iorPubl
i
cat
ion

Peti
ti
onersassail
thenon- publ
icati
onoft heRul
esoft heSenateCommi t
teeoftheWhole.
Respondentcountersthatpubli
cationisnotnecessarybecausetheSenateCommi tt
eeofthe
Wholemer el
yadoptedt heRulesoft heEthi
csCommi tteewhichhadbeenpubl i
shedinthe
Off
ici
alGazetteon23Mar ch2009.Respondental l
egesthatther
eisonlyonesetofRulesthat
gover
nsbot htheEthicsCommi tteeandtheSenat eCommi tt
eeoftheWhol e.

I
nNer iv
.SenateCommi tt
eeonAccount abil
i
tyofPubl
icOff
icersandInvesti
gat
ions,
24theCourt
decl
aredvoi
dunpublishedrul
esofprocedureinSenatei
nquir
iesinsof
arassuchr ul
esaff
ectthe
ri
ght
sofwi t
nesses.TheCourtci
tedSect i
on21,Art
icl
eVIoftheConstit
utionwhichmandates:

Sec.21.TheSenateort
heHouseofRepr esent
ati
vesoranyofi t
srespect
iveCommi tt
eesmay
conductinqui
ri
esi
naidoflegi
slat
ioni
naccordancewit
hi t
sdulypubli
shedrulesofprocedur
e.
Theright
sofpersonsappeari
nginoraf
fectedbysuchinquir
iesshal
lberespected.(
Emphasis
suppli
ed)

TheCour
texpl
ainedi
ntheResol
uti
on25deny
ingt
hemot
ionf
orr
econsi
der
ati
on:
ThelanguageofSect i
on21,Arti
cleVIoft heConst i
tut
ionrequi
ri
ngthatt
heinquir
ybeconducted
i
naccor dancewiththedulypubli
shedr ul
esofprocedur eiscat
egori
cal
.Iti
sincumbentuponthe
Senatetopublishtherul
esofitslegi
slati
veinqui
riesineachCongressorotherwi
semaket he
publ
ishedrulesclear
lyst
atethatthesameshal lbeef fect
ivei
nthesubsequentCongr
essesor
unti
ltheyareamendedorr epealedtosuffici
entl
yputpubl i
connoti
ce.

Ifi
twastheint
enti
onoftheSenatef
orit
spresentrul
esonl
egi
slat
ivei
nquir
iestobeeff
ect
ive
eveni
nthenextCongress,
itcoul
dhaveeasi
lyadoptedt
hesamelanguageithadusedi
nit
s
mainrul
esregardi
ngeff
ecti
vi
ty.

LesttheCour tbemi sconstrued,i


tshoul
dlikewisebest ressedthatnotal lordersi
ssuedor
proceedingsconduct edpursuanttothesubjectRulesarenul landvoid.Onl ythosethatresul
tin
viol
ati
onoft herightsofwi t
nessesshouldbeconsi derednullandv oi
d, consideri
ngthatthe
rati
onaleforthepublicati
oni stoprot
ectt
her i
ghtsoft hewi t
nessesasexpr essedinSect i
on21,
Arti
cleVIoft heConst i
tut
ion.Sanssuchviolat
ion,ordersandpr oceedingsar econsideredval
id
andef f
ectiv
e.26( Emphasissupplied)

I
nther
ecentcaseofGuti
err
ezv
.TheHouseofRepr
esent
ati
vesCommi
tt
eeonJust
ice,
etal
.
,27
t
heCourtf
urt
hercl
ari
fi
ed:

xxxi nquiri
esinaidoflegislat
ionunderSect i
on21, Arti
cleVIoftheConst i
tutioni sthesole
i
nstancei ntheConsti
tuti
onwher ether
ei sacat egori
caldir
ecti
vetodulypubl ishasetofr ul
esof
procedure.Signi
fi
cant
lynot ableinNeriisthatwi t
hrespecttotheissueofpubl i
cation,theCourt
anchoredi t
sruli
ngonthe1987Const i
tuti
on’sdirect
ive,wit
houtanyreli
anceonorr eferenceto
the1986caseofTañadav .Tuv er
a.Tañadanat urall
ycouldneit
herhavei nterpreteda
fort
hcomi ng1987Const i
tutionnorhadkeptat i
ghtr ei
nont heConstit
ution’sintentionsas
expressedt hr
oughtheallowanceofei theracategor i
calter
m oragener alsenseofmaki ng
knownt heissuances.
28

TheConstit
uti
ondoesnotr equi
republi
cationoftheinter
nalrul
esoftheHouseorSenate.Si
nce
rul
esoftheHouseort heSenatethataff
ectonl yt
heirmember sar
einter
nal
totheHouseor
Senate,
suchrulesneednotbepubl i
shed,unlesssuchrulesexpr
essl
yprovi
defort
heir
publi
cat
ionbeforet
her ul
escantakeeffect.

I
nt hi
scase,t
heproceedingsbef
oretheSenateCommi t
teeoftheWhol
eaf f
ectonl
ymembersof
theSenatesi
ncetheproceedi
ngsinvol
vetheSenate’
sexerci
seofit
sdi
scipl
inar
ypowerover
oneofitsmembers.Clearl
y,t
heRulesoftheSenat
eCommi t
teeoft
heWholeareint
ernal
tothe
Senate.However
,Sect
ion81,Rule15oftheRulesoftheSenateCommit
teeoftheWhole
provi
des:

Sec.81.EFFECTI
VITY.TheseRulesshal
lbeef
fect
iveaf
terpubl
i
cat
ioni
ntheOf
fi
cial
Gazet
teor
i
nanewspaperofgener alci
rcul
ati
on.
29

Hence,
int
hispar
ti
cul
arcase,
theRul
esoft
heSenat
eCommi
tt
eeoft
heWhol
eit
sel
fpr
ovi
det
hat
theRulesmustbepubl i
shedbeforetheRulescantakeef f
ect.Thus, ev enifpublicati
oni snot
requir
edundert heConstit
uti
on,publi
cati
onoftheRul esoftheSenat eCommi t
teeoft heWhol e
i
sr equir
edbecauset heRulesexpresslymandatetheirpubl
icat
ion.Themaj orit
yoft hemember s
oftheSenat eapprovedtheRulesoftheSenateCommi tt
eeoft heWhol e,andt hepubl icati
on
requir
ementwhi chtheyadoptedshouldbeconsider edasthewi l
loft hemaj ori
ty.Respondent
cannotdispensewi t
hthepublicat
ionrequi
rementjustbecauset heRul esoft heEthics
Commi tteehadalreadybeenpublishedintheOffi
cialGazett
e.Tor eiterate,theRul esoft he
SenateCommi t
teeoftheWhol eexpressl
yrequir
epubl i
cati
onbef oret heRul escant akeef fect
.
Tocompl ywit
hduepr ocessrequir
ements,theSenat emustfoll
owi tsowni nternal r
ulesifthe
ri
ghtsofi t
sownmember sareaf f
ected.

Inci
dentall
y,wenot ethatSecti
on4, Rul e1oft heRul esoft heSenat eCommi t
teeoft heWhol e30
i
sanexactr eproduct i
onofSect i
on4, Rule1oft heRul esoftheSenat eCommi tteeonEt hics
andPr i
vil
eges31whi chstatesthattheEt hicsCommi t
teeshallbecomposedofsev enmember s,
contrar
yt othef actthattheSenat eCommi tteeoft heWhol econsistsofall
member softhe
Senate.Inaddi t
ion,Secti
on5( B),Rule1oft heRul esoft heSenateCommi tt
eeoft heWhol e32i s
anexactr eproductionofSection5( B),Rule1oft heRul esoftheSenat eCommi tteeonEt hics
andPr i
vil
eges33whi chstatesthatonl ytwomember soft heEthicsCommi tt
eeshal lconstitutea
quorum, contrarytorespondent’sallegati
oni nitsCommentt hateightmember soft heSenat e
Commi tteeoft heWhol eshallconst i
tuteaquor um. 34

Howev er,ift
heSenat eisconsti
tutedasaCommi tteeoftheWhol e,amajor i
tyoftheSenat eis
requi
redt oconsti
tuteaquor um todobusinesspur suanttoSecti
on16( 2),Art
icleVIoft he
Constit
ution.
35Ot herwise,t
herewi l
lbeacircumv enti
onofthisexpressprovisionoft he
Constit
utiononquor um requir
ement .Obvi
ously
, t
heRul esoftheSenateCommi tt
eeoft he
Wholer equir
emodi fi
cationtocompl ywit
hrequirement sofquorum andv oti
ngwhi chtheSenate
musthav eov er
lookedint hi
scase.Inanyev ent
,incaseofconf l
ictbetweent heRul esofthe
SenateCommi t
teeoft heWhol eandt heConsti
tution,
thelat
terwillofcoursepr evail
.

WHEREFORE, weGRANTt hepet


iti
oninpar
t.Ther
eferr
aloft
hecompl ai
ntbytheCommi t
teeon
Ethi
csandPriv
ilegestot
heSenateCommi t
teeoft
heWhol eshal
ltakeeff
ectonlyupon
publ
icat
ionoftheRulesoftheSenateCommitt
eeoftheWhole.

SOORDERED.

ANTONI
OT.CARPI
O

Associ
ateJust
ice

WECONCUR:
G.
R.No.204605

I
NTELLECTUALPROPERTYASSOCI
ATI
ONOFTHEPHI
LIPPI
NES,
Pet
it
ioner
,

v
s.

HON.PAQUITOOCHOA,I
NHISCAPACITYASEXECUTIVESECRETARY,HON.ALBERTDEL
ROSARI
O,INHISCAPACI
TYASSECRETARYOFTHEDEPARTMENTOFFOREI GNAFFAIRS,
AND
HON.RI
CARDOBLANCAFLOR,INHI
SCAPACITYASTHEDI RECTORGENERALOFTHE
I
NTELLECTUALPROPERTYOFFI
CEOFTHEPHILIPPI
NES,Respondent
s.

DECISION

BERSAMI
N,J.
:

I
nt hi
sspeci alci
vilacti
onf orcert
iorar
iandpr ohibi
ti
on,theInt
ell
ectualPropertyAssociati
onof
thePhili
ppines(IPAP)seekst odecl ar
et heaccessionofthePhili
ppinestothePr otocolRelati
ng
totheMadr idAgr eementConcer ningtheInternati
onalRegist
rat
ionofMar ks( MadridProtocol)
unconstit
utionalont hegroundoft helackofconcur r
encebyt heSenate,andi nthealter
native,
todeclaretheimpl ementationthereofasunconst i
tut
ionalbecauseitconfli
ctswithRepubl i
cAct
No.8293, otherwiseknownast heIntel
lectualProper
tyCodeoft hePhili
ppines(IPCode) .
1

Wefi
ndanddeclarethatt
hePresi
dent'
srati
fi
cat
ionisvali
dandconst
it
uti
onalbecauset
he
Madr
idProt
ocol
,beinganexecuti
veagreementasdeterminedbyt
heDepart
mentofForeign
Af
fai
rs,
doesnotrequir
etheconcurr
enceoftheSenate.

Ant
ecedent
s

TheMadr i
dSy stem fortheInternati
onalRegistr
ati
onofMar ks(Madri
dSy stem),whichi
sthe
centr
ali
zedsystem pr ov
idi
ngaone- stopsol
ut i
onforregi
steri
ngandmanagi ngmar ksworl
dwide,
al
lowsthetrademar kownert of i
leoneapplicati
oninonelanguage,andtopayonesetoff eesto
prot
ecthismar kintheterrit
oriesofupt o97member -
stat
es.2TheMadr i
dSy stem i
sgoverned
bytheMadr i
dAgr eement ,concludedin1891, andtheMadr i
dProt
ocol,concludedin1989.
3

TheMadr i
dPr ot
ocol,whi
chwasadoptedi
nor dertoremovet
hechal lengesdeterr
ingsome
count
riesf r
om accedingtotheMadri
dAgreement ,
hastwoobject
ives, namely:
(1)tofaci
li
tat
e
secur
ingpr otect
ionformarks;and(
2)tomaket hemanagementoft her egi
ster
edmar kseasi
er
i
ndiff
er entcountri
es.
4

In2004;theInt
ellectualPropertyOff
iceofthePhil
ippines(I
POPHL),t
hegovernmentagency
mandatedt oadmi nist
erthei nt
ell
ectualpr
oper
tysy st
em ofthecountr
yandtoimplementthe
stat
epolici
esoni ntell
ectualpropert
y ;
beganconsi
der i
ngthecountr
y'
saccessiontot
heMadr id
Prot
ocol.Howev er,basedoni t
sassessmentin2005, theI
POPHLneededt ofi
rsti
mproveits
ownoper at
ionsbef oremaki ngtherecommendationinf av
orofaccessi
on.TheIPOPHLthus
i
mpl
ementedref
ormstoel
imi
nat
etr
ademar
kbackl
ogsandt
oreducet
het
urnar
oundt
imef
or
t
her
egi
str
ati
onofmarks.
5

Inthemeanwhi l
e,t
heIPOPHLmount edacampai gnf ori
nfor
mationdisseminati
ont oraise
awarenessoftheMadr i
dPr otocol.I
tlaunchedaser i
esofconsult
ati
onswi thstakeholdersand
vari
ousbusinessgroupsregardingthePhi l
ippi
nes'accessi
ontotheMadr idProtocol
.It
ult
imatel
yarri
vedattheconclusiont hataccessi
onwoul dbenefi
tthecountryandhel praisethe
l
evelofcompet i
ti
venessforFili
pinobrands.Hence, i
trecommendedinSept ember2011t othe
Depart
mentofFor ei
gnAf f
airs(DFA)t hatthePhil
ippi
nesshouldaccedet otheMadr i
dPr otocol
.6

Afteri
tsownr evi
ew, theDFAendor sedtot hePresidentthecount r
y'saccessiontotheMadrid
Protocol.Conf
ormabl ywit
hi t
sexpressauthor i
tyunderSection9ofExecut i
veOr derNo.459
(Provi
dingfortheGuideli
nesi nt
heNegot i
at i
onofI nt
ernati
onal Agreement sanditsRati
fi
cat
ion)
datedNov ember25, 1997,theDFAdet er
mi nedt hattheMadr i
dPr otocolwasanexecut i
ve
agreement .
1âwphi1TheI POPHL, t
heDepar t
mentofSci enceandTechnol ogy,andthe
DepartmentofTr adeandI ndust
ryconcurredi ntherecommendat i
onoft heDFA. 7

OnMar ch27,
2012, Presi
dentBenignoC.AquinoII
Irati
fi
edt
heMadri
dProt
ocolt
hroughan
i
nstr
umentofaccessi on,Theinst
rumentofaccessi
onwasdeposi
tedwi
ththeDi
rect
orGener
al
oft
heWor ldI
ntel
lectualPropert
yOrgani
zati
on(WIPO)onApril
25,
2012.
8TheMadr i
dProt
ocol
ent
eredint
oforceinthePhi l
ippi
nesonJuly25,2012.9

Petit
ionerIPAP, anassociat ionofmor ethan100l awfir
msandi ndi
vi
dual pr
act i
ti
onersin
I
ntellectualPropertyLawwhosemai nobj ect
ivei stopromot eandprotectint
ellectualpropert
y
ri
ghtsi nthePhi l
ippinesthroughconst antassistanceandi nvolvementinthelegislati
onof
i
ntell
ect ualpr
oper tylaw,10hascommencedt hi sspecialciv
ilacti
onforcerti
orariand
prohibit
ion11t ochal l
enget hev al
i
dit
yoft hePr esident'
saccessiontotheMadr idPr otocol
withoutt heconcur r
enceoft heSenate.Citi
ngPi ment el
,Jr.v.Offi
ceoftheExecut i
v eSecretar
y,
theIPAPhasav err
ed:

Nonethel
ess,whil
ethePr esidenthasthesol
eaut horitytonegot i
ateandenterint
ot r
eaties,
the
Consti
tut
ionprovi
desal imitati
ontohispowerbyr equir i
ngt heconcur r
enceof2/3ofal lthe
member softheSenatef orthevali
dit
yofthetreatyent eredintobyhi m.Secti
on21, Arti
cleVIIof
the1987Constit
uti
onpr ovidesthat"notr
eatyorinternational agreementshal
lbev ali
dand
eff
ecti
veunlessconcurredinbyatl eastt
wo-thi
rdsofal ltheMember softheSenate."The1935
andthe1973Const i
tut
ional sorequir
edtheconcur r
encebyt helegisl
atur
etothetreati
es
enter
edintobytheexecut i
ve.12

AccordingtotheI
PAP,theMadridProt
ocoli
sat r
eat
y,notanexecut
iveagreement;
hence,
respondentDFASecr
etaryAlber
tDelRosar
ioactedwit
hgraveabuseofdiscret
ioni
n
determini
ngtheMadri
dPr ot
ocolasanexecut
iveagr
eement.13

TheIPAPhasar
guedt hatt
hei
mplementat
ionoft
heMadri
dProt
ocoli
nthePhi
li
ppi
nes;
speci
fi
cal
l
ytheprocessi
ngoff
orei
gntr
ademarkappl
i
cati
ons,
confl
i
ctswit
htheI
PCode,14
whoseSect
ion125st
ates:

Sec.125.Repr esentat
ion;Addr essf orService.-Iftheapplicantisnotdomi ci
ledorhasnor eal
andeffecti
vecommer cialestablishmenti nt hePhili
ppines;heshall designatebyawr i
tt
en
documentf i
ledint heoffi
ce,thenameandaddr essofaPhi li
ppineresidentwhomaybeser ved
noti
cesorpr ocessi nproceedingsaf f
ecti
ngt hemar k.Suchnoticesorser v i
cesmaybeser ved
upontheper sonsodesi gnatedbyl eavingacopyt hereofattheaddr essspeci fiedinthelast
desi
gnationfil
ed.Iftheper sonsodesi gnatedcannotbef oundatt headdr essgi veni
nthelast
desi
gnation,suchnot i
ceorpr ocessmaybeser vedupont heDirector.(Sec.3; R.A.No.166a)

I
thasposi
tedt
hatAr
ti
cle2oft
heMadr
idPr
otocol
prov
idesi
ncont
rast
:

Ar
ti
cl
e2

Secur
ingPr
otect
iont
hroughI
nter
nat
ional
Regi
str
ati
on

(1)Wher eanappl i
cat i
onf ort heregist
rat i
onofamar khasbeenf il
edwi ththeOf ficeofa
Cont racti
ngPar ty,orwher eamar khasbeenr egi steredint her egisteroftheOf ficeofa
Cont racti
ngPar ty,theper soni nwhosenamet hatappl i
cation( hereinaft
erreferredt oas" the
basicappl i
cation;'
)ort hatr egist
rati
on( herei
naf terr efer
redt oas" thebasicregist ration")stands
may ,subjecttot hepr ovisionsoft hi
sPr otocol secur eprotectionf orhismar kint het err
itoryof
theCont r
act i
ngPar ti
es,byobt ai
ningther egistrationoft hatmar ki ntheregisteroft he
I
nt ernati
onal Bur eauoft heWor l
dIntel
lectual Proper tyOr ganization( her
einafterr eferredt oas
"theinternat i
onal registr
ation,""t
heI nt
er nati
onal Regi ster,
"" t
heI nternati
onal Bureau"and" the
Or ganization'
',respect i
vely),provi
dedt hat,

(i
)wherethebasicappl
icat
ionhasbeenf il
edwiththeOffi
ceofaCont racti
ngStateorwherethe
basicr
egist
rati
onhasbeenmadebysuchanOf fi
ce,thepersoni
nwhosenamet hatappl
i
cati
on
orregi
str
ati
onstandsisanationalofthatCont
ract
ingState,ori
sdomi cil
ed,orhasarealand
eff
ecti
veindustr
ial
orcommer cial
establi
shment,i
nthesaidContract
ingSt at
e,

(i
i
)wher et
hebasi capplicationhasbeenfil
edwi ththeOffi
ceofaContract
ingOr gani
zat
ionor
wheret hebasicregist
rationhasbeenmadebysuchanOf fi
ce,t
hepersoninwhosenamet hat
appli
cationorregist
rati
onst andsisanati
onal ofaStat
ememberoft hatCont r
acting
Organizati
on,orisdomi ciled,orhasarealandef f
ecti
veindust
ri
alorcommer ci
al est
abl
i
shment ,
i
nt heterri
tor
yoft hesaidCont r
acti
ngOrganizati
on.

(2)Theappl icati
onf ori
nter
nationalregi
str
ati
on( herei
naft
errefer
redtoas"t
heinter
nati
onal
applicati
on")shallbef i
l
edwi t
ht heInter
nat
ionalBureauthroughtheint
ermediar
yoftheOffi
ce
withwhi chthebasi cappli
cati
onwasf il
edorbywhi chthebasicregi
str
ati
onwasmade
(hereinaft
erreferr
edt oas"theOf f
iceofori
gin")
,ast hecasemaybe.

(3)Anyref
erencei
nt hisPr
otocolt
oan" Off
ice"oran"Offi
ceofaContr
acti
ngParty
"shall
be
constr
uedasar ef
erencetotheoffi
cethati
si ncharge,
onbehalfofaCont
ract
ingPart
y,oft
he
regi
str
ati
onofmar ks,andanyref
erenceinthisProt
ocolto"mar
ks"shal
lbeconstr
uedasa
r
efer
encet
otr
ademar
ksandser
vicemar
ks.

(4)ForthepurposesofthisProt
ocol,"
terri
toryofaContr
acti
ngParty
"means,
wherethe
Contracti
ngPartyisaState,
theter
ri
toryoft hatSt
ateand,wher
etheContr
act
ingPart
yisan
i
ntergovernmentalor
ganizat
ion,
theterri
toryinwhicht
heconsti
tut
ingtr
eat
yofthat
i
ntergovernmentalor
ganizat
ionappl
ied.

TheIPAPhasi nsist
edt hatArti
cle2oftheMadr i
dProtocolmeansthatforei
gntrademark
appl
icantsmayf i
letheirappl
icati
onsthroughtheInt
ernat
ionalBur
eauort heWIPO, andt
hei
r
appl
icati
onswillbeaut omati
call
ygrantedtrademarkprot
ecti
onwithouttheneedfor
desi
gnatingt
heirresidentagentsinthecountry.
15

Moreover,t
heIPAPhassubmit
tedthattheprocedureoutl
inedi
nt heGui
det
otheI
nter
nati
onal
Regi
strat
ionofMarksr
elat
ingt
orepresentat
ionbeforetheInt
ernati
onal
Bur
eaui
sthefol
l
owing,
towit
:

Rule3(1)
(a)09. 02Ref erencesint heRegul at
ions,Administr
ati
veInstructionsori
nthisGui det o
repr
esentationr elateonlytor epresent at
ionbeforetheInter
nati
onal Bureau.Thequestionsof
theneedf orar epr esentati
vebef oret heOffi
ceofor i
ginortheOf f
iceofadesi gnat
ed
Contract
ingPar ty( forexampl e,i
nt heeventofar ef
usalofprotecti
oni ssuedbysuchanOf f
ice),
whomayactasar epresentativ
ei nsuchcasesandt hemet hodofappoi ntment
,areoutsidet he
scopeoft heAgr eement ,Protocol andRegulationsandar egovernedbyt hel
awandpr acticeof
theContractingPar t
yconcer ned.

whichpr
ocedur
eisi
nconfl
ictwi
ththatunderSect
ion125oftheI
PCode,
andconst
it
utesi
n
ef
fectanamendmentoft
helocall
awbyt heExecuti
veDepar
tment
.16

TheIPAPhaspr ayedthattheimplement
ati
onoftheMadridProt
ocol
inthePhil
i
ppinesbe
rest
rai
nedinordertopreventfut
urewrongsconsi
deri
ngthatt
heIPAPanditsconsti
tuency
haveaclearandunmistakableri
ghtnottobedepri
vedofther
ight
sgrant
edthem bytheIPCode
andexist
inglocal
laws.17

Initscommenti nbehalfofther espondent s,t


heOf fi
ceoft heSolici
torGener al(OSG)hasst at
ed
thatt heIPAPdoesnothav ethel ocusst andit
ochal l
enget heaccessiont otheMadr i
dPr ot ocol
;
thatt heIPAPcannoti nvoketheCour t
'sorigi
nalj
ur i
sdicti
onabsentashowi ngofanygr ave
abuseofdi screti
onont hepartoft herespondents;thatthePr esi
dent'srati
fi
cationoftheMadr i
d
Protocol asanexecut i
veagreementi svalidbecauset heMadr idProtocolisonlyprocedur al,
doesnotcr eatesubst anti
verights,anddoesnotr equiretheamendmentoft heI PCode;t hatthe
IPAPi snotentitl
edtot herestraini
ngor derorinj
unct i
onbecausei tsuffersnodamagef rom t he
rati
ficati
onbyt hePresident,andt hereisalsonour gencyf orsuchreli
ef;andt heIPAPhasno
clearunmi stakablerighttother eli
efsought .
18

I
ssues

Thef
oll
owi
ngi
ssuesar
etober
esol
ved,
namel
y:
I
.WhetherornottheI
PAPhasl
ocusst
andi
tochal
l
enget
hePr
esi
dent
'sr
ati
fi
cat
ionoft
he
Madr
idProtocol
;

II
.Whet
herornott
hePr
esi
dent
'
srat
if
icat
ionoft
heMadr
idPr
otocol
isv
ali
dandconst
it
uti
onal
;
and

I
II
.Whet
herornott
heMadr
idPr
otocol
isi
nconf
li
ctwi
tht
heI
PCode.

Rul
i
ngoft
heCour
t

Thepet
it
ionf
orcer
ti
orar
iandpr
ohi
bit
ioni
swi
thoutmer
it
.

A.

Thei
ssueofl
egal
standi
ngt
osue,
orl
ocusst
andi

TheIPAPar guesi nit


srepl
y19thatithasthel
ocusstanditofil
ethepr esentcasebyvir
tueofit
s
beinganassoci at
ionwhosemember sstandtobeinj
uredasar esultoftheenforcementofthe
Madr i
dProtocol i
nthePhil
ippi
nes;thatt
heinj
urypert
ainstotheaccept anceandapprovalof
appli
cati
onssubmi t
tedthr
oughtheMadr idPr
otocolwit
houtlocalrepresentat
ionasrequi
redby
Secti
on125oft heIPCode;20andthatsuchwi l
ldi
mini
shther i
ghtsgr antedbytheIPCodeto
Int
ell
ectual
Pr opertyLawpract
it
ionersli
kethemember softheIPAP. 21

Thear
gumentoft
heI
PAPi
sunt
enabl
e.

Legalst
andingr eferst o"arightofappearanceinacourtofjusti
ceonagi venquestion."
22
Accordi
ngt oAgan, Jr.v.Phil
ippi
neInternat
ionalAi
rTerminalsCo.,
Inc.
,23standi
ngis" apecul
iar
concepti
nconst itutionallawbecausei nsomecases, sui
tsarenotbroughtbypar t
ieswhohav e
beenpersonall
yi njuredbyt heoperationofalaworanyot hergovernmentactbutbyconcer ned
ci
ti
zens,taxpayersorv oterswhoact ual
lysueinthepubli
cinter
est.
"

TheCour thasfrequentl
yfel
ttheneedtodwel lontheissueofstandinginpubli
cor
consti
tuti
onalli
ti
gati
onst osif
ttheworthyfrom t
heunwor thypubli
clawlit
igant
sseeki
ng
redr
essorr el
ief
.Thef ol
lowingeluci
dat
ioninDeCast rov.Judici
alandBarCouncil
24off
ersthe
generalunder
standingofthecont ex
toflegalst
anding,orlocusstandif
orthatpur
pose,vi
z.:

Inpubl i
corconst i
tut
ionalli
ti
gations,theCour tisoft
enbur denedwi ththedeterminati
onoft he
l
ocusst andiofthepeti
tioner
sduet otheev er-
presentneedt oregulatetheinvocati
onoft he
i
ntervent i
onoftheCour ttocorrectanyof f
icialacti
onorpol i
cyinor dertoavoidobstructingthe
effi
cientfuncti
oningofpubl i
coff i
cial
sandof ficesi
nv ol
vedinpublicser vi
ce.I
tisrequired,
therefore,t
hatthepetit
ionermusthav eaper sonalstakeintheout comeoft hecont r
over sy,f
or,
asindicatedinAgan, Jr.v.Phil
i
ppi neInt
ernationalAirTerminalsCo.,Inc.
:

Thequestiononlegalst
andingiswhet
hersuchpar t
ieshave"'all
egedsuchapersonalstakei
n
theoutcomeofthecontroversyast
oassurethatconcreteadv er
senesswhichsharpensthe
present
ati
onofissuesuponwhi cht
hecourtsolargelydependsf ori
l
luminat
ionofdiff
icul
t
constit
utionalquesti
ons,"Accordingl
y, i
thasbeenhel dthattheinter
estofapersonassaili
ng
theconst i
tuti
onali
tyofast at
utemustbedi rectandpersonal.Hemustbeabl etoshow, notonly
thatthelaworanygov ernmentacti sinval
id,butalsothathesustainedorisini
mmi nentdanger
ofsustainingsomedi recti
njuryasar esultofitsenfor
cement ,andnotmer el
ythathesuffer
s
therebyinsomei ndefini
teway .Itmustappeart hatthepersoncompl aini
nghasbeenori sabout
tobedeni edsomer i
ghtorprivil
egetowhi chhei slawful
lyenti
tledorthatheisabouttobe
subjectedtosomebur densorpenal ti
esbyr easonofthest at
uteoractcompl ai
nedof.

I
tist r
uethatasear l
yasin1937, i
nPeoplev .Vera,
theCour tadoptedt hedirectinjur
yt estfor
determini
ngwhet herapet i
ti
oneri
napublicactionhadlocusst andi.There, t
heCour thel dthat
thepersonwhowoul dassailthev
ali
dit
yofast atut
emusthav e"aper sonal andsubst antial
i
nterestinthecasesucht hathehassustained,orwil
lsustaindir
ectinjuryasar esul
t."Ver awas
fol
lowedi nCustodiov.Presi
dentoftheSenate,Manil
aRaceHor seTr ainers'Associationv .De
l
aFuent e,Anti
-ChineseLeagueofthePhil
ippinesv.Fel
ix,andPascual v.SecretaryofPubl ic
Wor ks.

Yet,theCour thasalsoheldthattherequir
ementoflocusstandi,bei
ngamer epr ocedural
technical
ity,
canbewai vedbytheCour ti
ntheexer
ciseofit
sdi scret
ion.Forinstance,i
n1949, i
n
Aranetav .Dingl
asan, t
heCourtli
beral
izedtheappr
oachwhent hecaseshad" t
ranscendental
i
mpor t
ance."Somenot abl
econtr
ov er
sieswhosepetit
ioner
sdidnotpasst hedirectinj
urytest
wereal l
owedt obet reatedi
nthesamewayasi nAranetav.Dinglasan.

Inthe1975deci sioninAqui nov.Commi ssi


ononElections, t
hisCourtdecidedtoresolvethe
i
ssuesr aisedbyt hepetit
ionduetot hei
r" f
arr
eachi
ngi mpl i
cati
ons,
';evenifthepetit
ionerhadno
personalit
ytofil
et hesuit.Theli
beralapproachofAquinov .Commi ssi
ononEl ecti
onshasbeen
adopt edinseveralnotablecases,permitti
ngordi
narycitizens,l
egi
slator
s,andci v
ic
organizati
onstobr ingtheirsui
tsi
nv ol
vingtheconst
itutionali
tyorvali
dit
yofl aws,r
egulati
ons,
andr uli
ngs.

Howev er,t
heasserti
onofapubl i
cr i
ghtasapr edicat
eforchall
engingasupposedl yill
egalor
unconstit
uti
onalexecuti
veorl egisl
ati
veacti
onr estsonthetheorythatthepet i
ti
onerrepresents
thepubli
cingeneral.Alt
houghsuchpet i
ti
onermaynotbeasadv ersel
yaffectedbytheact ion
complainedagainstasar eothers,i
tisenought hathesuff
ici
entl
ydemonst r
atesinhispet i
tion
thatheisenti
tl
edt oprotecti
onorr el
ieffr
om theCour ti
nthevindicati
onofapubl i
cright.
25

Thei njurythatt heI PAPwi l


lal
legedlysuf ferf
rom theimpl ement ati
onoft heMadr idPr otocoli
s
i
magi nary,
incident al andspeculativeasopposedt oadi rectandmat eri
al i
njuryrequiredbyt he
foregoingtenet sonl ocusstandi.Addi t
ionall
y,ast
heOSGpoi ntsoutinthecomment ,26t he
I
PAPhasmi sinterpr etedSecti
on125oft heIPCodeont hei ssueofr epresentation.The
provisiononl yst at
est hataforeigntrademar kapplicant"shall desi
gnatebyawr i
ttendocument
fi
ledint heof fi
ce, thenameandaddr essofaPhi li
ppineresi dentwhomaybeser vednot i
cesor
processi npr oceedi ngsaf fect
ingthemar k;"i
tdoesnotgr antany oneinpar ti
cularther ightto
representthef orei
gnt rademarkappl icant.Hence,theIPAPcannotj ustl
ycl ai
mt hatitwi l
lsuff
er
i
rr
eparabl
einj
uryordi
minut
ionofr
ight
sgrant
edt
oitbySect
ion125oft
heI
PCodef
rom t
he
i
mplementati
onoftheMadri
dProt
ocol.

Nonet heless,theIPAPalsoemphasi zest hatthepar amountpubl i


cinterestinvolvedhas
transcendent ali
mpor t
ancebecausei tspet i
ti
onasser tsthattheExecut i
veDepar t
menthas
ov er
steppedt heboundsofi tsauthoritybytherebycut ti
ngintoanotherbr anch'sfuncti
onsand
responsi bi
li
ti
es.27Theasser t
ionoft heIPAPmaybev ali
dont hi
sscor e.Therei sli
tt
lequesti
on
thatthei ssuesraisedherei
nagainstt heimplement ationoftheMadr i
dPr otocolareof
transcendent ali
mpor t
ance.Accordingly,wer ecognizeIPAP'slocusstandi tobr i
ngthepresent
challenge.Indeed, t
heCourthasadopt edaliberalatti
tudetowardslocusst andi wheneverthe
i
ssuepr esentedforconsider
ationhast r
anscendent alsi
gnifi
cancetot hepeopl e,orwhenever
theissuesr aisedareofparamounti mpor tancet othepubl i
c.28

B.

Accessi
ont
othe

Madr
idPr
otocol
wasconst
it
uti
onal

TheIPAPsubmi tst hatrespondentsExecut i


veSecr
etaryandDFASecr et
aryDel Rosari
ogr avel
y
abusedt hei
rdi
scr eti
oni ndeterminingthatther
ewasnoneedf orthePhil
ippineSenate'
s
concurrencewitht heMadr i
dPr otocol;t
hattheMadridProtocoli
nv ol
veschangesofnat ional
poli
cy,anditsbeingofaper manentchar acterr
equi
restheSenate'sconcurrence,29pursuantto
Secti
on21, Art
icl
eVI IoftheConst i
tut
ion,whichst
atesthat"notreatyori
nternati
onalagreement
shal
lbev al
i
dandef fect
iveunlessconcur r
edinbyatleasttwo-t
hirdsofallt
heMember soft he
Senate."

Befor
egoingfurt
her,
wehavetodisti
nguishbetweentreat
iesandi
nternati
onal
agr eements,
whichrequi
ret
heSenate'
sconcur
rence,ononehand, andexecut
iveagreements,whichmaybe
val
idl
yenter
edintowit
houtt
heSenate'sconcurr
ence.Executi
veOrderNo.459,Seriesof
1997,
30notesthefol
lowi
ngdefi
nit
ions,t
owi t
:

Sec.2.Def
ini
ti
onofTer
ms.

a.I
nternati
onal agreement-shal
lrefertoacontractorunderst
anding,
regar
dlessof
nomencl at
ure,enteredint
obetweent hePhi
li
ppinesandanot hergover
nmentinwrit
tenf
orm and
governedbyi nt
ernational
law,whetherembodiedinasi ngl
einstr
umentorintwoormore
rel
atedinstr
ument s.

b.Treat
ies-i
nternati
onalagr
eementsent er
edint
obythePhi
li
ppi
neswhi
chrequi
relegi
slat
ive
concurr
enceafterexecuti
verat
ifi
cat
ion.Thi
sterm mayi
ncl
udecompact
sli
keconventi
ons,
decl
arati
ons,
cov enantsandacts.

c.Execut
iveAgr
eement
s-si
mil
art
otr
eat
iesexceptt
hatt
heydonotr
equi
rel
egi
slat
ive
concurr
ence.
TheCourthashighl
ightedt
hedi f
ferencebet
weentreat
iesandexecut
iveagr
eement
sin
CommissionerofCustomsv .EasternSeaTradi
ng,
31thusl
y:

I
nternati
onalagreementsinvolv
ingpol i
ti
cali
ssuesorchangesofnationalpoli
cyandt hose
i
nvolvingint
ernati
onalarr
angement sofaper manentcharact
erusual
lytakethef orm oftreat
ies.
Butinter
nati
onal agr
eement sembody ingadj
ustmentsofdetail
carr
yingoutwel l
-establi
shed
nati
onal pol
i
ciesandt r
adit
ionsandt hoseinv
olvi
ngarrangementsofamor eorlesst emporary
natureusuall
ytaketheform ofexecutiveagr
eements.

InthePhili
ppines,theDFA, byvi
rtueofSecti
on9,Executi
veOrderNo.459,32isiniti
all
ygiv
enthe
powert odeterminewhet heranagreementistobetreat
edasat reatyorasanexecut iv
e
agreement .Todet er
minetheissueofwhetherDFASecr et
aryDelRosariogravel
yabusedhi s
discret
ioninmaki nghisdeterminat
ionrel
ati
vetotheMadr i
dProtocol,
wer evi
ewt he
j
ur i
sprudenceont henatureofexecuti
veagreements,aswellasthesubjectmatterstobe
coveredbyexecut iveagr
eement s.

Thepr
onouncementi
nCommi
ssi
onerofCust
omsv
.East
ernSeaTr
adi
ng33i
sinst
ruct
ive,
towi
t:

xxxTheconcur renceofsaidHouseofCongressisrequi
redbyourfundamental
lawint he
makingof"tr
eati
es"(Consti
tut
ionoft
hePhil
ippines;Ar
ti
cleVI
I,
Secti
on10[7])
,whichare,
however
,disti
nctanddiff
erentfr
om "
execut
iveagreements,
"whichmaybev al
idl
yenteredint
o
wit
houtsuchconcur r
ence.

"Tr
eat
iesareformaldocument
swhichr
equir
erati
ficat
ionwit
htheapprov
aloftwothi
rdsoft
he
Senat
e.Executi
veagreement
sbecomebindi
ngthroughexecuti
veact
ionwit
houtt
heneedofa
vot
ebytheSenat eorbyCongr
ess.

xxxx

"xxxtherightoftheExecut ivetoent eri


ntobindingagreementswithoutt
henecessityof
subsequentCongr essi
onal approvalhasbeenconf i
rmedbylongusage.From theearl
iestday
s
ofourhist
orywehav eenteredi nt
oexecut i
veagr eementscover
ingsuchsubjectsas
commer ci
alandconsul arrelati
ons,most -
favored-nati
onri
ghts,
patentri
ghts,
trademarkand
copyri
ghtprotect
ion,postal andnavigati
onarrangement sandthesettl
ementofclaims.The
val
idi
tyofthesehasnev erbeenser i
ouslyquestionedbyourcour t
s.

xxxx

Agreementswit
hrespectt
otheregi
strati
onoftr
ademarkshav
ebeenconcludedbythe
Execut
ivewit
hvari
ouscountr
iesundertheActofCongr
essofMarch3,1881(21Stat.502)
,xx
x

xxxx

I
nthi
sconnect
ion,
Franci
sB.Say
re,
for
merU.
S.Hi
ghCommi
ssi
onert
othePhi
l
ippi
nes,
sai
din
hi
swor
kon"
TheConst
it
uti
onal
i
tyofTr
adeAgr
eementAct
s":

Agr eement sconcl udedbyt hePr esi dentwhi chf allshor toft reat iesar ecommonl yref erredt oas
execut iv eagr eement sandar enol esscommoni nourschemeofgov ernmentt hanar et hemor e
formal inst rument s-t reati
esandconv entions.Theysomet imest aket hef orm ofexchangesof
not esandatot hert imest hatormor efor mal document sdenomi nated' agreement s' or
'
pr otocol s'.Thepoi ntwher eor dinarycor respondencebet weent hisandot hergov ernment sends
andagr eement s-whet herdenomi nat edexecut i
v eagr eement sorexchangesofnot esor
other wi se-begi n, maysomet imesbedi fficultofr eadyascer tainment .I
twoul dbeusel esst o
under taket odiscussher et helar gev ar i
etyofexecut i
v eagr eement sassuch, concl
udedf rom
ti
met ot i
me.Hundr edsofexecut iveagr eement s, othert hant hoseent eredi nt oundert het rade-
agr eement sact ,hav ebeennegot iatedwi thfor eigngov ernment s.xxxI twoul dseem t obe
suf fi
cient ,inor dert oshowt hatthet r
adeagr eement sundert heactof1934ar enotanomal ous
i
nchar act er,thatt heyar enott reat i
es, andt hatt heyhav eabundantpr ecedenti nourhi st ory, t
o
refertocer taincl assesofagr eement sher etofor eent eredi ntobyt heExecut ivewi t
houtt he
appr ov al oft heSenat e.Theycov ersuchsubj ect sast hei nspect ionofv essel s,navigat iondues,
i
ncomet axonshi ppi ngpr of i
ts,theadmi ssionofci vilaircraft, customsmat ter s,andcommer ci
al
relati
onsgener ally, i
nt ernational claims, post almat ters, ther egi strat
ionoft r ademar ksand
copy right s, etceter a.Someoft hem wer econcl udednotbyspeci fi
ccongr essi onalaut hor izat i
on
buti nconf ormi tywi thpol i
ciesdecl aredi nact sofCongr esswi t
hr espectt ot hegener al subj ect
mat t
er, suchast ar iffact s;whi lest i
llother s,par ticular l
yt hosewi threspectoft heset tlementof
claimsagai nstfor eigngov ernment s,wer econcl udedi ndependent lyofanyl egi sl
ati
on.
(Emphasi sour s)

Astheforegoingpronouncementindi
cates,t
her egist
rat
ionoftrademarksandcopy r
ight
shave
beenthesubjectofexecuti
veagreementsent er
edi nt
owi t
houttheconcurr
enceoftheSenate.
Someexecut i
veagreementshavebeenconcl udedi nconformit
ywiththepoli
ciesdeclar
edi
nthe
actsofCongresswi t
hrespectt
ot hegeneralsubjectmatter.

I
tthenbecomesr el
evantt
oexami
neourst
atepol
i
cyoni
ntel
l
ect
ual
proper
tyi
ngener
al,
as
r
eflect
edinSect
ion2ofourIPCode,
towit
:

Secti
on2.Decl arat
ionofSt at
ePolicy.-TheSt aterecognizesthatanef fectiv
eintel
lect
ualand
i
ndust r
ialpr
opertysystem isvi
taltothedev el
opmentofdomest i
candcr eati
veactivi
ty,
faci
li
tatestr
ansferoftechnology,att
ractsforeigninvestments,andensur esmar ketaccessfor
ourproducts.Itshal
lprotectandsecur etheexclusiveright
sofscientists,invent
ors,art
ist
sand
othergift
edciti
zenstot hei
rint
ell
ectual pr
opertyandcr eati
ons,part
icularlywhenbenef i
cialt
o
thepeople,forsuchper i
odsasprov i
dedi nthisAct.

Theuseofi nt
ell
ectualpr
opert
ybear
sasoci
alf
unct
ion.Tothi
send, t
heStateshal
lpr
omot
ethe
di
ffusionofknowledgeandinf
ormat
ionf
ort
hepromotionofnat
ionaldev
elopmentand
progressandthecommongood.

I
tisal
sot
hepol
i
cyoft
heSt
atet
ost
reaml
i
neadmi
nist
rat
ivepr
ocedur
esofr
egi
ster
ingpat
ent
s,
tr
ademarksandcopyri
ght,
tol
iber
ali
zetheregi
str
ationonthet
ransferoft
echnol
ogy
;andt
o
enhancetheenf
orcementofi
ntel
l
ectualpr
oper
tyrightsi
nthePhil
i
ppines.

I
nviewoftheexpr essionofst
atepoli
cyhavi
ngbeenmadebytheCongressit
self
,theI
PAPi s
pl
ainl
ymistakeninasser t
ingt
hat"
therewasnoCongressi
onal
actt
hatauthor
izedtheaccessi
on
oft
hePhili
ppinestot heMadri
dProtocol
."
34

Accor dingly,DFASecr et
ar yDel Rosario’
sdet er
mi nati
onandt r
eatmentoft heMadr idPr otocol as
anexecut i
veagr eement ; beinginappar entcontempl at
ionoft heexpr essst atepol icieson
i
nt el
lectualproper t yaswel l
aswi thi
nhispowerunderExecut i
veOr derNo.459, ar euphel d.We
obser veatt hispoi ntthatt herear enohar dandf astrulesont hepr opr i
etyofent eringi ntoa
treatyoranexecut iveagr eementonagi vensubjectasani nst
rumentofi nternational r
elat i
ons.
Thepr imar yconsi der ati
oni nthechoi ceoft heform ofagr eementi st hepar ti
es' intentand
desiretocr aftt heirinternati
onal agreementi nthef ormtheysowi sht of urthert heirr espect i
ve
i
nt er
est s.Themat teroff ormt akesabackseatwheni tcomest oef fectivenessandbi nding
effectoft heenf orcementofat reatyoranexecut iveagreement ;i
nasmuchasal lthepar ties;
regardlessoft hef orm, becomeobl i
gedtocompl yconformabl ywitht het ime- honor edpr inciple
ofpact asuntser v
anda. 35Thepr inci
plebindsthepar ti
estoper for
mi ngoodf aiththei rpar tsin
theagr eement s.36

c.

Ther
eisnoconf
li
ctbet
weent
he

Madr
idPr
otocol
andt
heI
PCode.

TheIPAPalsor estsit
schall
engeonthesupposedconfl
ictbet
weent heMadridProt
ocolandthe
IPCode,contendingthatt
heMadridProtocoldoesawaywitht
her equi
rementofaresi
dent
agentunderSection125oftheIPCode;andt hatt
heMadridProt
ocol i
sunconst
it
uti
onalfor
bei
nginconflictwiththel
ocall
aw,whichitcannotmodi
fy.

TheIPAP'scontent
ionsstandonafault
ypremise.Themet hodofregi
str
ati
onthr
ought he
I
POPHL, aslai
ddownbyt heIPCode,
isdist
inctandsepar
at efr
om themethodofregist
rat
ion
thr
oughtheWI PO,assetintheMadri
dPr ot
ocol.Compari
ngt hetwomethodsofregistr
ati
on
despi
tethei
rbeinggovernedbytwoseparatesystemsofregist
rat
ionist
husmisplaced.

InarguingthattheMadr i
dProtocolconfl
ict
swi t
hSect ion125oftheI PCode, t
heIPAP
highli
ghtstheimportanceoftherequir
ementf orthedesignati
onofar esidentagent.I
t
underscoresthattherequir
ementisintendedtoensur ethatnon-
residententi
ti
esseeking
protecti
onorprivi
legesunderPhil
ippi
neI nt
ell
ectualPropert
yLawswi l
lbesubjectedtothe
country'
sjuri
sdict
ion.I
tsubmitsthatwithoutsuchr esi
dentagent,t
herewi l
lbeaneedt oresor
t
tocostly,t
imeconsumi ngandcumber someext raterri
tor
ial
servi
ceofwr it
sandpr ocesses.
37

TheIPAPmisapprehendst
heproceduref
orexami
nati
onundertheMadr i
dProtocol
,The
di
ff
icul
ty,
whichtheIPAPil
l
ust
rates,i
sminimal
,i
fnotal
toget
herinexi
stent
.TheIPOPHLact
ual
l
y
requiresthedesi gnationofther esidentagentwheni trefusest her egist r
ationofamar k.Local
representationisf ur
therrequir
edi nthesubmi ssi
onoft heDecl arationofAct ualUse, aswellas
i
nt hesubmi ssionoft heli
censecont ract
.38TheMadr i
dPr otocol accor dswi ththeintentand
spiri
toftheI PCode, part
icul
arlyont hesubj ectoftheregistr
ationoft rademar ks.TheMadr i
d
Protocol doesnotamendormodi fyt heIPCodeont heacqui si
tionoft rademar kri
ght s
consi der
ingt hattheapplicati
onsundert heMadr i
dPr otocolarest il
l exami nedaccor dingtothe
rel
ev antnat i
onallaw, I
nthatregar d,theIPOPHLwi llonlygrantpr otect i
ont oamar kthatmeet s
thelocal regist
rati
onr equir
ement s.

WHEREFORE,
thi
sCourtDISMISSESthepeti
ti
onforcer
ti
orar
iandpr
ohi
bit
ionf
orl
ackofmer
it
;
andORDERSt
hepet
it
ionertopaythecostsofsui
t.

SOORDERED.

LUCASP.BERSAMI
N

Associ
ateJust
ice

WECONCUR

Br
otherMARI
ANOMI
KEZ.VELARDE,
Pet
it
ioner
,v.SOCI
ALJUSTI
CESOCI
ETY,
Respondent
.

DECISION

PANGANI
BAN,
J.:

Adecisionthatdoesnotconf ormt otheform andsubst ancerequiredbyt heConsti


tut
ionand
thel
awi svoidanddeemedl egall
yinexi
stent.Tobev al
id,deci
sionsshoul dcomplywit
ht he
for
m, t
heprocedur eandt hesubstanti
verequirementsl ai
doutint heConst i
tut
ion,
theRulesof
Courtandrelevantcir
culars/
ordersoftheSupr emeCour t
.Forthegui danceofthebenchandthe
bar,
theCour therebydiscussesthesefor
ms, proceduresandrequi r
ement s.

TheCase

BeforeusisaPetit
ionf
orRevi
ew1underRul
e45oftheRulesofCour
t,assai
l
ingt
heJune12,
2003Decision2andJul
y29,2003Or
der3oftheRegi
onal
Trial
Court(
RTC)ofManila(
Branch
49).4ςrνl
l

Thechal lengedDeci sionwast heof fshootofaPet i


ti
onf orDecl
arator yRelief
5filedbeforethe
RTC-Mani l
abyher einRespondentSoci alJusticeSociety(SJS)againsther einPet i
ti
onerMar i
ano
MikeZ.Vel ar
de,toget herwit
hHi sEmi nence, JaimeCar dinalSi
n,Execut i
v eMinisterErao
Manal o,BrotherEddi eVill
anuev aandBr otherEliseoF.Sor i
anoasco- respondent s.ThePet i
ti
on
prayedf ortheresolutionofthequest i
onwhet herornott heactofar eli
giousleaderlikeanyof
hereinrespondent s,inendorsingt hecandidacyofacandi dateforelecti
veof fi
ceori nurgingor
requir
ingt hemember sofhi sflocktov oteforaspeci f
iedcandidate,isv i
olati
veoft helett
eror
spir
itoft heconstituti
onal pr
ovisionsxxx. 6ςr νl
l
AllegingthatthequestionedDecisiondidnotcontainast
atementoff actsandadispositi
ve
por ti
on,herei
npet i
ti
onerfil
edaCl ari
fi
cator
yMot i
onandMot ionforReconsider
ati
onbef orethe
tri
al court
.Soriano,hi
sco-respondent,si
milar
lyf
il
edasepar ateMoti
onf orReconsider
ation.I
n
response, t
het ri
alcour
tissuedtheassailedOrder,whi
chheldasf ol
l
ows: ςηαñr
οblεšνιr
†υαl l
αω
l
ιbr αrÿ

xxx[T]hisCourtcannotr econsider ,becausewhati twasaskedt odo,wasonlyt oclari


fya
Consti
tut
ionalprovi
sionandt odecl arewhet heract
sar evi
olati
vether
eof.TheDeci si
ondidnot
makeadi sposit
iveport
ionbecauseadi sposit
iveport
ionisrequir
edonlyincoerciv
er el
ief
s,
wherearedressf r
om wr ongsuf feredandt hebenefi
tthattheprevai
li
ngpartywrongedshoul d
get
.Thest epthatthesemov antshav etotake,isdi
rectappealunderRule45oft heRulesof
Court
,foraconclusiveinterpret
at i
onoft heConstit
uti
onalprovi
siontotheSupremeCour t
.7

TheAnt
ecedentPr
oceedi
ngs

OnJanuar y28,2003, SJSf i


ledaPet i
ti
onforDeclarator
yRel i
ef(SJSPet i
tion)bef
or etheRTC-
ManilaagainstVelardeandhi saforesaidco-r
espondents.SJS, aregisteredpolit
icalparty,
soughttheinterpr
etationofsev er
alconstit
uti
onalprovi
sions,8specifical
lyont hesepar ati
onof
churchandst at
e;andadecl arat
oryjudgmentont heconstitut
ionali
tyoft heactsofr eli
gious
l
eadersendor si
ngacandi dateforanelecti
veoffi
ce,orurgingorrequi ri
ngt hemember softhei
r
fl
ocktov oteforaspeci f
iedcandidate.

Thesubsequentpr
oceedi
ngswerer
ecount
edi
nthechal
l
engedDeci
sioni
nthese
words:
ςηαñrοbl
εšνιr
†υαll
αωlι
brαr
ÿ

xxx.Br o.EddieVi ll
anuev asubmi tted,wi t
hint heori
ginalperiod[t
of ileanAnswer ],aMot i
ont o
Dismiss.Subsequent ly
,Execut i
veMi nisterEraoManal oandBr o.Mi keVel arde,fi
ledt heir
Mot i
onst oDismi ss.Whi l
eHi sEmi nenceJai meCar dinalL.Sin,f
il
edaCommentandBr o.Eli
Soriano,fi
ledanAnswerwi thintheext endedper i
odandsi mi l
arl
ypray edf orthedismi ssal ofthe
Petiti
on.Allsoughtt hedi smissaloft hePet it
ionont hecommongr oundst hatitdoesnotst atea
causeofact ionandt hatthereisnoj ust i
ciablecontrover
sy .Theywer eorderedt osubmi ta
pleadingbywayofadv isement ,whichwascl oselyfoll
owedbyanot herOr derdeny ingall t
he
Mot i
onst oDismi ss.Bro.Mi keVel arde,Bro.Eddi eVill
anuev aandExecut i
veMi nisterErao
Manal omov edt oreconsi derthedeni al.HisEmi nenceJai meCar dinal L.Sin,askedf or
extensiontof i
l
ememor andum.Onl yBr o.EliSorianocompl iedwitht hefirstOrderbysubmi tt
ing
hisMemor andum.xxx.chanr oblesv i
rtual
lawl i
brary

xxxtheCourtdeniedt
heMot i
onst oDismiss,andtheMot i
onsforReconsi
derat
ionfil
edbyBro.
MikeVel
arde,Br
o.Eddi
eVil
lanuevaandExecut iveMini
sterEraoManalo,whi
chraisednonew
ar
gumentsotherthant
hosealreadyconsideredint
hemot ionstodi
smi ssxxx.9ςrνl
l

Afternar
rati
ngtheaboveinci
dents,
thetr
ialcour
tsai
dthati
thadjur
isdi
ctionoverthePetit
ion,
becauseinpray
ingforadeterminat
ionastowhethert
heacti
onsimputedt ot
her espondents
areviol
ati
veofArti
cleI
I,
Secti
on6oft heFundamentalLaw,
[thePet
it
ion]hasraisedonlya
questi
onofl aw.10Itthenproceededt oalengt
hydiscussi
onoftheissueraisedinthePeti
ti
on
thesepar at
ionofchurchandst at
eev entr
acing,t
osomeext ent
,thehist
ori
cal backgr
oundof
theprincipl
e.Throughitsdi
scourse,thecourtaquoopinedatsomepoi ntthatthe
[e]
ndor sementofspecifi
ccandidatesinanelect
iontoanypubli
cofficeisaclearviol
ati
onofthe
separat i
onclause.
11ςrνll

Aft
erit
sessayont hel
egali
ssue,however,
thetr
ialcour
tfai
ledtoi
ncludeadi
sposit
iveport
ioni
n
i
tsassail
edDecisi
on.Thus,Vel
ardeandSori
anofil
edseparateMoti
onsforReconsi
derat
ion
which,
asmentionedearl
ier
,weredeniedbythel
owercourt.

Hence,
thi
sPet
it
ionf
orRev
iew.
12ςr
νll

Thi
sCour t
,inaResol ut
ion13datedSeptember2, 2003,r
equi
redSJSandtheOffi
ceofthe
Sol
ici
torGeneral(OSG)t osubmitthei
rrespect
ivecomments.InthesameResol
uti
on,t
heCourt
gavetheotherparti
es--impleadedasrespondentsintheori
ginal
casebel
ow--t
heopport
uni
ty
tocomment ,i
ftheysodesi r
ed.

OnApr
il13,
2004,
theCour
tenbancconduct
edanOr
alAr
gument
.14

TheI
ssues

I
nhisPeti
ti
on,Brot
herMikeVelar
desubmit
sthef
oll
owi
ngi
ssuesf
ort
hisCour
ts
r
esol
uti
on:ςηαñr
οblεšν
ιr†υαl
lαωlιbr
αrÿ

1.Whetherornott
heDeci
si
ondated12June2003r
ender
edbyt
hecour
taquowaspr
operand
val
id;
chanrobl
esvi
rt
ual
l
awli
brar
y

2.
Whet herornotther
eexistsj
usti
ceabl
econtr
over
syi
nher
einr
espondent
sPet
it
ionf
or
decl
aratoryr
eli
ef;
chanrobl
esvi
rt
uall
awli
brar
y

3.Whetherornotherei
nrespondenthasl
egal
int
eresti
nfi
l
ingt
hePet
it
ionf
ordecl
arat
ory
rel
ief
;chanr
oblesv
irt
ual
lawli
brar
y

4.Whet
herornottheconsti
tuti
onalquesti
onsoughttober
esol
vedbyher
einr
espondenti
sri
pe
forj
udi
cial
deter
mi nat
ion;
chanrobl
esvi
rtual
lawl
i
brar
y

5.
Whet
herornott
her
eisadequat
eremedyot
hert
hant
hedecl
arat
oryr
eli
ef;
and,

6.Whetherornott
hecour
taquohasj
uri
sdi
cti
onov
ert
hePet
it
ionf
ordecl
arat
oryr
eli
efofher
ein
respondent
.15ςrν
ll

Duri
ngtheOr
alArgument
,thei
ssueswer
enar
roweddownandcl
assi
fi
edasf
oll
ows:
ςηαñr
οbl
εš
νι
r†υαl
lαωlι
brαr
ÿ

A.
Procedur
alI
ssues

Di
dthePet
it
ionf
orDecl
arat
oryRel
i
efr
aiseaj
ust
ici
abl
econt
rov
ersy
?Di
ditst
ateacauseof
act
ion?Didr
espondenthav
eanyl egal
standi
ngt
ofi
l
ethePet
it
ionf
orDecl
arat
ory
Rel
ief?
chanr
obl
esvir
tual
awli
brar
y

B.
Subst
ant
iveI
ssues

1.Di
dtheRTCDeci
sionconformtotheform andsubst
ancer
equi
redbyt
heConst
it
uti
on,
thel
aw
andtheRul
esofCourt
?chanrobl
esv
irt
ualawl
ibrar
y

2.Mayrel
igi
ousleader
sli
keherei
npetit
ioner
,Bro.MikeVel
arde,beprohi
bitedfrom endorsi
ng
candi
datesforpubl
icoff
ice?Cor
oll
ari
l
y,mayt heybebannedfrom campaigningagainstsai
d
candi
dates?

TheCour
tsRul
i
ng

ThePet
it
ionofBr
otherMi
keVel
ardei
smer
it
ori
ous.

Pr
ocedur
alI
ssues:

Requi
sit
esofPet
it
ions

f
orDecl
arat
oryRel
i
ef

Sect
ion1ofRul
e63oft heRulesofCour
t,whi
chdeal
swi
thpet
it
ionsf
ordecl
arat
oryr
eli
ef,
prov
idesi
npart
:ςηαñr
οblεšν
ιr†υαll
αωlι
brαrÿ

Secti
on1.Whomayf i
l
epet i
ti
on.-Anypersoni nter
estedunderadeed, will,
contractorother
writ
teninstrument,whoser i
ghtsareaffect
edbyast atute,executi
veorderorregulati
on,
ordi
nance, oranyothergovernment al
regulati
onmay , beforebreachorviol
ati
ont hereof
,bri
ng
anactionint heappropri
ateRegionalTri
alCour ttodetermineanyquest i
onofconst ruct
ionor
vali
dit
yarising,andforadeclarati
onofhisr i
ghtsordut i
est her
eunder.

Basedont hef oregoing,anact i


onf ordecl ar
atoryreliefshoul dbefil
edbyaper soninterested
underadeed, awi l
l
,acont r
actorot herwr i
tt
eninstrument ,andwhoser ight
sar eaf f
ectedbya
statute, anexecut iveorder,aregul ati
onoranor di
nance.Thepur poseofther emedyi sto
i
nt erpretortodet erminethev alidi
tyoft hewritt
eni nst r
umentandt oseekaj udicial
declaration
oft hepar ti
esr ightsordut i
esther eunder .
16Theessent i
alrequisi
tesoftheact ionareasf oll
ows:
(1)t hereisaj ustici
ablecont r
ov ersy ;
( 2)thecont r
ov ersyisbet weenpersonswhosei nterestsare
adv erse; (
3)thepar t
yseekingt her eliefhasal egal i
nt erestinthecontr
ov er
sy ;and(4)thei ssue
i
sr ipef orjudi
ci aldeterminati
on. 17

Just
ici
abl
eCont
rov
ersy

BrotherMi keVelardecont
endst hattheSJSPetiti
onfailedtoal
lege,
muchlessestabli
shbefor
e
thetrialcourt,
thatther
eexist
edaj ustici
abl
econtrov
ersyoranadv ersel
egali
nter
estbetween
them; andt hatSJShadalegal r
ightthatwasbeingviolatedorthr
eat
enedtobev i
olat
edby
peti
tioner.Ont hecontr
ary
,Velardeallegest
hatSJSpr emi sedi
tsact
iononmerespeculati
ons,
conti
ngentev
ents,andhy
pothet
ical
issuest
hathadnoty
etr
ipenedi
ntoanact
ual
cont
rov
ersy
.
Thus,i
tsPeti
ti
onforDecl
arat
oryRel
iefmustfai
l
.

Aj usticiablecontr
oversyrefer
st oanexisti
ngcaseorcont r
ov ersyt hatisappr opri
ateorr ipefor
j
udi cialdet er
minati
on,notonet hati
sconjecturalormer el
yant icipatory.
18TheSJSPet i
tionfor
Decl aratoryReli
effell
shortofthistest.I
tmiserablyfai
ledtoal legeanexi stingcont roversyor
disput ebet weenthepetit
ionerandt henamedr espondentst her ein.Furt
her,thePet i
tiondi dnot
suf f
icientlystat
ewhatspeci fi
clegalri
ghtofthepet i
ti
onerwasv iolat
edbyt her espondent s
therein; andwhatpar t
icul
aractoract softhelatt
erwer einbr eachofi t
srights,thelawort he
Const i
tuti
on.

Aspoi ntedoutbyBr otherEliseoF.Sor ianoinhisComment ,19whatexactl


yhashedonet hat
mer it
edt heat
tenti
onofSJS?Heconf essest hathedoesnotknowt heanswer,becausetheSJS
Petiti
on( aswellastheassai ledDecisionoft heRTC)yiel
dsnothinginthi
srespect
.His
Emi nence,Jai
meCar dinalSin,addsthat ,att
het i
meSJSf i
ledit
sPet i
ti
ononJanuary28, 2003,
theelectionseasonhadnotev enstart
edy et
;andthat
,inanyevent,hehasnotbeenact i
vely
i
nv olvedinpart
isanpol it
ics.

Aninit
iatorycomplai
ntorpet
it
ionfil
edwit
ht het
ri
alcourtshoul
dcontai
napl ain,
conciseand
di
rectstatementoftheult
imatefact
sonwhi cht
hepartypleadi
ngrel
i
esf orhisclai
m xxx.20
Yet,
theSJSPet it
ionstat
ednoultimatef
acts.

Indeed,SJSmer el
yspecul atedorant i
cipatedwithoutfactual moor i
ngst hat,asreli
giousleaders,
thepet i
tionerandhisco-respondent sbelowhadendor sedort hreatenedtoendor seacandi date
orcandi datesforel
ectiveoff i
ces; andthatsuchact ualort hreatenedendor sementwi l
lenable
[them]toel ectment opubl i
cof ficewho[ would]inturnbef oreverbeholdent otheirl
eaders,
enablingt hem t
ocont r
ol t
hegov ernment [;
]21andpos[ i
ng]acl earandpr esentdangerofser ious
erosionoft hepeoplesfaithint heel ect
oral pr
ocess[;
]andr einforc[i
ng]theirbeli
efthatr el
i
gious
l
eader sdet ermi
netheul t
imat er esultofelecti
ons,22whi chwoul dthenbev iolat
iveoft he
separ at
ionclause.

Suchpr emi sei


shighlyspeculati
veandmer elytheoreti
cal,
tosaythel
east.Clear
ly,i
tdoesnot
suff
icet oconstit
uteaj usti
ci
ablecontroversy.ThePet i
ti
ondoesnotevenallegeanyi ndi
cat
ion
ormani festint
entont hepartofanyoft her espondentsbelowtochampionanel ectoral
candidate,ortourgetheirso-cal
ledfl
ockt ov otefor
, ornottovot
efor
,apar t
icul
arcandidat
e.It
i
sat ime- honor
edr ul
et hatsheerspeculati
ondoesnotgi veri
setoanacti
onableright.

Obviously,t
hereisnofactualall
egati
onthatSJSr i
ghtsarebeingsubjectedtoanyt hreat
ened,
i
mmi nentandi nevi
tabl
ev i
olat
ionthatshouldbepreventedbyt hedeclar
at or
yr el
i
efsought.
The
j
udicialpoweranddut yofthecourtstosett
leactualcontrov
ersiesi
nvolvingrightst
hatare
l
egallydemandabl eandenforceable23cannotbeexercisedwhent herei
snoact ualor
t
hreat enedvi
olati
onofalegal r
ight.

Al
lthatt
he5-
pageSJSPet
it
ionpr
ayedf
orwast
hatt
hequest
ionr
aisedi
npar
agr
aph9her
eofbe
resolved.24Inot herwor ds,i
tmer el
ysoughtanopi ni
onoft het ri
alcourtonwhetherthe
speculatedact sofr eli
giousleadersendor
singelecti
v ecandidatesforpoli
ti
caloff
icesv i
olat
ed
theconst i
tut
ionalprinci
pleont heseparat
ionofchur chandst ate.SJSdidnotaskf ora
declarationofitsrightsanddut ies;
neit
herdiditprayf ort
hest oppageofanythreatened
viol
ationofi t
sdecl aredr i
ght
s.Courts,
howev er
,arepr oscr
ibedf r
om renderi
nganadv isory
opinion.25

CauseofAct
ion

RespondentSJSassertsthati
nordertomaintai
napet i
ti
onfordecl
arat
oryreli
ef,acauseof
act
ionneednotbeal l
egedorproven.Supposedl
y,forsuchpet
iti
ontoprosper,t
hereneednotbe
anyviol
ati
onofaright,br
eachofdutyoractualwrongcommi t
tedbyonepar t
yagainsttheot
her
.

Petit
ioner,ontheot herhand,arguest hatthesubjectmat t
erofanact ionfordeclarat
oryreli
ef
shouldbeadeed, awil
l,acontract(orot herwri
tt
eninstrument),astatute,anexecutiveorder,a
regul
ationoranor di
nance.Butt hesubjectmat t
eroft heSJSPet i
ti
oni stheconst i
tut
ional
ityof
anactofar eli
giousleadertoendor sethecandidacyofacandi dateforel ect
iveoffi
ceortour ge
orrequirethemember soft heflocktov oteforaspecifi
edcandidate.26Accor di
ngt opeti
tioner,
thi
ssubj ectmat teri
sbey ondther eal
m ofanact ionfordeclar
atoryrelief
.27Peti
tioneravers
thatintheabsenceofav al
idsubjectmat t
er,
thePetit
ionfail
stostateacauseofact ionand,
hence,shoul dhav ebeendismissedout rightbythecourtaquo.

Acauseofact ioni sanactoranomi ssi


onofonepar tyinvi
olat
ionoft hel egalri
ghtorrightsof
another,
causingi njurytothelatter
.28Itsessenti
alel
ement sar
et hefol l
owing:(1)ar i
ghtin
favorofthepl
ai nti
ff;(2)anobl i
gati
onont hepartofthenameddef endantt orespectornott o
viol
atesuchri
ght ;and( 3)suchdef endantsactoromi ssi
onthatisv i
olati
veoft herightofthe
plai
nti
fforconst i
tutingabr eachoft heobli
gati
onofthef or
mertot hel at
ter.29ςrνl
l

Thefail
ureofacompl ainttostateacauseofact i
onisagroundforit
soutr
ightdismissal.
30
Howev er,
inspeci al
civi
l acti
onsfordeclar
ator
yreli
ef,t
heconceptofacauseofactionunder
ordi
narycivi
lactionsdoesnotst r
ict
lyapply
.Thereasonfort
hisexcepti
onisthatanactionfor
decl
aratoryr
eliefpresupposest hatther
ehasbeennoact ualbr
eachoftheinstrumentsinvol
ved
orofri
ghtsarisingthereunder .
31Nev er
thel
ess,abreachorvi
olat
ionshoul
dbei mpending,
i
mmi nentoratl eastthr
eat ened.

Aper usalofthePet it
ionf il
edbySJSbef or
etheRTCdi scl osesnoexpli
citall
egat i
onthatt he
formerhadanyl egalrightinitsf avort
hati
tsoughttopr otect.Wecanonl yinfertheinterest
,
supposedlyinitsf avor,from it
sbar eall
egati
onthatithast housandsofmember swhoar e
cit
izens-t
axpayer s-
registeredv otersandwhoar ekeenlyinterest
edinajudicialclar
if
icati
onof
theconstitut
ionalit
yoft hepar tisanpart
ici
pati
onofreli
giousl eader
sinPhili
ppi nepolit
icsandin
theprocesst oinsureadher encet otheConsti
tuti
onbyev eryonexxx.32ςr νll

Suchgeneral
aver
mentdoesnot ,
however
,suffi
cetoconsti
tutealegalr
ightori
nter
est
.Notonly
i
sthepresumedint
erestnotper
sonal
incharact
er;
iti
slikewiset
oov ague,hi
ghl
yspeculat
ive
anduncert
ain.
33TheRulesrequi
ret
hattheinter
estmustbemat
eri
alt
otheissueandaff
ected
bythequesti
onedactori
nstr
ument,asdist
ingui
shedf
rom si
mpl
ecuri
osi
tyorinci
dent
ali
nterest
i
nthequestionrai
sed.
34ςrν
ll

Tobol steritsstance,SJSci testheCorpusJur isSecundum andsubmi t


st hatthe[p]l
ainti
ffi
na
decl
ar atoryjudgmentact i
ondoesnotseekt oenforceacl ai
m against[
the]defendant,butseeks
ajudicialdeclarati
onof[ the]r i
ghtsofthepar ti
esf orthepurposeofguiding[thei
r]fut
ur e
conduct ,andtheessent ial dist
inct
ionbet weenadecl ar
atoryjudgmentactionandt heusual
acti
oni st hatnoact ualwr ongneedhav ebeencommi tt
edorl osshaveoccurredinordert o
sustainthedecl arat
or yjudgmentact i
on,althought heremustbenouncer t
aintythatthelosswil
l
occurort hattheasser tedr ightswill
bei nvaded.35ςr νl
l

SJShas, howev er
,ignoredthecrucialpoi
ntofitsownr efer
encethattheremustbeno
uncert
aintythatthelosswilloccurorthattheassert
edr i
ghtswi l
lbeinvaded.Preci
sely,
as
di
scussedear li
er,i
tmer el
yconjecturesthatherei
npetiti
oner(andhisco-respondentsbelow)
mightactivel
ypar t
ici
pateinparti
sanpol i
ti
cs,usetheawesomev oti
ngst r
engthofitsfai
thful
fl
ock[to]enableittoelectment opublicoffi
cexxx, enabli
ng[it
]tocontrolthegovernment.36
ςrνl
l

Duri
ngt heOr alAr
gument ,
though,Peti
ti
onerVelardeandhisco-
respondentsbelowal
lstr
ongly
assert
edt hattheyhadnotinanywayengagedori ntendedt
oparti
cipat
einparti
san
poli
ti
cs.Theyallfi
rmlyassuredthi
sCour tt
hattheyhadnotdoneany thi
ngtotri
ggert
heissue
rai
sedandt oent i
tl
eSJSt otherel
iefsought.

Indeed,theCour tfi
ndsint hePetiti
onf orDeclarat
oryRel iefnosingleall
egati
onoff actupon
whichSJScoul dbasear i
ghtofr el
ieffrom thenamedr espondent s.I
nanyev ent,evengranti
ng
thatitsuff
icient
lyassertedal egalrightitsoughttoprotect ,
therewasnev ert
helessnocer tai
nty
thatsuchr i
ghtwoul dbei nvadedbyt hesaidrespondent s.Notev entheall
egedpr oximityoft
he
electi
onstot hetimethePet i
ti
onwasf il
edbelow( Januar y28,2003)woul dhav eprovidedthe
certai
ntythatithadal egal r
ightthatwoul dbejeopardizedorv iolatedbyanyoft hose
Respondent s.

Legal
Standi
ng

Legalstandingorl
ocusstandihasbeendef i
nedasaper sonalandsubst
anti
alint
eresti
nthe
case,suchthatthepart
yhassustainedorwillsustai
ndir
ectinj
uryasaresul
tofthechall
enged
act.
37Interestmeansamat er
iali
nteresti
nissuethatisaff
ectedbythequesti
onedactor
i
nstrument,asdisti
ngui
shedfr
om amer ei
ncidentali
nter
estinthequest
ioninv
olved.
38ςrνl
l

Petit
ionerall
egest hat[
i]
nseekingdeclarat
oryr
eli
efastotheconsti
tuti
onali
tyofanactofa
rel
igi
ousl eadertoendorse,orrequi
rethemember softherel
i
giousfl
ockt ovoteforaspeci
fi
c
candidate,hereinRespondentSJShasnol egali
nter
estint
hecontroversy
;39ithasfai
ledto
establi
shhowt heresol
uti
onoft heproff
eredquesti
onwouldbenefi
torinjur
eit.
Parti
esbringingsui t
schal l
engingtheconst i
tut
ionalit
yofalaw, anactorastat
utemustshow
notonlythatthel aw[oract ]i
sinvalid,
butalsot hat[t
heyhave]sustai
nedor[ar
e]ini mmediate
orimminentdangerofsust ai
ningsomedi rectinjuryasaresultofi
tsenfor
cement, andnot
merelythat[they]suffertherebyinsomei ndefi
ni t
eway .
40Theymustdemonst r
atet hatt
hey
havebeen, orareaboutt obe, deniedsomer ightorpr i
vi
legetowhichtheyarel
awf ull
yenti
tled,
orthattheyar eaboutt obesubj ectedtosomebur densorpenalti
esbyreasonofthest at
uteor
actcompl ai
nedof .
41ςr ν
ll

Fi
rst,part
iessuingast axpayersmustspecif
icall
yprovethattheyhav esuffi
cienti
nter
esti
n
preventi
ngt heil
legalexpendit
ureofmoneyraisedbyt axati
on.42At axpayer
sact i
onmaybe
properl
ybr oughtonlywhent herei
sanexercisebyCongr essofitstaxingorspendingpower.
43
I
nt hepresentcase, t
hereisnoal l
egat
ion,
whet herexpressorimpli
ed, thatt
axpayersmoneyis
beingil
legall
ydisbursed.

Second, therewasnoshowi nginthePeti


ti
onforDeclarat
oryReli
efthatSJSasapol i
ticalpart
y
oritsmember sasregist
eredvoter
swouldbeadv er
selyaffect
edbyt heall
egedactsoft he
respondent sbel
ow, i
fthequesti
onatissuewasnotresolved.Therewasnoal legat
iont hatSJS
hadsuf feredorwouldbedeprivedofvotesduetotheactsi mput
edt othesaidrespondent s.
Neitherdiditall
egethatanyofitsmember swoul
dbedeni edtherightofsuff
rageort he
priv
ilegetobev otedforapubli
coffi
cetheyareseeki
ng.

Finall
y ,t
heallegedlykeeni
nterestofitsthousandsofmember swhoar eciti
zens-taxpay
ers-
registeredvot er
sistoogeneral44andbey ondt hecontemplati
onoft hestandardssetbyour
j
ur i
spr udence.Notonlyist
hepr esumedinterestimpersonali
ncharacter;
itisli
kewisetoovague,
highlyspecul ati
veanduncertai
nt osatisf
yther equi
rementofstanding.45

Tr
anscendent
alI
mpor
tance

Inanyev
ent,
SJSurgestheCourtt
otakecogni
zanceoft
hePeti
ti
on,ev
ensansl
egal
standi
ng,
consi
der
ingt
hatt
heissuesrai
sedareofpar
amountpubl
ici
nter
est.

Innotafewcases, theCour thasl i


berali
zedthelocusstandirequir
ementwhenapet i
ti
onraises
anissueoftranscendent alsigni
ficanceorpar amountimpor t
ancetot hepeople.46Recentl
y,
aft
erholdingthattheI BPhadnol ocusst anditobri
ngthesuit,theCourtinIBPv .Zamora47
nevert
helessentertainedthePet it
iontherein.I
tnotedthattheIBPhasadv ancedconstit
uti
onal
i
ssueswhi chdeser vetheat tent
ionoft hi
sCour ti
nv i
ewoft heirser
iousness,nov el
tyandweight
asprecedents.48ςr νl
l

Simil
arl
yint heinst
antcase, t
heCourtdeemedt heconstit
utionali
ssuerai
sedintheSJSPetit
ion
tobeofpar amountinteresttotheFil
ipi
nopeople.Theissuedi dnotsi
mplyconcerna
deli
neati
onoft heseparationbetweenchurchandstate,butransmacki nt
othegov er
nanceof
ourcountry
.Thei ssuewasbot htr
anscendent
alinimportanceandnov eli
nnatur
e,sincei
thad
neverbeendeci dedbefore.
TheCour t,thus,call
edforOr alArgumenttodet er
mi newit
hcertai
ntywhetheri tcoul
dresolve
theconstituti
onalissuedespi t
et hebarr
enallegat
ionsintheSJSPet i
ti
onaswel lasthe
abbrevi
atedpr oceedingsint hecourtbelow.Mucht oitschagri
n,however,counselsforthe
parti
es--par t
icul
arlyforRespondentSJS- -madenosat i
sfact
oryall
egati
onsorcl ari
fi
cations
thatwouldsuppl ythedef i
cienciesherei
nabov edi
scussed.Hence,eveniftheCour twould
exemptt hiscasef rom thestri
ngentlocusstandirequir
ement,suchheroi
cef fortwouldbef ut
il
e
becauset het r
anscendent ali
ssuecannotber esolvedanyway.

Pr
operPr
oceedi
ngsBef
ore

t
heTr
ial
Cour
t

Topreventarepeti
ti
onofthi
swast eofpreciousjudi
cialti
meandeffor
t,andfort
heguidanceof
thebenchandthebar,theCourtr
eiter
atestheelement ar
yprocedur
e49thatmustbefoll
owed
bytri
alcour
tsintheconductofci
vilcases.
50ςr νl
l

Prefatorily,
thet r
ial
cour tmay- -motupropri
ooruponmot i
onoft hedefendant--di
smi ssa
compl aint51( orpeti
ti
on, inaspeci
al ci
vi
lacti
on)thatdoesnotallegetheplai
ntiff
s(or
peti
tioner s)causeorcausesofact i
on.52Acompl aintorpeti
ti
onshoul dcontai
napl ain,conci
se
anddi rectstatementoft heulti
matefactsonwhicht hepart
ypleadingrel
iesforhisclaim or
defense. 53Itshouldlikewi secl
ear
lyspecif
ythereli
efsought.54ςrνll

Upont hef i
li
ngoft hecomplaint/
pet it
ionandthepay mentoft herequisitelegalfees,thecler
kof
cour tshallfort
hwi t
hissuethecorr espondi
ngsummonst ot hedef endantsort her espondents,
withadi r
ectivethatthedefendantanswer 55withi
n15day s,unlessadi fferentperiodisfi
xedby
thecour t.56Thesummonsshal lalsocontai
nanot icethatifsuchansweri snotf il
ed,the
plainti
ffs/peti
tionersshal
ltakeaj udgmentbydef aultandmaybegr antedt herel
iefappli
ed
for.57Thecour t,however,
may- -uponsucht ermsasmaybej ust--al
lowananswert obef i
l
ed
afterthet imef i
xedbyt heRules.
58ςr νl
l

I
ftheanswerset
sfort
hacount
ercl
aim orcr
oss-cl
aim,
itmustbeansweredwit
hinten(10)day
s
f
rom ser
vice.
59Arepl
ymaybefil
edwithi
nten(10)daysf
rom ser
viceofthepl
eadi
ngresponded
t
o.60ςrνl
l

Whenananswerf ailst ot enderani ssueoradmi t


st hemat eri
alall
egationsoft headv ersepar t
ys
pleading, thecourtmay ,onmot ionoft hatpar ty,dir
ectj udgmentonsuchpl eading( excepti n
actionsf ordeclarati
onofnul li
tyorannul mentofmar ri
ageorf orlegalseparation).61
Meanwhi le,apar t
yseeki ngt orecov eruponacl aim, acount ercl
aim orcrosscl ai
m- -ort oobtain
adecl aratoryrel
ief--may ,atanyt imeaf tertheanswert heretohasbeenser ved, mov ef ora
summar yj udgmenti ni tsf avor.62Si milarly,
apar tyagai nstwhom acl ai m,acount erclaim or
crossclaimi sasser ted- -oradecl arator yrel
iefsought- -may ,atanyti
me, mov eforasummar y
j
udgmenti nit
sf avor.63Af terthemot ioni sheard, t
hej udgmentsoughtshal lberender ed
fort
hwi t
hi fthereisashowi ngthat ,exceptast otheamountofdamages, thereisnogenui ne
i
ssueast oanymat erialfact ;andt hatt hemov i
ngpar t
yi sentitl
edtoaj udgmentasamat t
erof
l
aw.
64ςr
νll

Withi
nt hetimef or--butbef ore--f
il
ingtheanswert ot hecomplaintorpet it
ion,t
hedef endant
mayf il
eamot i
ont odismi ssbasedonanyoft hegr oundsstatedinSect ion1ofRul e16oft he
RulesofCour t.Duringt hehearingofthemot i
on, t
hepar ti
esshallsubmi ttheirargument sonthe
questionsofl aw, andt heirevi
denceont hequest i
onsoff act.
65Af t
ert hehear i
ng, t
hecour tmay
di
smi sst heact i
onorcl ai
m, denythemot i
on,orordert heamendmentoft hepleadings.Itshall
notdef ertheresol uti
onoft hemot ionforthereasont hatthegroundr eli
eduponi snot
i
ndubi table.I
nev erycase, t
heresoluti
onshallstat
ecl earl
yanddistinctlythereasonst herefor
.66
ςrν
ll

I
ft hemot ioni
sdenied,t
hemov antmayf i
leananswerwi thi
nt hebal
anceoftheperi
odorigi
nal
l
y
prescribedtofi
leananswer,
butnotl essthanfi
ve( 5)daysi
nanyev ent
,computedfrom t
he
receiptofthenoti
ceofthedenial
.Ift
hepl eadi
ngi sorder
edtobeamended, t
hedefendantshal
l
fi
leananswerwi t
hinfi
ft
een(15)days,countedfrom theser
viceoftheamendedpleadi
ng,
unlesst hecour
tprovi
desalongerper i
od.
67ςr ν
ll

Afterthelastpl eadi
nghasbeenser v
edandf i
led,t
hecaseshal lbesetforpretri
al,
68whi chisa
mandat or
ypr oceedi
ng.69Apl aint
if
fs/pet i
ti
oners(orit
sdulyaut hori
zedrepresentat
ives)non-
appear anceatt hepret
rial
,i
fwi t
houtv al
idcause,shallr
esultinthedismissaloftheact i
onwith
prejudi
ce, unlessthecourtordersotherwise.Asimilarfai
l
ureont hepar
tofthedef endantshal
l
beacausef orall
owingtheplainti
ff
/peti
tionertopresentevidenceexparte,andt hecourtto
renderjudgmentont hebasisthereof.
70ςr νll

Thepar tiesarerequi
redtof i
l
et hei
rpretr
ialbrief
s;fai
lur
etodososhal lhavethesameef f
ectas
fail
uretoappearatt hepretri
al.
71Upont het erminati
onthereof
,thecourtshalli
ssueanorder
reci
tingindet ail
themat t
erstakenupatt heconf er
ence;theacti
ontakenont hem, the
amendment sal l
owedtot hepleadi
ngs;andt heagreement soradmissi
ons,ifany,madebythe
parti
esr egardinganyofthemat ter
sconsi dered.
72Thepar ti
esmayf urt
herav ai
lthemsel
vesof
anyoft hemodesofdi scovery,
73iftheysowi sh.

Thereafter
,thecaseshallbesetf ortr
ial
,74i
nwhi chthepart
iesshall
adducetheirr
especti
ve
evi
dencei nsuppor toft
heirclai
msand/ ordefenses.Bythei
rwrit
tenconsentoruponthe
appl
icationofeitherpar
ty,oronitsownmot ion,thecourtmayalsoorderanyoralloft
heissues
tober efer
redtoacommi ssi
oner, whoistobeappointedbyitortobeagreeduponbyt he
part
ies.75Thet r
ialorheari
ngbef orethecommi ssi
onershal
lproceedinall
respect
sasi twoul
d
i
fheldbef orethecourt.
76ςr ν
ll

Uponthecompl eti
onofsuchpr oceedings,thecommi ssi
onershall
fil
ewi t
hthecourtawri
tt
en
repor
tont hemat t
ersref
er r
edbyt hepar t
ies.77Thereportshal
lbesetforheari
ng,aft
erwhich
thecourtshal
lissueanor deradopting,modi fy
ingorrej
ecti
ngiti
nwhol eorinpart
;or
recommi t
ti
ngitwithinst
ructions;orrequir
ingtheparti
estopresentfurt
herevi
dencebefor
et he
commi ssi
onerorthecourt .
78ςr ν
ll
Fi
nall
y,ajudgmentorfinalorderdeterminingthemerit
soft hecaseshallberendered.The
deci
si
onshal l
beinwriting,personal
lyanddi r
ectl
ypreparedbythejudge,stat
ingclearlyand
di
sti
nctl
ythefactsandt helawonwhi chitisbased,
signedbytheissuingmagi st
rate,andfi
led
wit
htheclerkofcourt
.79ςr νl
l

Basedontheseel
ement
arygui
del
i
nes,
letusexami
net
hepr
oceedi
ngsbef
oret
het
ri
alcour
tin
thei
nst
antcase.

Fir
st,
withrespecttotheini
ti
atorypl
eadingoftheSJS.Ev enacursor
yper usalofthePeti
ti
on
i
mmedi atel
yrevealsi
tsgrossinadequacy.I
tcontai
nednost atementofulti
mat efact
supon
whichthepeti
ti
onerreli
edforitscl
aim.Furt
hermore,itdidnotspeci
fythereli
efitsoughtf
rom
thecourt
,butmer el
yaskedittoanswerahy pothet
icalquest
ion.

Reli
ef ,
ascont emplatedinalegalacti
on, r
efer
stoaspecifi
ccoer ci
vemeasur eprayedforasa
resultofav i
olat
ionofther i
ghtsofapl ai
nti
fforapeti
ti
oner.
80Asal readydiscussedearli
er,the
Petit
ionbeforethet r
ialcour
thadnoal l
egati
onsoffact
81orofanyspeci ficvi
olati
onofthe
petit
ioner
sr i
ghts,whichtherespondent shadadutytorespect.Suchdefici
encyamount edt oa
fail
uretostateacauseofact i
on;hence,nocoerci
vereli
efcouldbesoughtandadj udicated.The
Petit
ionevidentl
ylackedsubst ant
iverequir
ementsand,wer epeat,
shouldhav ebeendismi ssed
attheoutset.

Second, withrespecttot hetri


al courtproceedi ngs.Wi thi
nt heperi
odsett of il
et heirrespect i
ve
answer stotheSJSPet i
ti
on,Velar de,Vil
lanuevaandManal of i
l
edMot ionst oDi smi ss;Car di
nal
Sin, aComment ;
andSor i
ano, withinapr i
orl
ygr ant edextendedper i
od,anAnsweri nwhi chhe
l
ikewi sepr ayedfort hedismissal ofthePet it
ion.82SJSf i
ledaRej oi
ndert otheMot ionof
Vel arde,whosubsequent lyfi
ledaSur -Rejoi
nder .Supposedl y,t
herewer esev eralschedul ed
sett i
ngs,inwhi chthe[ c]
ourtwasappr isedoft her espectiveposit
ionsoft hepar t
ies.83The
nat ureofsuchset t
ings--whet herpr etri
alortri
al hearings--wasnotdi sclosedi nt her ecords.
Bef oreruli
ngont heMot ionstoDi smiss,thetri
al courtissuedanOr der84dat edMay8, 2003,
directingthepar ti
est osubmi tthei rmemor anda.I ssuedshor tl
ythereafterwasanot herOr der
85
dat edMay14, 2003, deny i
ngal l
theMot ionstoDi smi ss.

I
nthel
att
erOr
der
,thet
ri
alcour
tper
funct
ori
l
yrul
ed:
ςηαñr
οbl
εšν
ιr†υαl
lαωl
ιbr
αrÿ

TheCour
tnowresol
vestodenyt
heMotionst
oDismiss,
andaft
eral
lthememor andaar
e
submi
tt
ed,t
hen,t
hecaseshal
lbedeemedassubmit
tedforr
esol
uti
on.86ςr
νll

Appar
entl
y,contr
arytotherequirementofSect i
on2ofRul e16oft heRulesofCourt,
the
Moti
onswer enotheard.Worse,t
heOr derpur
portedl
yresol
vingtheMotionstoDismissdidnot
st
ateanyreasonatallfort
heirdenial,incont
raventi
onofSection3ofthesaidRule16.There
wasnotevenanystatementoft hegr oundsreli
eduponbyt heMot i
ons;muchless,ofthel
egal
fi
ndi
ngsandconclusionsofthet ri
alcourt.

Thus,
Vel
arde,
Vil
l
anuev
aandManal
omov
edf
orr
econsi
der
ati
on.Pendi
ngt
her
esol
uti
onof
t
heseMotionsforReconsi derati
on,Vil
l
anuevafi
ledaMot i
ontosuspendt hefi
li
ngoft
heparti
es
memoranda.Butinsteadofsepar atel
yresol
vi
ngt hependi
ngMotionsfair
lyandsquar
ely
,the
t
ri
alcour
tagaintransgressedt heRulesofCourtwhenitimmedi
atelyproceededtoi
ssueit
s
Deci
si
on,evenbeforetacklingtheissuesrai
sedinthoseMoti
ons.

Further more,t
heRTCi ssueditsDecisionwithoutall
owingthepar ti
estofil
et heiranswers.For
thi
sr eason, t
herewasnoj oinderoftheissues.Ifonl
yithadallowedt hefi
li
ngoft hoseanswers,
thetrialcourtwouldhaveknown, ast heOralArgumentr ev
ealed,thatthepetiti
onerandhi sco-
respondent sbelowhadnotcommi t
tedorthreatenedtocommi tt
heactat t
ributedt ot
hem
(endor singcandi
dates)--t
heactt hatwassupposedl yt
hef act
ual basisofthesui t.

Parent
hetical
l
y,t
hecourtaquofur
therf
ailedt
ogiv
eanot i
ceofthePeti
ti
ontotheOSG,which
wasentit
ledtobehearduponquesti
onsinvol
vi
ngtheconst
it
uti
onal
it
yorv al
i
dit
yofst
atut
esand
othermeasures.
87ςrν
ll

Moreover
,aswi
l
lbedi
scussedi
nmoredetail
,thequest
ionedDecisi
onoft
het
rial
cour
twas
ut
terl
ywanti
ngi
nther
equir
ementspr
escr
ibedbytheConst i
tut
ionandt
heRul
esofCourt
.

All
inall
,duri
ngthel
ooselyabbr
eviatedproceedingsofthecase,thetr
ial
courti
ndeedact
ed
wit
hinexpli
cabl
ehaste,
withtot
alignoranceoft helaw--or
,worse,i
ncavali
erdi
sregar
dofthe
rul
esofprocedure-
-andwithgrav
eabuseofdi screti
on.

Contrar
ytothecontent
ionsoft het
rial
judgeandofSJS,pr
oceedi
ngsfordecl
arator
yreli
efmust
sti
llf
oll
owtheprocessdescribedabove--thepet
it
ionmustst
ateacauseofaction;t
he
proceedi
ngsmustunder gotheprocedureoutl
i
nedintheRul
esofCourt
;andthedecisionmust
adheretoconst
it
uti
onal andlegalr
equir
ements.

Fi
rstSubst
ant
iveI
ssue:

Fundament
alRequi
rement
s

ofaDeci
sion

TheConst it
uti
oncommandst hat[n]odecisi
onshallberender
edbyanycour
twi t
hout
expressingther
einclear
lyanddi sti
nctl
ythef act
sandt helawonwhi
chiti
sbased.NoPet i
ti
on
forReviewormot ionforreconsiderati
onofadeci si
onofthecour
tshal
lberefusedduecourse
ordeniedwi t
houtstati
ngt hebasistherefor.
88ςrνll

Consist
entwit
hthisconsti
tut
ionalmandat
e,Sect
ion1ofRul
e36oft
heRul
esonCi
vi
lPr
ocedur
e
si
milarl
yprov
ides:
ςηαñrοbl
εšν ι
r†υαll
αωlιbr
αrÿ

Sec.1.Rendi
ti
onofjudgmentsandf i
nalorder
s.Ajudgmentorfinalorderdetermini
ngthe
merit
softhecaseshallbeinwrit
ingpersonal
l
yanddi r
ect
lypreparedbyt hejudge,st
ati
ng
cl
earl
yanddisti
nct
lythefactsandthelawonwhi chiti
sbased,signedbyhi m andfil
edwit
hthe
cl
erkofcourt
.
I
nthesamevei
n,Sect
ion2ofRul
e120oft
heRul
esofCour
tonCr
imi
nal
Procedur
ereadsas
f
oll
ows:
ςηαñr
οbl
εšν ι
r†υαl
lαωl
ιbr
αrÿ

Sec.2.For
m andcont ent
sofj udgments.--Thejudgmentmustbewrit
tenintheoff
ici
al
l
anguage,personall
yanddirectl
ypreparedbyt hejudgeandsi
gnedbyhim andshal
lcontai
n
cl
earl
yanddistinctl
yastatementofthef actsprovedoradmit
tedbyt
heaccusedandt helaw
uponwhichthej udgmenti
sbased.

x x x.

PursuanttotheConstit
uti
on,t
hisCour
talsoi ssuedonJanuar y28,1988,Admi ni
str
ati
veCircular
No.1, pr
ompt i
ngallj
udgestomakecompl etef i
ndi
ngsoff actsint
heirdeci
sions,andscruti
nize
closel
ythelegalaspect
softhecaseinthelightoftheevidencepresented.
Theyshouldav oi
d
thetendencytogeneral
izeandfor
m conclusionswithoutdetail
i
ngthefactsfrom whichsuch
conclusi
onsarededuced.

Inmanycases,89thisCourthast i
meandt imeagainremi ndedmagistrat
estoheedthedemand
ofSection14,
Arti
cleVIIIoftheConst i
tuti
on.TheCour t
,throughChiefJusti
ceHil
ari
oG.Davi
de
Jr.inYaov.Cour
tofAppeal s,90discussedatlengththeimpl i
cat
ionsofthispr
ovi
sionand
strongl
yexhor
tedthus:ςηαñrοblεšνιr†υαll
αωl ι
brαrÿ

Faithfuladher encet ot herequi rement sofSect i


on14, Ar t
icl
eVI I
Ioft heConst itutioni s
i
ndisput ablyapar amountcomponentofduepr ocessandf airplay .Itisl i
kewisedemandedby
theduepr ocesscl auseoft heConst it
ution.Thepar ti
est oal i
tigat i
onshoul dbei nfor medofhow
i
twasdeci ded, withanexpl anationoft hefactualandl egalreasonst hatledt ot heconcl usi ons
ofthecour t.Thecour tcannotsi mpl ysayt hatjudgmenti srender edi nf avorofXandagai nstY
andj ustleav ei tatt hatwi t
houtanyj ustif
icati
onwhat soev erfori tsact ion.Thel osi ngpar tyi s
entitl
edt oknowwhyhel ost ,sohemayappeal tothehi ghercour t,ifpermi tt
ed, shoul dhe
beli
ev ethatt hedeci sionshoul dber eversed.Adeci si
ont hatdoesnotcl earlyanddi stinctlyst at
e
thef actsandt helawonwhi chitisbasedl eavesthepar t
iesint hedar kast ohowi twasr eached
andi spr ecisel yprejudicialtot helosi ngpar t
y,whoi sunabl et opi npoi ntthepossi bleer r
or sof
thecour tforr ev iewbyahi ghert ri
bunal .Mor ethanthat ,t
her equi rementi sanassur ancet othe
partiesthat ,inr eachingj udgment ,thej udgedidsot hrought hepr ocessesofl egal reasoni ng.It
i
s, t
hus, asaf eguar dagai nstt heimpet uosityofthejudge, prevent inghi mf r
om deci dingi pse
dixi
t.Vouchsaf ednei thertheswor dnort hepursebyt heConst it
ut ionbutnonet helessv est ed
witht hesov er eignpr erogat iveofpassi ngjudgmentont hel i
fe, l
iber tyorpr opert yofhi s
fell
owmen, thej udgemustul ti
matelydependont hepowerofr easonf orsustainedpubl ic
conf i
dencei nt hej ustnessofhi sdeci sion.

I
nPeopl
ev.Bugar
in,
91t
heCour
tal
soexpl
ained:
ςηαñr
οbl
εšν
ιr†υαl
lαωl
ιbr
αrÿ

Therequi
rementthatthedecisi
onsofcour
tsmustbeinwr
iti
ngandt
hattheymustsetforth
cl
earl
yanddist
inctl
ythefactsandthelawonwhicht
heyar
ebasedserv
esmanyf unct
ions.Iti
s
i
ntended,
amongot herthi
ngs, t
oinf
ormthepar
ti
esofther
easonorr
easonsfort
hedecisionso
thatifanyoft hem appeal
s,hecanpoi ntoutt otheappell
atecourtt
hefindi
ngoffact
sort he
rul
ingsonpoi ntsofl
awwithwhi chhedi sagrees.Morethanthat,
therequir
ementisan
assurancet ot hepar
ti
esthat,i
nr eachingjudgment,t
hejudgedidsot hr
oughtheprocessesof
l
egal reasoning.xxx.chanrobl
esv ir
tuall
awlibr
ary

I
ndeed,el
ementar
yduepr ocessdemandsthatthepart
iestoali
ti
gati
onbegi
veninfor
mat i
onon
howthecasewasdecided,aswellasanexplanat
ionofthefact
ualandl
egal
reasonsthatledt
o
theconcl
usi
onsofthecourt.
92ςrνl
l

I
nMadr i
dv.CourtofAppeal
s,
93t hi
sCourthadi nst
ruct
edmagi st
rat
est oexer
teffortt
oensure
t
hatthei
rdeci
sionswouldpr
esentacompr ehensiveanaly
sisoraccountofthefactualandl
egal
f
indi
ngsthatwouldsubst
anti
all
yaddresstheissuesrai
sedbythepar t
ies.

Inthepresentcase, i
tisstarklyobvioust hattheassailedDecisioncont ainsnost atementof
facts--muchl essanassessmentoranal ysisthereof--orofthecour tsfindingsast othe
probablefacts.Theassai ledDeci si
onbegi nswithast at
ementoft henatur eoft heact i
onand
thequest i
onori ssuepr esented.Thenf ollowsabr iefexplanati
onoft heconst i
tutional
provisi
onsinv olv
ed, andwhatt hePet i
ti
onsoughtt oachieve.Thereafter,theensui ngprocedural
i
ncidentsbef orethet r
ialcourtaretracked.TheDeci si
onpr oceedst oaf ull-
lengthopi ni
onont he
natureandt heext entofthesepar ationofchur chandst ate.Withoutexpr esslyst ati
ngthefinal
conclusionshehasr eachedorspeci fyi
ngt herel
iefgrantedordeni ed,thet r
ialjudgeendsher
Decisionwitht heclauseSOORDERED.

Whatwer etheantecedentsthatnecessit
atedthefil
ingofthePet i
ti
on?Whatexactl
ywer et he
disti
nctfactsthatgaverisetothequestionsoughttober esolvedbySJS?Mor eimportant,what
wer ethefactualfi
ndi
ngsandanal ysi
sonwhi chthetri
alcourtbasedit
slegalf
indi
ngsand
conclusions?Nonewer est at
edorimplied.Indeed,t
heRTCsDeci si
oncannotbeupheldf orit
s
fail
uretoexpr esscl
earl
yanddi sti
nctl
ythef act
sonwhi chitwasbased.Thus,thetri
alcourt
clearl
ytransgressedtheconstit
uti
onaldirecti
ve.

Thesigni
fi
canceoffact
ualfi
ndi
ngsliesint hevalueofthedecisionasaprecedent
.Howcanit
besoifonecannotapplyt
herul
ingtosi milarci
rcumstances,simplybecausesuch
cir
cumstancesar
eunknown?Ot her
wi sestated,howwi l
lther
ulingbeappliedi
nthefut
ure,
if
ther
eisnopointoff
actualcompari
son? chanroblesv
irt
ual
awli
br ary

Moreover,thecourtaquodidnoti
ncl
udearesolut
or yordisposi
ti
veport
ioni
nit
sso-call
ed
Deci
sion.Thei mport
anceofsuchport
ionwasexplainedintheearl
ycaseManalangv.Tuason
deRickards,94fr
om whichwequote:
ςηαñr
οblεšνι
r †υαll
αωl ιbr
αrÿ

Theresolut
ionoft heCourtonagivenissueasembodi edinthedisposi
ti
vepar
tofthedeci
sion
ororderi
sthei nvesti
ti
veorcontr
oll
ingfactort
hatdeter minesandsettl
esther
ight
softhe
part
iesandt hequestionspr
esentedtherei
n,notwi
thstandingtheexist
enceofstat
ementsor
decl
arati
onint hebodyofsaidorderthatmaybeconf usi ng.
Theassai l
edDecisi
oninthepresentcaseleavesusinthedar kastoitsfi
nal r
esolut
ionofthe
Peti
tion.Torecal
l,t
heori
ginalPeti
ti
onwasf ordecl
aratoryrel
ief
.So,whatr eli
efdidt
hetri
al
courtgrantordeny?Whatright
soft heparti
esdidi
tconclusivel
ydeclare?Itsfi
nalst
atement
says,SOORDERED.Butwhatexact l
ydidthecourtorder?Ithadthetemer i
tytolabeli
tsi
ssuance
aDeci si
on,whennothi
ngwasi nfactdecided.

RespondentSJSi nsiststhatthedisposi
ti
veport
ioncanbef oundint hebodyoft heassai
led
Decisi
on.Itcl
aimst hattheissueisdisposedofandthePet i
ti
onf i
nallyr
esolv
edbyt he
st
atementoft hetri
al courtfoundonpage10ofi ts14-pageDecisi
on, whichreads:Endor
sement
ofspecif
iccandidatesinanel ecti
ontoanypubli
cof f
iceisaclearviolat
ionoftheseparat
ion
cl
ause.95ςrνl
l

Wecannotagr
ee.

InMagdalenaEstat
e,I
nc.v
.Caluag,
96theobl
igat
ionofthepart
yimposedbytheCourtwas
all
egedl
ycontai
nedinthet
extoftheor
igi
nalDeci
sion.TheCourt
,however
,hel
d:ςηαñr
οbl
εš
νιr
†υαll
αωl ι
brαr
ÿ

xxxThequotedf i
ndi
ngoft helowercour tcannotsupplydefi
cienciesint
hedispositi
vepor t
ion.
I
tisamereopinionofthecourtandt heruleissett
ledthatwheret herei
saconf l
ictbetweent he
di
sposi
ti
vepartandtheopinion,thefor
mermustpr evail
overthelat t
eronthetheorythatthe
di
sposi
ti
veporti
onisthefi
nal orderwhil
et heopini
onismer el
yast at
ementorderingnothi
ng.
(I
tal
i
csintheori
ginal
)

Thus, thedispositi
veportioncannotbedeemedt obethest atementquot edbySJSand
embeddedi nthel astparagraphofpage10oft heassailed14- pageDeci sion.Ifatall
,that
statementi smer elyananswert oahypot
heti
cal l
egalquestionandj ustapar toftheopini
onof
thet r
ialcourt
.Itdoesnotconcl usi
vel
ydecl
arether i
ghts(orobl igat
ions)oft heparti
est othe
Petiti
on.Nei t
herdoesi tgrantany--muchless,theproper--reliefundertheci rcumstances,as
requiredofadi spositi
vepor ti
on.

Fai
lur
etocomplywiththeconsti
tuti
onali
njunct
ionisagraveabuseofdi
screti
onamount
ingt
o
l
ackorexcessofj
uri
sdicti
on.Decisi
onsorordersissuedi
ncarel
essdisr
egardofthe
const
it
uti
onalmandateareapatentnull
i
tyandmustbest ruckdownasv oi
d.97

Par
tsofaDeci
sion

Ingeneral
,theessenti
alpartsofagooddecisionconsistofthefol
lowi
ng:(1)statementofthe
case;(
2)statementoffacts;(3)i
ssuesorassignmentofer r
ors;
(4)cour
truli
ng, i
nwhicheach
i
ssueis,asar ul
e,separat
elyconsider
edandr esolv
ed;and,fi
nal
ly,
(5)di
spositi
veporti
on.The
ponentemayal soopttoincludeanintr
oducti
onorapr ologueaswellasanepilogue,especi
all
y
i
ncasesi nwhichcontroversi
alornoveli
ssuesar ei
nvolved.
98ςrνl
l

Anint
roductionmayconsi
stofaconci
sebutcomprehensiv
estatementoft
heprinci
palfactual
orl
egalissue/soft
hecase.I
nsomecases--par
ti
cular
lythoseconcer
ningpubl
i
ci nt
erest;
or
i
nvolvi
ngcompl icat
edcommer ci
al,sci
ent
ifi
c,t
echni
calorother
wiseraresubjectmatters--a
l
ongerint
r oducti
onorpr ologuemayser vetoacquai
ntreaderswit
hthespecificnatureofthe
contr
oversyandt heissuesi nvol
ved.Anepil
oguemaybeasummat i
onoft heimportant
pri
nci
plesappl i
edtother esoluti
onoftheissuesofpar
amountpubl i
cinter
estorsignifi
cance.I
t
mayalsol aydownanendur ingphil
osophyoflaworguidingpri
nci
ple.

Letusnow,agai
nfort
hegui
danceoft
hebenchandt
hebar
,di
scusst
heessent
ial
par
tsofa
gooddeci
sion.

1.
Stat
ementoft
heCase

TheSt atementoft heCaseconsi st


sofal egaldefi
niti
onofthenat ureoftheaction.Atthef i
rst
i
nstance, t
hispartstateswhet hert
heactionisaci vi
lcaseforcol
lection,ej
ectment ,qui
etingof
ti
tl
e,forecl
osureofmor tgage,andsoon;or,ifiti
sacr i
minalcase,thispartdescribesthe
specif
iccharge--quot edusuallyfr
om theaccusatoryporti
onoft heinformati
on- -andt heplea
oftheaccused.Al soment i
onedherearewhet herthecaseisbeingdeci dedonappeal orona
peti
ti
onf orcert
iorari,thecourtofori
gin,
thecasenumberi nthetrialcourt
,andthedi sposit
ive
porti
onoft heassai l
eddecision.

Inacriminal
case,t
heverbat
imrepr
oducti
onofthecri
minali
nformat
ionservesasaguidein
determini
ngthenat
ureandthegrav
ityoft
heoff
enseforwhichtheaccusedmaybef ound
cul
pable.Asarul
e,t
heaccusedcannotbeconv
ictedofacri
medi f
fer
entfrom orgr
averthant
hat
charged.

Also,
quoti
ngv erbati
mt hetextoft
heinformati
onisespecial
l
yimportantwhenther
eisa
questi
onont hesuffi
ciencyofthecharge,oronwhetherqual
if
yingandmodify
ing
ci
rcumstanceshav ebeenadequatelyall
egedther
ein.

Toensur ethatduepr ocessisaccor ded,


itisimpor t
anttogiveashortdescri
pti
onofthe
proceedingsregar dingthepleaoft heaccused.Absenceofanar rai
gnment ,oraseri
ous
i
rregular
itytherein,mayr enderthejudgmentv oi
d, andfurt
herconsi
derati
onbyt heappel
lat
e
courtwoul dbef uti
le.Insomei nstances,especi
all
yi nappeal
edcases,itwouldalsobeuseful
to
ment i
ont hefactoft heappellantsdetenti
on,inordertodisposeofthepreli
minaryquer
y--
whetherornott heyhav eabandonedt hei
rappeal byabscondingorjumpingbail.

Mentioni
ngthecourtofor
igi
nandthecasenumberor
igi
nal
lyassi
gnedhel
psinfaci
li
tat
ingthe
consoli
dati
onoftherecor
dsofthecasei
nbotht
hetri
alandtheappel
l
atecour
ts,af
terentr
yof
fi
nalj
udgment.

Fi
nal
l
y ,
therepr
oduct
ionoft
hedecr
etal
port
ionoft
heassai
l
eddeci
sioni
nfor
mst
her
eaderof
howtheappeal
edcasewasdeci
dedbythecour
taquo.

2.
Stat
ementofFact
s

Ther
ear
edi
ff
erentway
sofr
elat
ingt
hef
act
soft
hecase.Fi
rst
,undert
heobj
ect
iveorr
epor
tor
ial
method, t
hejudgesummar izes--withoutcomment- -thet esti
monyofeachwi tnessandt he
cont
entsofeachexhi bit.Second,undert hesynthesismet hod,thefactualt
heor yoftheplai
ntif
f
orprosecuti
onandt hent hatofthedef endantordef ensei ssummar i
zedaccor dingtothe
j
udgesbestl i
ght.Thir
d,inthesubj ecti
vemet hod,thev ersionoft hefactsacceptedbyt hejudge
i
ssimpl ynarr
atedwi t
houtexpl ai
ningwhatt hepar t
iesv ersionsare.Final
ly
,througha
combinationofobjecti
veandsubj ectivemeans, thet esti
monyofeachwi t
nessi sreportedand
thej
udget henformulateshi sorherownv ersi
onoft hef acts.

I
ncr iminal cases,itisbett
ertopresentboththeversi
onoft heprosecutionandt hatofthe
defense, intheinterestoffai
rnessandduepr ocess.Adetail
edev aluat
ionoft hecontenti
onsof
thepar t
iesmustf oll
ow.Ther esol
uti
onofmostcr i
minalcases,unli
kecivilandothercases,
dependst oal ar
geext entonthefactuali
ssuesandt heappreciati
onoft heev i
dence.The
plausibil
i
t yortheimpl ausi
bil
i
tyofeachv er
sioncansomet imesbei niti
all
ydr awnfrom areadi
ng
ofthef acts.Thereaf t
er,t
hebasesoft hecourtinarr
ivi
ngati t
sfindi
ngsandconcl usi
onsshould
beexpl ained.

Onappeal,thefactt hattheassail
eddecisi
onoft helowercourtf
ull
y,int
ell
i
gent l
yandcorr
ect
ly
resol
vedallfactual andlegal i
ssuesinv
olvedmaypar tl
yexplai
nwhyt hereviewingcour
tfi
ndsno
reasontoreverset hef i
ndingsandconclusionsoftheformer.Conversel
y,thelowercour
ts
patentmisappreciat i
onoft hefactsormisappl
icati
onofthelawwoul daidinabet t
er
underst
andingofwhyi tsruli
ngisrever
sedormodi fi
ed.

Inappealedcivi
lcases,theopposi ngset soffactsnol ongerneedt obepr esented.Issuesf or
resol
utionusuall
yinvol
vequest i
onsofl aw,graveabuseofdi scretion,orwantofj uri
sdicti
on;
hence,thefactsofthecasear eof tenundisputedbyt hepar t
ies.Wi t
hf ewexcept ions,factual
i
ssuesar enotentertai
nedinnon- criminalcases.Consequently,thenar rat
ionoff actsbyt he
l
owercour t,i
fexhausti
veandcl ear, mayber eproduced; ot
herwise, t
hemat eri
alfactual
antecedentsshouldber est
atedint hewor dsoft hereviewingmagi strate.

I
nadditi
on,t
her easoningoft
helowercourtorbodywhosedeci
sionisunderrevi
ewshouldbe
l
aidout,i
norderthatthepart
iesmayclearl
yunderst
andwhythelowercourtrul
edinacertai
n
way,andwhyther evi
ewingcourtei
therf
indsnoreasont
orever
seitorconcludesother
wise.

3.
IssuesorAssi
gnmentofEr
ror
s

Bothf actualandlegalissuesshoul dbest ated.Onappeal,t


heassignmentofer r
ors,as
ment ionedintheappellantsbrief,mayber eproducedi
ntotoandtackledseri
atim,soastoav oi
d
mot ionsforreconsider
at i
onoft hefinaldecisiononthegroundthatthecourtfai
ledtoconsider
allassignederrorsthatcouldaf f
ectt heoutcomeoft hecase.Butwhent heappellantpr
esents
repetiti
veissuesorwhent heassigneder r
or sdonotstri
keatthemai nissue,t
hesemaybe
restatedinclearerandmor ecoherentterms.

Thoughnotspecif
ical
lyquest
ionedbyt hepar
ti
es,addit
ional
issuesmayal sobeincluded,i
f
deemedimportantforsubst
anti
aljust
icetoberendered.Notethatappeal
edcri
mi nalcasesar
e
gi
vendenov orevi
ew,i
ncont r
astt ononcri
mi nalcasesinwhichtherevi
ewingcour
tisgeneral
ly
l
imitedt oissuesspeci
fical
lyr
aisedintheappeal .Thef ewexcepti
onsareerr
orsofjur
isdi
cti
on;
questionsnotr ai
sedbutnecessar yi
narri
v i
ngataj ustdecisi
onont hecase;
orunassi
gned
err
orst hatareclosel
yrelat
edtot hoseproper l
yassigned,oruponwhichdependsthe
determinationofthequesti
onpr operl
yraised.

4.
TheCour
tsRul
i
ng

Thispartcontainsaf ulldi
scussi
onoft hespeci f
icerrorsori ssuesrai
sedinthecompl ai
nt,
peti
ti
onorappeal ,asthecasemaybe; aswel lasofot herissuesthecourtdeemsessent i
altoa
j
ustdisposit
ionoft hecase.Wheretherear eseveralissues, eachoneofthem shouldbe
separat
elyaddr essed, asmuchaspr acti
cable.Ther espectivecontenti
onsofthepar t
iesshoul
d
al
sobement ionedher e.Whenpr oceduralquestionsar eraisedinaddit
iontosubstanti
veones,
i
tisbettertoresolv etheformerprel
iminari
ly.

5.
TheDi
sposi
ti
onorDi
sposi
ti
vePor
ti
on

Inacriminalcase,thedispositi
onshouldincludeaf i
ndingofinnocenceorguilt
,thespecif
ic
cri
mecommi t
ted,t
hepenal tyimposed,thepar t
ici
pati
onoftheaccused, themodifyi
ng
cir
cumst ancesifany,andtheci vi
ll
iabi
li
tyandcost s.I
ncaseanacqui t
tali
sdecreed,thecourt
mustor dertheimmedi atereleaseoftheaccused, i
fdetai
ned,(unl
esst heyar
ebei ngheldfor
anothercause)andor derthedi r
ectoroftheBureauofCor r
ections(orwherevertheaccusedis
detai
ned)t orepor
t,withi
namaxi mum oft en(10)daysfrom noti
ce,theexactdatewhent he
accusedwer esetfree.

Inacivi
l caseaswel lasinaspeci alci
vilacti
on,thedispositi
onshoul dstatewhet herthe
complaintorpet i
ti
onisgr antedordenied,thespecifi
cr eli
efgranted,andthecost s.The
fol
lowi
ngt estofcompl et
enessmaybeappl i
ed.Fir
st,theparti
esshoul dknowt hei
rr i
ght
sand
obli
gati
ons.Second, theyshoul dknowhowt oexecutet hedecisionunderal ternat
ive
conti
ngenci es.Thi
rd,
t her
eshoul dbenoneedf orfurt
herpr oceedingstodisposeoft hei
ssues.
Fourt
h,thecaseshoul dbet erminat
edbyaccor dingthepr operreli
ef.Thepr operrel
iefusuall
y
dependsuponwhatt hepar tiesseekintheirpl
eadings.Itmaydecl ar
etheirrightsandduties,
commandt heperf
ormanceofposi t
iveprestati
ons,oror dert
hem t oabstainf r
om specif
icacts.
Thedisposi ti
onmustal soadj udi
cat
ecost s.

Theforegoingpartsneednotalway sbedi scussedinsequence.Buttheyshoul


dallbepresent
andplainlyi
denti
fiabl
einthedecision.Dependingonthewrit
erscharacter
,genr
eandstyle,the
l
anguageshoul dbef r
eshandf r
ee- f
lowing,notnecessar
il
ystereot
ypedorinafixedfor
m; much
l
esshighf al
uti
n,hackneyedandpr etenti
ous.Atallt
imes,however,t
hedecisi
onmustbecl ear,
conci
se, completeandcorrect
.

SecondSubst
ant
iveI
ssue:

Rel
i
giousLeader
sEndor
sement
ofCandi
dat
esf
orPubl
i
cOf
fi
ce

Thebasicquestionposedi ntheSJSPet i
ti
on- -WHETHERENDORSEMENTSOFCANDI DACI ES
BYRELI GI
OUSLEADERSI SUNCONSTI TUTIONAL- -undoubtedlydeserv
esseriousconsiderati
on.
Asst at
edearli
er,theCourtdeemst hi
sconstitut
ional i
ssuetobeofpar amountinter
esttot he
Fi
li
pinociti
zenry,f
oritconcernsthegovernanceofourcount r
yandi t
speople.Thus,despitethe
obviousproceduraltr
ansgressi
onsbybot hSJSandt hetri
alcourt,t
hisCourtsti
ll
call
edf orOral
Argument,soasnott oleaveanydoubtt hattheremi ghtberoom toentert
ainanddisposeoft he
SJSPet i
ti
onont hemer i
ts.

CounselforSJShasut t
erl
yfail
ed,
however
,toconvi
ncetheCourtthatt
her
eareenoughf act
ual
andlegalbasestoresol
vetheparamounti
ssue.Ontheotherhand,t
heOff
iceoft
heSolicit
or
Generalhassidedwithpet
iti
oneri
nsof
arastherear
enof act
ssupporti
ngt
heSJSPetit
ionand
theassail
edDecisi
on.

Wer eiteratethatthesaidPeti
ti
onfai
ledtostat edi
rectl
ytheul t
imatefactst hatitreli
eduponf or
i
tscl aim.Dur i
ngtheOr alAr
gument,counselforSJScandi dlyadmitt
edt hatt herewer eno
factual al
legati
onsinit
sPet i
ti
onforDeclarat
oryRelief
.Neitherweretherefact ualfindingsinthe
assailedDeci si
on.Atbest,SJSmerelyaskedt hetri
alcourttoanswerahy pot heti
cal quest
ion.In
effect,i
tmer elysoughtanadv i
sor
yopinion,t
her endit
ionofwhi chwasbey ondt hecour ts
const i
tuti
onal mandateandjuri
sdi
cti
on.99ςrνll

I
ndeed,theassailedDeci
sionwasr
enderedinclearvi
olati
onoft heConst
it
ution, becauseit
madenof indi
ngsoff act
sandfi
naldi
sposi
tion.Hence,iti
sv oi
danddeemedl egallyi
nexist
ent
.
Consequently
,therei
snothingf
ort
hisCourttorevi
ew,af f
ir
m, r
ev er
seorevenj ustmodi f
y.

Regrett
ably,iti
snotl egal
lypossiblefort
heCour tt
ot akeup,onthemer i
ts,theparamount
questi
oninv olv
ingaconst it
utionalpri
nci
ple.Iti
satime-honoredrulet
hattheconst i
tut
ional
i
tyof
astatute[oract]willbepassedupononl yif,andtotheextentthat,
iti
sdirectl
yandnecessaril
y
i
nvolvedinaj usti
ciablecontroversyandisessenti
altotheprotect
ionofther i
ght
soft heparti
es
concerned.100ςrνll

WHEREFORE, t
hePeti
tionforRevi
ewofBrotherMikeVelardeisGRANTED.Theassail
edJune12,
2003DecisionandJuly29,2003Orderoft
heRegi onalTr
ialCour
tofManil
a(Branch49)are
herebyDECLAREDNULLANDVOI DandthusSETASI DE.
TheSJSPetit
ionf
orDeclarator
yRel
ief
i
sDI SMISSEDforfai
l
uret ostat
eacauseofaction.

Letacopyoft hi
sDeci si
onbef urnishedt
heOf fi
ceoftheCourtAdmini
str
atortoev
aluateand
recommendwhet herthetr
ialj
udgemay ,
afterobser
vingdueprocess,
beheldadmini
strat
ivel
y
l
iableforrender
ingadecisionv i
olati
veoftheConsti
tuti
on,t
heRulesofCourtandrel
evant
cir
cularsofthi
sCour t
.Nocost s.

SOORDERED.

You might also like